You are on page 1of 243

1/25/22, 10:24 AM

Chapter 12 - Introduction to Transfer Taxat ion

CHAPTER 12
INTRODUCTION TO TRANSFER TAXATION
Chapter Overview and Objec~ves .
Thi chapter provides an overview of transfer taxation.
--------------------- ---------- ---- -- -- --- --- --- .......
After this chapter, reader are expected to comprehend: •·-...
1. Th concept of transfer and its types
2. The different transfer taxes
3. The concept of a complex tran fer
4. The rationalization of transfer taxation
5. The nature of transfer taxes
6. The types of transfer taxpayers and their scope of taxation
7. The reciprocity rule on non-resident aliens
8. The situs of properties for purposes of transfer taxation
9. The rules on timing of valuation of transfers
10. The difference between donation inter-vivo and mortis causa
11. The different non-taxable transfers and th ir nature
12. The concept of completion of tran fer

WHAT IS TRANSFER?

Transfers refer to any transmission of property from one per on to another.

A person may be a natural per on such a individuals or a juridical perso


created by law such as corporation, partnership or joint ventures.
Types of transfers:
1. Bilateral tran fers
2. Unilateral transfers
3. Complex transfers

BILATERAL TRAN. FERS


Bilateral tran ifers involve transmis ion of prop r for a con ideration.
They are referred to as onerous transactions or ex hanges.
Examples:
1. ale - exchange of property for money
2. B rter - exchange of property for anoth r prop rty

1/30
1/25/22, 10:24 AM

f
o d u c ti o n to Tra ns r Taxation
r 12 - In tr
r, 3pte

Sl lb lo ,, .
. - - T!Jts. Is- - -~ '-to
/luc·'•,·n<;•ss lo x
· -- - - 1
- No ta x

Income tax

AN. FERS
w4T£RAl TR-r . o lv h n r y by a p u
rson
Iii 1 ,era! tran 'I r. . m v tran missio o p ro p rt
1 .d r t,on. Th y re commonly fe rr e d to
as g ra tu it o
fl un ut n , • r
o r.
,r,ch-a tion o r imp1y, transfe
cr,1n:.i i er taxes'.
to tr n s f e r p ro p e rt s is s u b je c t to "transf
fht' ri h t or privil g
r a l Tran
fers
te
rype of Unila d
p ro p e rt y firom 1-1v•mg o n o r. to a
J Donation n fer o f a t·
th e g r a tu it o u s tr e rs o n it 1·5 called "uona 10n inter
" is
· •oonatio·n · 1s in g p
t
· m ad e b tw e e n liv ,
m c e 1
donee.
vivos.
d
p e rt ie s o f the decease
ucces io n g r a tu it o u s
tran fe r o f the p ro
- 2.
•success io
per on u p o
n"
n
is
h is
th
d
e
e a th to h is he
irs.
c lu d in g p r o prietary rig
hts are
his
h is le g a l id e n ti ty in o u s ly tr a nsferred to
s o n dies, b e gratuit en will.
When a p e r His p r o p e r ti e s w il l r b y v ir tu e o f a writt d by
extingui he
d. f la w o ent cause
h e r b y o p e r a ti o n o p e rt ie s o f the deced
e it th e p ro
succes o r d o n a ti o n o f a ll mortis causa
.
cce s io n is a d d o n a tio n
Su ll e
n c e , it is c a
his death. H e ausa
iv a s a n d mortis c
etw e e n in te r -v
Comparis on b
Int - ivosr Decedent
Livin lnvolunta
eror Vol Death
Trans s o f the
oture Gratui All p ro p e rt ie eath
d td
ason O n ly prop
ertie selecte decedent a
ifer or Estate
Scope o f the
tr a n b th e d o n
Gift Heir
£state tax
Done
Date o f death
Donor's tax
a ti o n
tax Date o f d o n
ation
'ming o f valu
donalion
417

2/30
1/25/22, 10:24 AM

cJli n
i11 tr n t 1, n f r
Ir 1

,nt onsld ration?


rn titul<' . •1n acfr•q •t rul on wh t con titut
11 t' I 111,rnt1t,1 I h th r
l ,1 •rn1111 .1tion w r n t a c
n. id rati n of th ac nd th

f th pric i influenced by the liquidity o_r the availability o


th on erned pr perty. H n e, a d1scoun of 200 f
d like gold would be con trued a gift du e to its re/'0 .
· e 11 mg
not be the a em · b1g
· rea I estates. ati\te

Ta rol on transfer. for adequate consideration


Tr n I or d qu ce con id ration are deemed pure exchanges and are
u l to income tax, not to transfer tax.

Tran fer for /es than adequate and full consideration


T n rs for less than full and adequate consideration are plit into its
compon nts: transfer element and exchange element. The transfer element
1s ubJ ct to transfer tax while the realized gain on the exchange element is
sub1e to income tax.

mustration
ume a property with a fair value of PS0,000 and tax basis of Pl0,000 is sold
or merely P30,000.

Fair value
P 50,000 to
Gratuity (indirect donation)
Consideration or selling price 20,QQQ Transfer tax
P 30,000
Les : Cost or tax basis
ReaJized gain
=~~ --+ Income tax
!~e transfer element is genera!I c .
it is a donation mortis-causa if· Y onsidered as an inter-vivos donation, but
a. the sale i mad • ·
b 1·r . e in contemplatio f h
• title to the property · °
n t e death of the seller or
seller. is agreed to be transferred upon the death of the

----------
3/30
1/25/22, 10:24 AM

d ction to Transfer Taxation


intro u
1Z ....
,Pee' 'f.RAN FER TAXATION
~ 1J of n,initnizati on th ~ory
.tJO joll r
,A' . c\/3· th ory
11 r,l
fl' •f.l ., 11P'
1 , f{f ri ·d theory
. •f.l·,,t'Ot rec rshiP th eo ry
• ~t' rtll ·
3 ~,,,1c p:1redi ·tribution theory
• ,vc,11th .,~y th •ory
, 'Ji! LO r
· 1\ 111 • nor Minimizat ion Theory . . _ _
, E 3 10 b intentional ly pric d to evad or mm1_rmze ~nco~e taxes.
13
f~c gc · Jlltl . in an exchange is actually a lost gain which will evade
·llJ', don t1 n tax loophole, th governm nt su~jects t h e gratmty . to
f\ 1 ill jjrcct this
f~c 01 ,. r pl_u~ t taxed in the absence of donativ intent on the part of
,11• r it I whno the sale is made in he normal course o f b usmess. .
t.l .' tlO' c ' n
(.I~· uer uch a
he c .
1 coupment Theory
t,e fa;( Re d liberate in ten to evade mcome tax, transfers have a
fgre11 w1·chout af decreasing future income tax co 1lect10ns · o f t h e governme n t -
ral effect o
113tU
[IIU trati:P p10,ooo,ooo properties which earn 10% ~r p1,.ooo,ooo yearly
,tji on ha . •ng to make his 5 children become financially independe nt, he
e Desin · · p 2, 000 ,000
1nc~rn · h. entire propertie to them . Each child received
dil'lded . •~ach child earns roughly P200,000 on the donated properties.
properties.
that the split of the properties and th spread of the income to several
ia:teayers will re ult in less r ~ax ~~llection to the governmen t because ~f the
r!re sive tax impo ed upon md1v1duals. The same effect would result 1f Mr.
Alis~n transfers his property to his children through succession.

To recoup on future losses in income taxes caused by transfers, the


government taxes the transfer of the properties .

The Benefit Received Theory


~ hen a per on transfers property by donation or succession , the
government is a party in the orderly transi r of the property to the donee or
heir. This is made possible by governmen t laws which enforce or effectuate
onation and succession.

The transferor is actually exercising a privil ge to transfer his property


,~:::ove~n ment s curity of an effective and orderly tran mis ion under
the tr which define and effect donation or ucce ion. Without these laws
ansfer could not have been convenient ly possible. '
Exercising thi . . .
lllorus c ~ special privilege to transfer property either inter vivos or
received~~sa is a hen fit to the trans eror. In accordanc e with the bene 1t
eory, the tran fer hould be taxed.
419

4/30
1/25/22, 10:24 AM

tion
cti on to Tra ns fe r Taxa
Chapter 12 - In tro du
.
an t ra tio na l' t1o
n fit r ce iv ed th eo ry is th e m os t do m in iza n
Th eb Of tt;i
ta ati on . Tlsfe

Th e St at e Pa rt ne rs hi
p Th eo ry
y wh
a civ ili ze d an d or de rly oc iet
en su re ,:r e col})tlie
The sta te
al th ac cu mu la tio n flo ur ish . Th go ve rn rer :;~1
al th ac cu mu lat io n e; t the 0 Is
un de rta ki ng an d w
hi nd all fo rm of we
an in di re ct pa rtn er be rt or th Y any Pers
us , wh en a pe rs on tra ns fe rs pa
wi th in th e tat e. Th ;h o/ e ofh~~
sh ou ld ta ke its fa ir sh ar e by taxing th
we alt h, th e go ve rn me nt transfer ;
rs on s.
th e we al th to ot he r pe

W ea lth Re di s tr ib ut
io n Th eory
as an eleme
ib ut io n of we al th is wi de ly ac ce pt ed d7t o_fsoeial
Eq ui tab le di str in wealth
pr og re s an d sta bi lit y.
ted
oc
wi
iet
th
ies
hi
wi th hig h in qu iti es
gh so cia l un re st, /awl ne ss
'
in itr1b11 tio11
rge ncies,
ar e no rm a/ Iy as so cia
w ar , an d chaos.
n a a .
Thus, go ve rn me nt s str iv
mm
e to
on
wa
too
rd
l in
qu
re
ita
di
b/ e w alth di str ib ut io
str ib ut in g we Ith to oc
iety. w~: 1r
0
policy. Ta xa tio n is a co tra n fer ho ul d be t ed o th t part of th
al h, th e e
on e tra n fers hi s w to be n fit soci ty.
ll be re di str ib ut ed
we al th wi
ry
Ab ili ty to Pa y Th eo at he could no t aff or d.
Th ability to
to us ly gi ve wh transfe
o on e co ul d gr at ui n of an bi lit y to pa y tax. Hence, the
in di ca tio
tra ns fer pr op er ty is an
is su bj ec t to tax.
E TW O TYPE OF TR
ANSFER TAX
COMPARISON OF TH
Do no r's ta x
Sub ·ect tra ns er
ln ter -vivos
n ·-time Lax J
An nual tax _- t - - -- - :D 'cede nt= -- -: -- -
Na tu re -- -, f- -- -- -: :D-o_n_o_r_ _
a =--e-r-
;.:.::.;.~
~ Ti....:ax · · trc ror or heirs
do no r him elf. Exe ut or -
Th e decedent.
Who ac tu all y pay in b
th e tax?
ER TAXES
NA TURE OF TRANSF . /t
x penaltJ:1 ~~ -a
1. Privileg e ta tax r th er tha n form
o
Tr an sfe r tax i a a fo rm
the
of pr ivi l ge
tra ns fe ro r (d on or or
d d nt ) i . er~;;ung
po se d b ca us e go v rn me nr in e e
is im
as sis tan ce rend re d by th e
privilege in the form of ion.
ns fer of pr op er tie s by
way of do n tion or u
the tra rtie
Ad va lo re m tax of the proP:er co
2.
is de pe nd en on th v lu in or
The am ou nt of tra n fer
tax
i n de d
us, va lua tio n of the pr op er ty tra n fe rr d
tra n £erred. Th
of th tax.
determine the am ou nt

5/30
1/25/22, 10:24 AM

2 _ introduction to Transf r Taxation


rer 1
cniJP
110 nal ta
prCIPo~ rax, u ndt•r th THAIN I w re impo. d at fl.it 6% of h • n, c tale
• 111 .,rcr
fr•
,,fl.
I I
ar1or1, I r. 1, Y th national gov rnm nt. Local gov rnmen
,. 1cr tax
r,,,,1~ u J all lu<l d rom irnpo ing th • ·ame.
LJlll( ( l, I •

1111 ot b hiftcd. The transferor-donor or transferor·


n subj t to tax.
Of

re I vied to rai e money for the support of the government


al
II]
OF TRANSFER TAXPAYERS AND THEIR EXTENT OF
LA fff TIO
s,
~ TJON
Re jdents or Citizens - such s:
C 1' a. Re ident citizens
ll
b. Re ident alien
e c. on-resident citiz ns
These are taxable on global transfers of property.

2. on-resident Aliens
These are taxable on Philippine transfers of property.

The citizenship of juridical per ons i determined by the incorporation tests.


Juridical persons that are organiz d in the Philippines are considered Philippine
citizens. Those organized abroad are considered aliens.
In donor's taxation, the term resident citizen or alien include domestic or
resident foreign corporation. Obviously, corporations are not subject to estate
taxation.

SlTUS OF TRANSFER
Transfers occur in the location of the property.
Properties are transferred mortis causa in the place where they are located
at the point of death. They are not transferred at the pla e wher the
decedent died.
Likewise, properties are tran ferred jnter-vivos in the plac where they are
~cated at the date of donation. They are not transfi rr d at th e pJace where
e donor executed the deed of donation.

421

6/30
1/25/22, 10:24 AM

Cha pt r 1 - Int roduction to Tr nsf r Taxa t ion

E · mple:
1 A r(' icknt • I n " ho has P10 M prop rtl in th Phil'
propc•rti in J.ipan di din an airp l~ n ra h in Malaysia. 'PPl n atid P
The P 10 I prop rt( I. d med transferred mortis c
" h1/e lh P40M prop rtie i al o d emed tran iferred m:~:~ In the Philt
is causa in PP111
io

n n-r id nt Fi lipino donated his ca .


\'\ 1ilt> in Kor "• }QPQn,
P ,00 . 00 t hi Arn ri an best riend. r In Japan
"-otui
Th P 1 ar i d m d tran iferr d inter-vivos In japan.

GE ERAL RULE IN TRANSFER TAXATION


___ Ta~'l,.\'ecs
Inter-vivos
t or citize ns Global dona ti on
ident alie n Phi li ine d o nation

lllu tration 1
r. ario, an American re iding in the Philippines, donat d a car in Mexico toa
friend and a motorbike in the Philippines o his brother in America.
ince the taxpayer is a resident, both the donadon of a car abroad and die
donation of a motorbike in the Philippines are subject to transfer tax. Since the
donor is living, the transfers are donations inter-vivos subject to donor's tax.
Illustration 2
Juan, a non-resident Filipino citizen, died leaving a building in the United States
and an agricultural land in the Philippine for hi heirs.
Since the taxpayer is a citizen, the transfer mortis causa of the building in the US
and the agricultural land in the Philippine i ubject to Philippine estate tax.

Illustration 3 din
Mr. Kounoman, a Japanese citizen residing in Japan, donated a par~el of la~ res
Japan to a resident Filipino friend. He also donated his investment m the 3
of stocks of a Philippine corporation to his Japanese i ter.

Since the donor is neither a Philippine resident nor a citizen, only the don:::~:~
domestic shares of stock in the Philippines is subject to transfer tax. _Also: ~er-vivas
donor is living at the date of donation, the transfer is a donatwn ,n
subject to donor's tax.

7/30
1/25/22, 10:24 AM

'f
ci,s P~r
12 _ introduction to Transfer Tax at·ion

tton4 ..
,l111 ~ , ng a Chinese ct izen residin i·n H
II r· yyO I • • ong Kon d'1 d
~'· 'J{ong and a car m the Philippines. g, e leaving a building ln
J{ortB
or is neither a resident nor a citizen O 1
ffie donto transfer tax. Since the transfer Is . !ffin [Y !he car in the Philippines is
svbf~causa subject to estate tax. e ecte by death, it is a donation
"'ofCI
pfJtTIES LOCAT~D IN THE PHILIPPINES I.
pflOfolloWing properties are considered located in the PhT .
1lle . . i 1ppmes:
Interest in a d~me_tic business
1. a. Share~, obhgati~ns, or bonds. issued by any corporation or sociedad
anon~ma organized or constituted in the Philippines in accordance
with its laws
b. Shares or rights in any partnership, business or industry established
in the Philippines
foreign securities, under certain conditions:
2· a. Shares, obligatjons, or bonds issued by any foreign corporation 85%
of the business of which is located in the Philippines
b. Shares, obligations, or bonds issued by any foreign corporation if
such shares, obligation , or bonds have acquired business situs in
the Philippines
3. franchise exerci ble in the Philippines
4, Any personal prop rty, whether ta ngible or intangible, located in the
Philippines
p I U EO N ID
The intangible person I propertie of non-resident ~liens_ are exem~t fr_om
Philippine tran fer ta es provided tha the country m ~h1ch su~? ~hen 1s a
citizen also exempt th intangible personal properties of Fihpmo non-
residents thcr in rom tran fer tax s.

ampl f intangibl properti : z. A counting intangi ble a sets


1. Financi I ass t a. Patent
a. Cash b. Franchise
b. Rec ivabl . or credit c. Lea ehold right
c. Inv ·tment m bond d. Copyright
d. hares ot ·tock in .t corporation e. Trademark
e. lntere t rn a p,1rtn r ·hip
. h) are mere representatjon of
It b II and ·oms (1.e., cas
mu t b pomt ,d out th.tt I s than tan 1ble as ets.
• l n , 1bles rat I1cr
Pu re ha 111 po\n•r. Tht..•V .irt• 111 a g

423

8/30
1/25/22, 10:24 AM

Chapter 12 _ In
tr od uc tio n to Transfer Taxatio
n
Illustration 1
Mr. Sh .n o Ja pa ne se ci tiz en
1 , , do na te d th e
Ph ili pp in e : folloWing
1. Car Pr op et ti
2. Ca h In ba nk es 11\
3. h r s of to ck
of a do m st ic co
rp or at io n
U nd er Japan se
la w s, no n- re si
in ta ng ib le pr op d nt Fi lip in os
rt ie in Japan. ar e ex em pt on
ince the reciprocity transfe
exemption applies, l's llf
the donation o f th M r. Shino is subject
e car. The donatio to dono ,
cash and shares o n o f the intangible
f stock are exempt personal pro;esr~ax
011/y 0
Illustration 2 es Sllch ~
A ss um in g th
am e da ta in th e
le av in g th os e pr pr ec ed in g pr ob
op er ti es in th e l m , ex ce pt th at
in ta ng ib le pr op Ph ili pp in es . T he Ja Mr h·
er ti es of no n- r pa ne s go ve rn m
si de nt Fi lip in o e~ t d tno died
th re on to es ta te
Only the tangible pr tax. 0
no t~
operty - car would be subj
ect to estate tax.
Illustration 3
Mr. Park, a K or
ea n ci tiz en re si
P3M in te re st in di ng in th e Ph ili
a bu si ne s an d pp in es , di ed leav
a PlOM co nd o ing PSM c b
K or ea n la w s, Fi

All of these will be


lipino no n- re si de
nt s th er ei n ar e
un it in th e Phili
ex em pt fr om tr an
ppines. u:e~
sf er taxation.
subject toestate ta
ns to the exclusion x ince reciprocity exemption applie
non-resident alie
of resident aliens. s only to
CLASSIFYING D
ONATION AS INTER-VIVOS O
T he ti m in g of R MO
th e gr at ui to u
pr op er ty to an ot transfer o f own RTIS CAU A
he r de te rm in es ership or lega l
th e cl as si fi ca ti on title over
of th e ra ns fe r.

U on de at h of th
e de ce de nt
If ow ne rs hi p ov
er pr op er ty is vo
lifetime, th is is lu nt ar ily tr an sf er
do na tio n inter-vi re d by th e ow ne
m om en t of his vos. If th e ow ne r du ring his
deat r re ta in ed ow ne r hi
tr an sf er is do na tio h, de at h will tr an sf er it hi s su p until the
n m or tis causa. cc es so rs in inte
rest. Thi
Illustration
Don l~am.co ha s
a ho te l an d a co
pr om is ed to do na mm
te th e hotel to so er ci al bu ild in g as hi s on ly pr op · He
w as able to do na n er n.es. He
te th e ho te l to Ju Juan an d th e bu il di ng to so n, Ju
fin~lizlng th e de a~ w he n th e sa anito. 1
m e w as w or th
accident an d dieded of do na tio n of th e bu ild in P40M - Wh1 e
. g fo r Ju an ito , D
on Juanico rnet an

424

9/30
1/25/22, 10:24 AM

rr 01aP~, l2 - Introduction to Transfer Tax t·


. .
a 10n

1and the bu1ldmg hast ir value of p 45 M


f~e ~ot~1icO, A year after hi death, the pro erti and PSO ~t the date of death of
c,l'I Juil sp ctively. p es have fair values of P48M and
pszt, 1 , re
nsfer of the hotel i a donation inter-vivo
rne rra nd hall be subject to donor's tax Si s. The same shall be valued at
p40 f ~cial building upon his death, the sam~ is;: Don Juani~o still owns the
c0rr1nb1e valued at PSOM and shall be subject to esea:nsfer mort,s causa. The same
ha// e tax.
oie: 3000 inter-vivo are valu d at the date of don ti
1. oonnation mort1s
. I on.
ausa are va ued at the date of death o f th e d ecedent.
2. oo
al rules on transfers Jnter-vivos Mortis causa
er in contem lation of death X ✓
er intended to take effec at death X ✓

Jete transfers ✓ ✓

TRANSFER IN CONTEMPLATION OF DEA TH


A donation that is inspired or motivated by the thought of death of the
decedent is ~on~ti~n. mortis ~us~. If the donation is inspired by motives
associated with life, it 1s a donation mter-vivos.

The motive of donation is the determining factor


The motive of an inter-vivos tr nsfer is very important in determining
whether it is actually an inter-vivos transfer or a mortis causa transfer. The
donor's motive is established out of the wordings of the deed of donation
prepared by the donor to effect the donation.

Thought of death
The pre ence of ex pres wordings in the deed of donation which indubitably
manife t that the donation i inspired by the d cedent's thought of dea h
will qualify a donation as a donation mortis causa.

Illustration
On his death bed, Don Pedro made a writt n donation aying "Death is imminent
upon me. 1 would like to ensure that Pablo will have my sports car as hi legacy.
For this, I am donating my car to him."
Though the donation is made during the lifetime of Don Pedro, th~ donation is
inspired by the thought of death. This is a transfer mortis cau a ub1ect to estate
tax upon Don Pedro ' death.
The evalu ti n of the d c dent's motive is done in pa_rticular whdehn tdhe
d l th rior to his death an a a
ecedent made a donation just severa mon 5 P d
severe illn s, uff ring rom a critical injury, or of too advance age.

425

10/30
1/25/22, 10:24 AM

hapter 12 - Introduction to Transfer Taxation

Tr:in . ll'r. in on temrl, lion of d ath a tually


propPrt to the tran~f •r, • t th dat f dona ionP~s 0wner hi
• l<1t' t~ , n l to donor' t .. , b au it is a don ti ut th e san, .P Ove
n rnortis c ts ta\,, t t~
. ausa. "'<1bt
Mot, e a. sociat d with lifi : ~1

Th f 11 \ ing moti s pr lud a tran r frorn being cl


nt mpl ti n of d "L th: SSifiect
as
1. T r ard s rvi s r nd r d
2. reli e th d nor f th burd n of ma nag
Otie • ,~
on inc me tax ment of the p
ropetty
4. hildr n financially ind pendent
hildr n enjoy the prop rty while the decedent ni .
6. ettl family disput s s 1 hves

mu tration
Rh_ d di ributed a ignificant part of his prop rtie worth PS
childr n:In th de d of donation, he ci d excessive income tax and ~~s~ to ·
ave on income tax as r asons of his donation. tntent to
The donation is a donation inter-vivos ubject to donor's tax.

TRANSFERS INTENDED TO TAKE EFFECT UPON DEATH


A donation that is made on the decedent's last will and testament .
donation mortis cau a. The 'last will and testament' is a docu~s a
.
expressing t h e d ece dent' s desire
· on how h.1s properties will be distributed
eot
after his death.

Similarly, a donation that is made during the lifetime of the decedent with a
stipulation that ownership shall transfer upon his death, the same is a
donation mortis causa

Illustration
During his lifetime, Don Juan transferr d a property to hi favorite
granddaughter, Kar n. Don Juan allow d Kar n to obtain pos es~ion of th e
property but under condition that ownership will not ran fer until hts death.

The transFer or property during the lifetime of the donor ts · tended to take
· not m . 0
'l' 'J • if
effect in ownership immediately but at the pomt o eat . d h Th e transfer ,s
donation mortis causa subject to estate tax.

INCOMPLETE TRANSFERS . . . ro erties fro(ll


In complete transfers involve the transm1ss1on or dehvery of P Ph point of
one per on to another, but ownership is not transferr~d at \ ~ure upoll
deliv ry. The actual transfer of ownership will take effect m ~he u ditions-
th happening of certain future events or sat1. fact10n
· o f certam con

426
-------
11/30
r
1/25/22, 10:24 AM

I11 trod ucti on to Transfer Taxation


12-
te'
01aP plet e transfi r s are not sub ject to tr
fi
. IIY' ifl co~cy. They r sub ject to tran
sfer ta:n~ne~htaxes upon delivery
1~il1~ pr0 P f own ersh ip occ urs.
e future whe n the
f ,~e Ansfer o
o :110"
Jell' tocolllPlete tran fer :
es of ·tiona\ tran sfer
{ cond t ble tran sfer s
l· ~evoc;ers wit h rese rva tion of title
to pro per ty unti l dea th
. 1rans
· . comPlete tra
n fer completed'?
rein f
~o,~ a d'tional tran s ers are com p Ieted mte
.
r-vi vos upo n the hap pen in of
1_ C0 1\ \ow ing dur ing the life tim e of the don or:
the ; lfillrnent of the c ndi tion by the tran g
sfer ee or
a. uaiver of the con diti on by the tran sfer
b. w or
ocable tran sfer s are com plet ed inte r-vi
vos upon:
2. Re" waiver by the tran fero r to exerci e
his righ t of revo cati on or
a. the lap e of his rese rve d righ t to rev oke
b.
-rransfers wit h rese rva tion of title to pro per
ty until dea th are com plet ed
3- by the dea th of the dec ede nt.

ditional tran sfer


and rev oca ble tran sfer s bec ome don
consa when t h e tran s1e
' r 1s
. pre -ter min
. ate d by the dea
atio n mortis
~:ey will be incl ude d in the pro per ties th of the dec ede nt
of the dec ede nt sub ject to esta te tax.
Timing of Taxation of Inc om ple te Tra
nsfers
Revocable and con diti ona l tran fer tha
t are com plet ed dur ing the lifetime
of the tran sfer or con stit ute don atio ns
inte r-vivos sub ject to don or's tax at
the fair value of the pro per ty at the date
of thei r completion or perfection.
Revocable tran sfer and con diti ona l tran
sfer s tha t are pre -ter min ated by
the death of the tran sfer or hall be sub ject
to esta te tax at he poin t of dea th
of the transferor.
Illustration 1
On June 1, 2019, Don Lucio donated a luxu
ry car with a value of P4,000,000 to
his son, Boy, und er a condi ion that Boy
must be a topno cher in the October
2019 CPA Board Exam. To motivate Boy
, Don Lucio delivered the car to him on
June 1, 2019.
The transfer of the car on June 1, 2019 shal
l not be subject to donor's tax eve~ if
there is a physical transfer of the car. Don
Lucio is still the owner of the car. Title
vests only upon fulfillment of the condition.
Assuming Boy topped the CPA Board Exa
m on October 2019, the completed
~onation shall be subje t to donor's tax at that
point if Boy failed ~o top t~~ exam,
here will be no donation to tax; however
, if Don Lucio waived his cond1t1on, the

427

12/30
1/25/22, 10:24 AM

. to Transfer Taxation ,
12 - Introduction
Chapter d at that time and will be taxab/
. . vertheles be perfecte e to clan
donation ~~II "~u at that time. 01'1
tax at its Jair va h exams, car would be transferred rn
. Don Lucio d,e. d be'ore
'I '
t e b']·ect to estate tax at I·tsfimr
.
value ortts·c0~
As ummq d would be su at the r~
a part of hi estate an ~04
of death.
lllustradon 2 M rk transferred a phone to Goldemaire but subj
On Febru ry 14, 2019, a eci ~
revocation if Mark o please .
. h sical transfer of prop~rty on February 14,201
Although there 1s an ~ctuaf ~!or's tax since there 1s no transfer of owners 9: tire
ame cannot be subject to hip o
that date.
. hi ri ht to revoke, the donation shall be subject to do
A suming Mark wa,vedth . '9 o'waiver. If Mark revoked the property, the~ ~0rs
tax at its fair value at e time 'I elS ~o
donation to speak of.
. M k d' d ,·chout revoking the phone, the same would be transr.
Assuming ar te w . b• ~erreo
.
mortJs-causa an d wou Id be included part of his estate su '}ect to e tate tax at 1.
fair value at the point of death.

Complex Incomplete Transfers


Incom plete transfers are sometimes made for less th~n full and adequate
consideration. imilar to complex transfers, the gratuity component of Ute
complex transfers is subject to the appropriate type of tran fer tax.

Test of Taxability of Complex Incomplete Transfers


The following must bee tablish d before a compl x incompl te is taxable:
1. The incomplete tran fer mu t have be n paid for Jes than full and
adequate consideration at th date of delivery of the prop rty.
2. At the completion of the transfer, the property must not have decrease
in value b low the consideration paid.
Valuation of com lex incom lete transfers
Mortis causa - - - -1-nter-vivo
Fair value at death I ss Fair value at co mpletion or
consideration upon transfer perfection f dona tion less
consider tion u on transfer
Example:
Type of
Donation At tr nsfor Fair value The donation is
Selling price
Fair value
lnter-vivos
P 4,000
at death com utedo~
~Mortis causa
4,000
P 10,000 P 15,000 P10 000 -P4,0_2!.
P 10,000 P 15,0 00 P1 5: ~

428

13/30
1/25/22, 10:24 AM

r. aP
,er 1
2 _ ,ntroduction to Transfer Taxation

ifstit io17, Mr. D tr ns rred hi


111 r worth P1,000,000 to E but for a
11 11111 'id r tion of P200,000 only. The transfer shall be revocable b O I
Ol'I 111.11 • n y n
n1•"1 •
,cJf"S·
S) i r b fore death
ct: 201 , o inti mat d to E hat he wa waiving his righ of revocation.
W3
111 jlll~ ' lue f th ar w P 00,000 nth t date.
l fair a
'f~l'
0 wa til~ living upon the perfection of the transfer, the transfer is a
. ;11 e . n int r-v1vo . It shall be valued at: PB00,000 less P200,000. Hence,
if 11°"0 00 hall be ubject to donor's tax.
()0,0
z: oeath without revoc~tion
ca in tead that Mr. D died on July 3, 2019 without waiving his right to
suk~,ethe transfer. The fair value of he property was P 900,000 at that time.
revo e
e revocable transfer is pre-terminated by death, it is a donation mortis
,nee th
It shall be valued at (P900,000 less P200,000). Hence, P 700,000 shall be
causa.
sUbject to estate tax.

mu tration 2
1,ordknight has a rare Egyptian artifact which has a fair value of P2,000,000. He
gave the artificact to Noventa for a consideration of Pl,950,000 but revocable if
oventa did not graduate as cum laude. Noventa subsequently graduated cum
laude when the artifact was worth P3,000,000.

The transfer was paid for an adequate consideration considering that the selling
price approximates the fair value at the date of delivery. The transfer is a bona
fide sale which will not be subject to transfer tax even if the fair value of the
property appreciated at the date of completion of the donation.

Illustration 3
Soren sold a gold bullion with a fair value of P2,500,000 to Leomilo at a price of
Pl,800,OOO but revocable within one year. The one-year period lapsed when the
gold bullion had a fair value of Pl,700,000.
The transfer is insufficient thus subject to donor's tax upon expiration of the one-
year period since oren is living. Since the fair value upon completion fell below
Pl,800,000, there is no gratuity subject to donor's tax.

Assuming Soren died before the one-year period when the gold bullion was worth
PZ,300,000. The donation mortis causa subject to estate tax shall be PS00,000
~:2~ 00,000 - Pl,800,000). Assuming the value fell ~e/ow the P1,800,000
nsiderations upon Soren's death, there is no gratuity sub1ect to estate tax.

429

14/30
1/25/22, 10:24 AM

. Transfer Taxation
,
12 - Introductio n to
chapter
RAN FERS
NON-TAXABLE T . crti s which are not actually do
fcrs of prop nauo~s
Ther ar trans . fi taxes uch a :
ub1· ct to trans r '
h n , no t
1. V id tr n fi rs
2. Qu ·i-tr n fi r

Void Tran fer that re prohibited by law or those th


Void tran fers r th0 _e nts for their validity. Void transfe at do
' '
t I gal . requiremeperty and are there,ore rs do
con1orm o not subject to ~~
tr nsfi r owner hr P over pro tti!JJ
ta .
E ample of void tran fers: h d
1. Donation of properties not owned by t e onor
2. Donation between spouses . . .
. h • h do not manifest all essential requisites to validity sUchas
3. Donat1ons w 1c
donations refu ed by the donee
4 _ Donations tha do not conform to formal requiremen ts such as or-a
donatjon of real properties

Illustration 1
Tired of feeding Zeus' derby roosters, Raymund donated them to Andri,\ h'
bestfriend "tupada " master.
Since Raymund does not have owner hip over the thing donated, the donation is
void. There is no valid donation to speak of; hence, no donor's tax is imposable.

Illustration 2
Jn an overnight drinking spree, Zeus orally donated his seven-herta t
agricultural land to Raymund.
Oral donation of real property is legally void because the law requires tr
execution of a public instrument. There is no imposable donor's tax since there i.!
no donation to be taxed.

Quasi-transfers
Th re transmissions of property which will never involve transfer of
ownership. For the purpose of our discussion, let u refer to thes-t
transmissions as "quasi-transfers." Quasi-transfers are not taxable.
Examples:
1. ~ransmission of the property by the usufructuary to the owner of the naked
title
2. Transmission of the prope ty b
3. Transm ission of th r ya trustee to the real owner . ln
accordance with th ~ p~operty from the first heir to a econd herr
e es,re of a predecessor
430

15/30
1/25/22, 10:24 AM

r,:-_
01•Pte
,ntroduction to Transf
er Taxation

tfllriOJl 1
11111s died Jeaving a track of land to C b .
~r.A
·II to give B a u Cufructuary
Aft right to useutand
smce Cwas
en1·0 h a minor' Ad ev1sed. in h'15
1 • git ver to . Y t eland th
er the lapse of 10 years, B transferred 10
fore Iandyears
~11r111n
1 0 to Cbefore
l d
,war)' B oes not own the /and. H ·
Us'~11t not ownec hip thereto. B's tu·m:v;as u;onted on(y the right to use the
1'~', of th• naked title), shall not be sub .,':{ e la nd to C, the real owner (i.e.
~::1,rof ownership. ~ to donor's tox since there Is no

· n2
111usu-auo .
r A died leaving . building as inheritance to C 5. C
a commerCial
.' or, A appointed
rn1n f h his older. brother
Cb B to be the fiduciary heir
.. tomhce was a
t e property
ta!<• care o t e same unll 1 ecomes 18 years old. When c turned 18 ears
:id. 8rransferred the property to C. Y
1h• tr0nsmission of th~ property from 8, o mere tru_stee, to C, the real heir, shall
,at be sub;ect to donors tax smce there 1s no transm1mon of legal ownership over

the property.
If the usufruct in the Illustration 1 and the fiduciary relationship in the
,n-ation 2 are pre-terminated by the death of the usufructuary or fiduciary
1110• the transfer of the property to the r al owner is likewise not subject to
. no trans ter of ownersh.,p.
he1r, taX for the same reason that there ,s
eslllte

16/30
1/25/22, 10:24 AM

Chapter 12 - Introduction to Transfer Taxation

CHAPTER 12: SELF-TEST EXERCISES

Discussion Questions
1. What is a transfer? Discuss the types of transfers.
2. What are the types of unilateral transfers and the type of
each? ta)( apPli
3. What is a "complex transfer"? Explain. ~ ~
4. Enumerate and explain the theories rationalizing transfer tax .
5. Enumerate and discuss the nature of tran fer taxe . ation.
6. What are the two taxpayer classifications in transfer tax? 0 .
of transfer taxation for each taxpayer cla s. · tscu ss the Sc
7. Why is situs of property relevant in tran ~ r taxation? E pl i
8. Discuss the reciprocity rule in transfer ta tion. n.
9. Enumerat examples of motive a oci t d with life.
10. Why is it essential to cla sify donation . E plain bn fly.
11. Enum rate the non-tax bl tran f r . it mpl .
12. Differentiate void tran fer . qu i-tr n r nd inc mpl
13. Di cuss how conditional tran f rand r vo bl tr n i r r
14. Di cuss the taxation of incompl te c mpl

True or False 1
1.
2. Uni It tt l
3. Bila tra t to 111mm • t 1
4. alt•~ mJ h. rt •r,
5. Bila 1 n-.
6. Don 1b1
7. Ad u
8. The a d on,u1 on 1
9. The
10. The
11. Th d
12. o .1 h th
13. Wh n >n,1d •r ,ll1 o n· 1 l qt
14. Bo n

nu t.1. 1t1 n.

17/30
1/25/22, 10:24 AM

troduction to Transfer Taxation


1-Z .... 111
9Pie' ta)( IS an in d ,rec
' t tax.
Cf1 11 sfer i a revenue tax.
fr3115 (er t.u< payers are classified into two groups: re ldents and non-
1 fr" fer tax
i;. frafl ts
q 1defl ·d· nt aliens are subject to tax only on transfers of Philippine
re rest e
~011· tie .
10· roPer re ubject to tax on transfers ofpropertl s regardless oflocation.
~101,el'l :e
subject to tax only on transfers of properties situated In the
.,re11. 5 a·nes.
ti· ,..,
tZ· ptii11PP 1 are subject to tax on transfers of properties regardless of
~es idents
13- 10cati 0 n: dents are taxed only on transfers of property located in the
on-rest
14, 1• pines.
phi 1P ·procity rule on intangible personal property applies only to non -
fhe rec•
15- resident aliens.

false 3
frtle or ciprocity rule may apply to movable personal property located in the
1'he re
1, h·Hppines.
~h~ reciprocity rule applie to intangibl properties of any alien located in
2, the Philippines.
F nchi es are subject to transfer tax in the place where th y ar xercised.
3· T~e shares and bonds of domestic corpora~ons are presumed ituated in
4· the Philippines for purpo e of transfer taxation.
Shares and bond of for ign corporations are always presumed situated
s. abroad for trans1er" tax purposes.
For purpos s of transf r ta . the interest in a business partnership
6.
organized abroad i presumed situat d abroad.
7. Ca his considered n int n ible property.
s. Share of sto k and bond r t ngibl p r nal pr p rti
9. Donations inter-vivo re subject to trans er tax at the point of death of the
donor.
10. Donations mortis cau a r ubject to transfer tax at the death o the
deced nt.
11. Donations inter-vivo r inspired by th g n ro ity of th don r.
12. Adonation morti c us i ted by th d ath f th d c d nt
13. As a rule, all prop rti s of th donor exi ting t th point of death constitute
his donation m rti u .
14. All orm of gr tuit u tran mi ion of property while th donor is living is
con id r d d n tion int r -vivo .
lS. Tran er in nt mpl tion of d ath ar donation made inter-vivos but are
actuc II don ti nm rti u .
16· Th tr n f r pr perty which con eys title to the property only upon
1 d th of th donor ar don tion m rti au a.
· E late t rat r high r th n donor's tax rates.

433

18/30
1/25/22, 10:24 AM

. t Transfer Taxation
Chapter 12 - Introduction o
. 'ers ar not ubj t to tax up n physi al tran
18. lncom rl te nan I' sfer Of~
property. whi h are pre- termin ted by the deati.
19 In ornplete transfers t t tax ,, or
· b' t to s a ·
tr. nsferor ar su JCC fer ar com p I ted during the lifetime
20. When incomplete tr ~s b'ectto donor's tax. or
tr nsferor, th tran fer J su J

Multiple hoice - Theory: Part 1


Which i subj ct to tr nsfer tax?
1. .
a Gratu1tou tr an 11'er c• Complex transfers
b: On r us transfer d. A a nd C

2. Which is ubject to transfer tax? .


a. ale c. Donation
b. Barter d. Loan

3. A property i transferred for I than full consideration when it is sold


a. above the fair valu of the property.
b. below the fair value of the property.
c. at the fair value of the property.
d. at any price which is deemed unacc ptable to the lier.

4. Gratuity is not charact rized by


a. Absence of consideration
b. Pre ence of adequate consid ration
c. Presence of inadequate consideration
d. A and C

5. Transfers for full or adequate con ideration is subject o


a. Income tax c. Both A and B
b. Tran fer tax d. Nei her A nor B

6. Tran fer for inadequate consideration is subj ect to


a. Income tax c. Both A nd B
b. Transfer tax d. Neither A nor B

7. The transfer for adequate consideration is


a. Exempt from transfer tax
b. Exempt from income tax
c. ubject to tran fer tax
d. Band C

8. The gratuitous com f


a. Sub·Ject to income
. ponent O a transfer for inadequat consid ration is
tax
b. Exempt from income tax

434

19/30
1/25/22, 10:24 AM

on to Transfer Taxation
I tro du cti
1.2 - n
rer
1P t {rom tra nsf er tax
e~erTlP
,. J\ and B
. d , ?
d, .
. che tax pa ye r m on or s tax
,, 110 1
c. Do nee
' Do or d. Be nef ici ary
~ 3 tee
. rru s
t,.
ati on ?
. the tax pay er in est ate tax
c. Ad mi nis tra tor
J0.
wn° 1
oec
de nt d
. xe cu tor
a. E rate
t,.
of tra nsf er tax ati on ?
h·ch is no t a rat ion aJi zat ion
'rax eva sio n c. Sta te pa rtn ers hip
11- \
d. AU of the se
a, TaX rec ou pm en t
b- in inc om e t.ax
lly rec ov er fut ure red uct ion
fer tax is im po sed to par tia to few or
fro m the spl it of inc om e pro du cin g pro pe rty
1Z- :i~ h wil l ari se
several taxpayers.
the sta tem en t des cri be?
What the ory tat em en t do es
Tax evasion the ory c. Benefit rec eiv ed the ory
d . Wealth red ist rib uti on the ory
a.
b. Tax rec ou pm en t the ory
ion and
sfe r of pro per ty by do nat
_ The go ver nm ent e~f ~rc es the ~a _n mu st hav e to be
13
ession . By exe rc1 mg hes e pn v,l ege , the tra nsf ero r
ucc
sta tem en t des cri be?
taXed. What the ory do es th
c. Benefit received the ory
a. Tax eva sio n the ory
d. Ability to pay the ory
b. Tax rec ou pm nt the ory
ent ire
of w alt h sho uld be tax ed so tha t it will benefit the
14. The tra n fer
tat em ent d cribe?
society. Which the ory do es the
a. State pa rtn er hip the ory
c. Wealth redistribution the ory
d. Tax rec ou pm ent the ory
b. Ability to pay the ory
ent 's
tio n cou ld no t be pos sibl wi tho ut the go ver nm
15. Wealth acc um ula be ubject to tax
tra nsf er of wealth should
indirect par tic ipa tio n. The har e on the w alth.
aus e it i ju t f ir for the go ver nm ent to take its jus t
bec

The sta t me nt d cri b s alth redistribution theory


a. tat par tn rsh ip the ory c. We
d. Tax minimization the ory
b. Ability to pay the ory
ome tax.
can b tru tu red in uch a way to av id paym nt of inc
16- Transfers
t of transfers mu st be taxed.
Thu , the gra tui tou s om po nen

435

20/30
1/25/22, 10:24 AM

Chapter 12 - Int rod uct ion to


Transfer Taxation

Th sta t me nt des cri bes


a. T x evasion h ory c. Ben ft rec ived th ory
b. Tax rec oup me nt th ory d.
Ability to pay the ory
17. Which i not a cha rac eri
tic of tra nsf er tax?
a. Privilege tax c. Local tax
b. Nati nal tax d. Direct tax
18. Tra n fer taxe are not
a. Fiscal taxe
c. Ad val ore m taxes
b. Pro gre ive taxes
d. Tra nsa ctio n taxes
19. Tra n fer taxes are
a. R gr ssive taxes
c. Specific taxes
b. Ind ire ct taxes
d. Revenue tax es
20. Who is sub jec t to ran sfe r
tax?
a. Tra nsf ero r
c. Both tra n fer or and transferee
b. Tra nsf re d. one of the se
21. Sta tem ent 1: All Philippine
tra nsf ers are sub jec t to transfer
Statement 2: All foreign tra nsf rs truces.
are sub jec t to ran fer taxes.
Which tat em ent is gen era lly cor
rec t?
a. Sta tem ent 1 c. Both sta tem ent s
b. Sta tem ent 2 d. eith er sta tem ent
22. Sta tem ent 1: The am oun t
of tra n fer tax i d p nde nt upo
thi ng tra n fer r d. n the value ofth
Statement 2: Donor's tax i imp '
o d upo n the pro per ti s of
wh ile est ate tax i imp o ed on a decedent
a donor.
Wh ich is inc orr ect ?
a. tat em ent 1 c. Bo h tat em ent s
b. Sta tem ent 2 d. Neither sta tem ent
Multiple Choice - Theory: Pa
rt 2
1. Tra nsf r tax on don tio n mo rtis
causa appli to a
a. Natural per son c. Both A and B
b. Juridical per on d. eith r A nor B
2. Tra nsf er tax on don atio n inte
r-vivos app lie to
a. Na tur al per on c. Both A and B
b. Juridical per son d. N ith er A nor 8
3. Which is taxable on all tra nsf ers
reg ard les s of lo atio n?
a. Reside nt citiz n c. on- res ide nt citizen
b. R sid nt alien d. All of th se

436

21/30
r _
1/25/22, 10:24 AM

introduction to Transfer Taxation


er 12
cl'aPt 1 not subject to estate tax?
5
~" bjch
·•1 Residen~ a1·ie n 1· ed. Domest1· ~ corporation
~- 3, Non-resident a ten • Non-resident citizen

vJ)lich is not subject to transfer tax on transfer of properties located

,. t,road? .. c. Non-resident alien


3 Resident c1?zen
d. Non-resident citizen
a. Resident ahen
b-
0 resident alien is
6, A0 \ ~ernpt from transfer taxes.
a, subject to transfer tax on Philippine properties.
b, subject to transfer tax on global properties.
c. subjec to transfer tax only on transfers of tangible properties.
d.
may be exempt on transfers on the basis of
Which of the following
7· reciprocity?
c. Non-resident alien
Resident alien
~- Non-resident citizen d. B and C

Which propertie does the reciprocity ex mption cover?


8· Intangible per onal prop rty locat d abroad
a.b. intangi ble per onal prop rty I cat d in the Phili ppines
c. Tangible and int ngible prop rti lo ted abroad
d. Tangibl nd intangible prop rti lo ated in he Philippines

9. Which of the followin is an in ngible per onal property?


a. Franchis c. Bonds
b. Stock d. All of th e

10. Which of these is subj ct to tax only on tran f rs of prop rties situated in
the Philippin ?
a. A citizen o ) pan r sidin in th Philippine
b. A citizen of th Philippin r siding in H ng Kong
c. A citizen of Am rica r iding in K r
d. A itizen of th Phihppin r iding in the Philippines

11. Which of th s i t bl n lobal transf rs of properties?


A. An A i in the Philippines
B. g M
C. in in
D. A , ,ppines

c. C onl
a. A nd
d. , B nd D
b. B nd D

437

22/30
1/25/22, 10:24 AM

Chapter 12 - Introduction to Transfer Taxation

12. Which is subject to donor's tax?


a. Donation mortis causa c. Sale for an adequat
d . Both B and C econ 'd
b. Donation inter-vivos si er.it·
loll

13. Which is not a characteri tic of a donation inter- ·


. . d b th v1vos?
a. inspire y e genero ity of the tran feror ·
b. inspired by. th thought of death of the trans£,eror
c. zero consideration
d. insuffici ncy of consideration

14. Which of these is a donation mortis cau a?


a. Donation in cont mplation of death
b. Donation to take effect upon death
c. Revocable donation pre-terminated by d ath
d. All of these

15. Wh!ch of th following donations made during the lifetime ofth


ub1ect to donor's tax? e do
a. Donation inspir d by motive associated with life
b. Donation made during the lifetime of the decedent in pir d
thought of the d cedent's d ath e by
c. Donation made in contemplation of death
d. All of these

16. Which of the following is not am tive ssociated with life?


a. To ettle family di put
b. To save on tr nsfer tax
c. To se childr n fin n i lly independent
d. To r lieve th donor of th burden of manag m nt of the property

17. Donation made on the decedent's last will . nd t s: m nt i


a. a don tion m rti cau a. c. subje t to incom . a .
b. a donation inter vivo . d. ubject to don r t •
will not be ubj<'(I
following form of non-ta bl
18. Which ofth
tran f, rt ? c. Incompl t tr n r
a. Void transfer d. A and B
b. Quasi-transfi r r rn y be subJeCI u
19. Which of the following form of non -t
transfi r tax in the future?
Void transfer . lncompl t tr n f, r
:: Quasi-tr nsfer d. All of th

23/30
1/25/22, 10:24 AM

2 - Introducti on to Transfer Taxation


ter 1
cPaP ·sin orrcct reg rding th• xation of Incomplet e transfers which are
O- \Vl 11 't~ \y complel •d?
~ e\fC 1~heY ar subj ct to c tate tax.
JI• 1 arc subjc t to donor's tax.
b- ~/c ar subj• t to both donor's tax and estate tax.
c. T~ y re ubject to either donor' tax or estate tax.
d-
a om plet ran fer?
• , 1•
\ hi ,1 .
1- conditiona l transfer .
3· Tr n fer in on tempi t1on of death

b- lrrevo able transfers


c. Transfer with re ervation of certain rights until death
d.
onditional d nation may be complete d by
22· c Fulfillment of the condition pecified by the transferee
~- Waiver of the condition by the transfero r
to revoke
c.· Lapse of the right
d. A and B
transfer which are still pending
23. conditional tran fers and revocable
completion at the point of the transfero r' death are
a. donation inter-vivo . c. subject to estate tax.
b. subject to donor's tax. d. subject to income tax.

Multiple-Choice - Problems: Part 1


1. Mr. Bonifacio purchase d a piece of land in 2011 for PS00,000 when it was
worth P450,000. He transferre d the property when it was worth
Pl,000,000. Sub equently, r. Bonifacio died when the property was worth
Pl,200,00 0.
Assuming that the donation is a donation inter-vivo s, what is the value to be
subjected to donor's tax?
a. P 450,000 C. p 1,000,000
b. P 500,000 d. P 1,200,000

2. Assuming that the donation is a donation mortis causa, what i the value to
be subjected to estate tax?
a. P 450,000 C. p 1,000,QQQ
b. P 500,000 d. P 1,200,000

3- Mang Juan transferr ed a property with a fair value of Pl,000,00 0. Title to


the property was stipulate d to be tran ferred immediat ely. Subseque ntly,
Mang Juan died. The property was worth P800,000 at the date ofhi death.

Which is correct?
a. The property is ubject to donor's tax at Pl,000,00 0.
b. The property i subject to donor's tax at P800,000 .
439

24/30
1/25/22, 10:24 AM

Chapter 12 - fn
operty is
.
troduct,on to

ubject to
Transfer Taxation
tate tax at P800,000.
e!tate tax a Pl,000,000.
,
c. The pr . ubject to e
Tl property is
d. 1e . 1 to the property was r served b
. d that legal tit e Y ~ill\
4 Assume in tea . rrect? gI~
• .1death which 1s co donor's tax at Pl,000,000.
unllTI pr~perty is subject to donor's tax at P800,000.
a. te ·
b The prop rty is su .
bject to 00 000 .
e tate tax at P8 '
. The property i sub!ect tto estate tax at p ,000,000.
dc. The property is • ub1ect o

. .. . d leaving the following property at the Poi


5. A re ident c1ttzen die nt or .
d ath : Fair Acqui ition
Value ~
House and lot p 4,000,000 P 2,000,000
Car 800,000 1,200,000
A riculturaJ land 2, 00,000 soo,ooo
t f the donation mortis causa.
Compute the amoun o c. p 6,900,000
a. PO p 7 300 000
b. p 3,700,000 d. ' '

6. A donor trans,erre
, d the following prop rties on a certain day:
1 ines r d
Cash p 400,000 P 0
Land 1,000,000
Car
3,000,000
As urning the donor is a r sident citizen, wh tis th
inter-vivos? moun of the donation
a. P 1,000,000 C. p 4,000,000
b. P 1,400,000 d. P 4,400,000

7. A suming the donor is a resident alien, what is th


in er-vivos? amount of th e donation
a. P 1,000,000
C. p 4,000,QQQ
b. P 1,400,000
d. P 4,400,000
8. Assumin the donor is a non-resident citiz
donation inter-vivos? n, wh t i th amount of the
a. P 1,000,000
b. P 1,400,000 C. p 4,000,0QQ
d. P 4,400,0 0
9. Assun:iin~ the donor is a non-re id nt Ji
donation inter-Vivos? n, wh t i the amount 0f tht
a. P 1,000,000
b. P 1,400,000 C. p 4,000,000
d. P 4, 00,000
440

25/30
1/25/22, 10:24 AM

,.,,er 12 - Introduction to Tran sf r Taxation


cf1i1"'
inA 1tw donor is ,1 non -r sld,•nt -111 •n w ·th
ufll1, ttw
10· .. 1l ,lllHllllll u I l Il ' tI 011,ltinn In , ·r-vivu
' ·?1 a r clproclty exempro
I n,
' h• I 1,000,000 . P '\,000,000
•1· p 1,'100,000 d. I' 1.400,000
i,.

hoic - Probl ms: Part 2


111ILfJllc·

Mr· Chu ho d d le. ving lh followin properties:


l,

Philippines Abroad
tm •nt in to ks
Ill V p soo,ooo p
uou andlot 4,000,000
R adential lot 3,000,000

urning Mr. Chu ho is a n n-r siden citizen or a resident alien, what is


the mount of donation morti causa?
a. PO C. p 3,500,000
b. p 3,000,000 d. P 7,500,000

A suming Mr. Chucho i a non-re ident alien, what is the amount of the
z. donation m rti causa?
a. P0 . P 3,500,000
b. p 3,000,000 d. P 7,500,000

3_ Assuming Mr. Chucho is non-r sident alien and the reciprocity rule
applies, what is the amount of the donation mortis causa?
a. PO c. P 3,500,000
b. P 3,000,000 d. P 7,500,000

4. A seller of goods made the following sales:

Fair market value P 5,500,000


Selling price 4,500,000
Cost 2,500,000
What is he amount subject to income tax?
a. p O C. p ,500,000
b. P 1,000,000 d. P 2,000,000

5. In the immediately preceding problem, what is the amount subject to


transfer tax?
a. P0 C. p 1,500,000
b. P 1,000,000 d. P 2,000,000

441

26/30
1/25/22, 10:24 AM

Chapter 12 - Int rOd uction to Transfer Taxation

Avalon sold
r. Data r Jating Lo the sale and the car are as fou
F,1ir market v, lue
clling pri · P 4,000.000 o\\rs,·
Ost s,000,000
2,soo.000
Wh 1 ; th amount ubj ct to income ax nd to tran fer tax r
. p O; po . Pl,S0O,00O; PJ,000.000 ' •spec,;,~)!
b. Pl, 00,000; PO d. P O; P2,soo,ooo

7. Mr. p ter made a r v c hie transfer of his stock inve tin ent on July
in favor of his broth r, Merto. Peter died on D cemb r 15, 2020. 4, 101
0
Th tocks had the foll wing fair valu s:

July 4, 2020
Augu t 20, 2020 P 1,200,000
December 15, 2020 1,100,000
1,600,000

As urning Peter waiv d the right to revoke on August 20, 2020, What is
amount ubject todonor's
a. P 1,200,000; tax and the typ of transl r tax to apply?
transfertax ~,
b. P 1,100,000; don r's tax
c. P 1,600,000; donor's tax
d. P 1,600,000; esta e tax

8. Assuming Mr. P ter did not r voke th property until the date of his dea~
apply?
what is the amount subject to tr nsfer tax and the type of transf r tax~
a. P 1,200,000; donor's tax
b. P 1,100,000; esta e tax c. Pl,600,000; donor's tax
d. Pl,600,000; e tate tax
9. In October 2020, Mr . Blend r transferred a prop rty oh r younger sister,
Careen, u bjec to the condition that Care n mu t graduat in March 2021.

The property was worth P 800,000 in Oc ober 2020 and P 900,000 in Marci
2021. Mrs. Blender di d on July 15, 2021 when he property was wortl
Pl,200,000.

Assuming that Careen failed to graduate in M rch 2021, what i the amount
of dona ion and the tran fer tax to apply?
a. P 800,ooo; donor's tax
b. P 900,000; donor's tax
c. P 1,200,000; estate tax
d. None; none

27/30
1/25/22, 10:24 AM

io n to Tra f
12 - In tro du ct ns er Taxation . ,,
e11aPcer
. 110 his bi rth da y, M r. Fu hen do na t d the follow ·
DtJ (I t,
10· ing Properties:
h do na tio n to hi s w i
ca . . f I an d lo t p soo.ooo
oral donal1011 0 lO U J,000,000
tion .
compute th e tax bl am ou nt of dona inter-vivos ·
P3 0
pO · , 00.000
,l , 500 00 0 d. P3,soo.ooo
b, p I

tu lri plc •C ho ic . - Pr
ob I rn : Pa rt 3
veyed th e follow · properties on the folio .
A re sid en t all n on ing wi ng terms:
J. . F . lue
.Consideration, _air va
car
P
O P 500 000
I 30 0,0o o 300'.ooo
Moto rcy e 80,000
20,000
1.,aptop
fe ta
ou nt su bj ec t to trans
compute th e to ta l am r x.
c. P 500,000
a. p 88 0,00 0 d. P 60,000
b. P 56 0, 00 0
by a seller:
s to a tra ns fe r m ad e
z. The following re la te
P 1,500,000
Selling price 4,000,000
of pr op er ty at da te of ale
Fa ir va lu e
se lle r 5,000,000
e at de at h of th e
Fair va lu
ale, what is
e pr op er ty is tra ns fe rre d on th e date of
to th
Assuming th at tit le d th ty pe of tra n fer tax to
apply?
do na tio n an
the am ou nt of c. P 2,500,000; do no r's
tax
a. PO ; no ne r's ta x
00 0; es ta te ta x d. P 3, 50 0,00 0; do no
b. P 1, 00 0,
ect on the
e pr op er ty w as ag re ed upon to take eff
3. Assuming th at tit
le to th of transfer tax to
t is th e am ou nt of do na tio n an d the type
date of de at h, w ha
apply? tax
c. P2,S00,000; do no r's
a. PO; nOne d. P3,S00,000; es ta te tax
te ta x
b. Pl ,0 00 ,0 00 ; es ta
ade by a seller:
re la te to th e di sp os iti on of pr op er ty m
4. The fo llo w in g
p 4,200,000
4,000,000
Selling price le
on da te of sa 5,000,000
Fair value of pr op rt y
Fair va lu e at d at h
of th ell er
ub j ct to donor's tax?
What i th e am ou nt c. p 80 0,00 0
a. p d. p 4, 00 0, 00 0
b. P 20 0, 00 0
443

28/30
1/25/22, 10:24 AM

d t' on to Transfer Taxat ion


Chapter 12 - Intro uc '
1
fi r is r vocable until the death of the d
5. As urning th~t the tran~nt subject to state ax? ecedent
eller, what is the amo p 800 000 ,
c. '
a. P O d p4 000,000
b. P 200 ,000 . '
, d properties to save on income taxes. The do
6. A donor trans1err nati 011 Is
ubj ct to state tax.
a income tax. c. e . d
· d , d. both income tax an donor's .....
b. onor s tax. '<Vl.

7. Mr. B made a revocable donation of a pro~erty valued at Pl,0OO,Ooo


is a correct statem nt regarding the taxation of th e donation? ·
a. The donation shall be subject to donor's tax.
b. The donation shall be subject to estate tax.
c. The donation is subject to income tax.
d. The donation shall neither be subject to donor's tax nor estate tat

8. Kumar made an irrevocable donation of real proper ty during his Jifeti11J .


favor of his wife. Which is true? e
a. The donation hall be subject to donor's tax.
b. The donation hall be s ubject to e tate tax.
c. The donation is s ubject to either donor's tax or e tate tax.
d. The donation is void and is not subject to tra n fe r tax.

9. Which of th se is not co nsidered within th Phili ppi ne ?


a. Fran his of a foreign franchisor which i exerci d in the Philippines
b. Stocks of a domestic corporation with 60% of bu iness in the
Phil ippine
c. Foreign shares which are traded in the Philippine stock market
d. Stocks of a fo reign corporation with 75% of bu ine in the Philippine1

10. Mr. Ku Loth, transferred a stock portfolio with a fair valu of P2,000,000for
Pl,990,000. Owner hip hall not tran fer until full payment is madebydit
transferee within 10 days. On the oth day, the buy r p id th price in full
when the portfolio was worth P 2,100,000. ub quently, the tran feror
died when the stock were worth P3,400,00.

What is the amount subject to transfer tax and the transfer tax that would
apply?
a. PO; None
b. P 50,000; Donor's tax
c. P 100,000; Donor' tax
d. P 1,400,000; Estate tax

444

29/30
1/25/22, 10:24 AM

- Introduction to Transfer Taxation


tef 12
C~aP donated t ck e unties . . to h is daught r, Mary, as reward to the
J. J\flton for services rend er d to Anton. The prop rty was worth Pl,100,000
1 1arter Anton wrote th ~eed of donation. The same had a value of
\\'henOoOO when he donation wa accept d by Mary.
p1 ,0S ,
. the amount of gratuity and the applicable transfer tax?
~vha;toso,OOO; donor' tax
a- p1,oso,ooo; state tax
b, pi,100,000; donor's tax
~-. Pl,100,000; estate tax

M Erick received in tru t a prop rty worth Pl,400,000. Erick was


12- r;u ted to b the fiduciary heir to th property left by his mother,
e~arciana. Whil still living, Erick transferred the ame property to his
~ ounger brother, Erwin, in accordance with the will of their mother,
~arciana. The property wa worth Pl,500,000 a the date of tran fer.

The property i
a. ubjcct to donor's tax at Pl,500,000.
b. ubject to estate tax at Pl,500,000.
c. subject to donor' tax a Pl,400,000.
d. exempt from transfer tax.

13. Raymund was indebted to Z us with a P 50,000 interest-b aring loan.


Raymund rendered ervices worlh PS 1,500 to Z us. Zeus cancell d
Raymund's indcbtednes · when it was wor h PSl,500 in luding intere t

The cancellation of ind btedn sis


a. subj ct to donor's tax.
b. subject to state tax.
c. exempt from transfer tax.
d. ubject to bo h donor's tax and esta e tax.

445

------
30/30
Chapter 13 - The Concept of Succession and Estate Tax

CHAPTER 13
-T_H_E_C_O_N_C
_E_P~T_ O==-
F=S=-
U-=
Cm=
C-=
ES=S=I=O=N~&
~ E~ST
!.:~~ T!_I
£1~
Chapter Overview and Objectives ~
This chapter discusses only the basic rules of succe ·
• d . ss1on anct
mtro uct1on to estate tax. Pro,•..IQ~S
q~
After this chapter, readers are expected to understand:-------------------...
1. The concept of succession ··
2. The types and elements of succession A
3. The nature of estate tax b
4. The types of decedents and their taxation rules
5. The model of estate taxation 1
l

SUCCESSION
"Succes~ion~' is a mode of acquisition by virtue of which the prope .
and obhgat1ons
. to the extent
.
of the value of the inheritance, of a prty,
~~
tights
transmitted through his death to another or others either by his wm are
operation of law (Art 774, Civil Code). orby

The inheritance includes all the property, rights and obligations of a pers
which are not extinguished by his death (Art 776, Ibid.). on

The rights to the succession are transmitted from the moment of the death
of the decedent (Art 777, Ibid.). The decedent is a deceased or dead person.

TYPES OF SUCCESSION
1. Testate or Testamentary Succession
Testamentary succession is that which results from the designation ofan
heir, made in a will executed in the form prescribed by law. (Art 779,
Civil Code)
5
A person can specify the recipient of his properties upon death. Thi.11
. d t called last w1
designation must be made t h rough a wntten acumen ,A
5
and testament. A person who died with a will is said to be "te tote.
person who died with a written will is calle d a "testator".

2. Legal or Intestate Succession . \'1d one, the


When a decedent dies without a will or with an mva default
th
distribution of the estate shall be in accordance with e
provision of the Civil Code on succession.
446


-The Concept of Succession and Estate Tax
13
pter
cna .
dsuccessIOn
,tile mission of the decedent properties shall be partly by virtue of a
J, 1r~ttn~n will and partly by operation of Jaw.
wfl

~;JI. is an act whereby a person is permitted, with the formalities


ft w'//·bed by Jaw, to control to a certain degree the disposition of this
prescri O take effect after his death (Art 783, /bid).
estate, t
. is an expression of the decedent's desire as to how his properties will
Awill ·buted after his or her death.
be
distri
king of a will is a strictly personal act; it cannot be left in whole or in
Tbe :~ the discretion of a third person, or accomplished through the
part entality of an agent or attorney. (Art 784, /bid.)
mstrum
es of will
'fYPH /ographic will - a will which is entirely written, dated, and signed by
1· th~ hand of the testat~r him~elf_
Notarial will - a notarized will signed by the decedent and witnesses
2· Codicil - a supplement or addition to a will, made after the ex~cuti~,~ of
3· a will and annexed to be taken as a part thereof, by which d1spos1t1on
made in the original will is explained, tlddcd to, or alter d {Art 825, lb1d.)

Every will must be acknowledged before , noti'lry public by the t s tator nnd
the witnesses. A hologr;iphic: will need not be witness d. A codicil nc d to
be executed as in the case of a will to be valid.

NATURE OF SUCCESSION
succession is a gratuit ous rran mission ot property trum ,1 decc,1scd person
in favor of his successors.

Succession involves only the 11er properut~'i uf till' dt•tedent The hei r will
inherit what ren1t1ins ot tht.• dt.•n•dt•11t 's prnpt•rt alter s.it1sty,ng the
decedent's indebted11es and obl,g.wons int lud111g the esl,He tax The heirs
shall not inherit the debt of the deredl•nt.

ELEMENTS OF SUCCE J0
1· Decedent - the general term applied ro the pt•r~on whos • property Is
transmtrted through succe!>SIOn, \~ht•ther or not he lett .1 will. It he left
Will, he 1s al o c,tlled the tescucor (Arl 775, lb1d,).

447
t of succession and Estate Tax
Chapter 13 - The ConceP
rope rty rig hts and obligations of the d
2 Estate - the P ' . "' d eced
. . . h d b his death. This is a1so re,erre to as the "i h f:!nt
ext111gu1s c Y n etitq ~n
the decedent. . . fie~ 1
3. Heirs - a pe rson called to th7e8s2u~~:J)swn either by the provision of ~
or by operation of la w (Art. , , .. a~'ill
WHO ARE THE HEIRS?

Heirs under intestate succession . .


In intestate s uccession, the heirs s hall be th e following m des cendin
· ·
o f pnonty: g Ot,1~et
1. Compulsory he irs . .
2. Relatives up to fifth degree of consangum1ty
3. Republic of the Philippines

The Jaw identified certain persons which it des ignated as "compu/so


These are the persons w ho will inhe rit the estate by default. Only? heirs.•
absence s hall the estate be partitioned to other r elatives. In the ab y their
senc
rela tives, the estate will go the government. eor

Types of compulsory heirs


1. Primary heirs: Legitimate children and their direct d escendants
2. Secondary heirs: Legitimate/illegitimate parents and ascendants
3. Concurring heirs: The s urviving spouse and illegitimate descendants

Definition ofterms
l . legitimate children are those born out of a legal marriage.
2. Direct descendants refer to children or, in their absence, grandchildren.
3. legitimate parents refer to biological parents.
4. /llegitimate parents are adopting parents to an adopted child.
5. The surviving spouse is the widow or widower of the decedent.
6. /llegitimate descendants are illegitimate children.

Note: Under the Revised Family Code, adoptive parents can now qualify as secondary heirs
sharing 50:50 with biological parents.

The secondary compulsory heirs shall inhe ri t only in default of the primary
heirs. Normally, only the pr imary heirs and concu rring heirs s hare in the
hereditary estate. In the absen ce of primary he irs, the secondary heirs and
concurring heirs s hall share in the he r editary estate.

In the a bsence of compulsory heirs, the fo llowing s hall inherit in the


following order of priority:
1. Collateral re latives up to the fi fth deg ree of consanguinity
2. The Philippine governm e nt

448
,,.. _rhe Concept of Succession and Estate Tax
3
cnaPter 1
. is given to collateral relatives in the closest degree.
prioritY
ry of Rules:
3
511111J11 urring heirs and
1. con~escendants, or in their default,
a. Ascendants
b, tives in the collatera l line up to fifth (5th) degree
z. Re1a blic of the Philippines
Repu
3.
111ustrat1·on
t
2 nd Priority

Spouse (Al)

1 st Priority

cs A3

3 rd Priority t
Model:
Al - Children C2 - 2nd degree relatives
A2 - Grandchildren (i.e., brothers and sisters)
A3 - Great grandchildren C3 - 3rd degree relatives
Bl - Parents (uncle, aunt, niece, nephew)
82 - Grandparents C4 - 4th degree relatives
83- Great grandparents (1 st cousins, 1 st cousins of grandparents)

Priority Levels:
1st
Priority- A = From Al onwards in descending order of priority
2nd
Priority- B = From Bl onwards in descending order of priority
:: Pr!0 r!ty- C = From C2 to CS in descending order of priority
Pnonty- Philippine government
Note:
1. 1
2. : includes both legitimate and illegitimate children.
econd cousins are in the 6th degree in the collateral line; hence, they cannot inherit.

449
f 5 ccession and Estate Tax
Chapter 13 - The concept o u

Illustration 1 . d by his wife and four children. Mr. X h


. H was survive
Mr. X died. e viving parent. as t\v
brothers and one sur a

The compulsory heirs are:


a. Mrs.X
b. The four children
d ry heirs) of Mr. X will not inherit because th
The surviving parent (secon a ere Qr
descendants (i.e.,four children). e

Illustra_tion _2 1 d without a child. Ms. X's parents, three brothers and


Ms. X died smg e an . ' tw0
sisters were her surviving relatives.

The compu Isory he1r. s are Ms· X's parents. The collateral
. relatives (brothersand
sisters) cannot inherit since there are compulsory heirs.

Illustration 3
Mr. y died a bachelor. He had no child. His parents ~ere all dead long beforehis
death. He only had a brother and a sister, a fi rst cousin, and a second cousin.
Since there is no compulsory heir, the brother and sister in the co/lateral line shall
inherit Without them, the first cousin shall inherit
Assuming further that only the second cousin is living, the government shall
inherit the estate. Succession in the collateral line cannot extend to the second
cousin because he/she is beyond the fifth degree of consanguinity.

Basic Intestate Partition Procedures


1. The decedent and the surviving spouse s ha ll fi rs t share in their common
properties.

The common pr~perties net of expenses and obligations chargeable to the


common properties of the spouses is divided between the decedent and the
surviving spouse.

2. Determination of the decedent's net inte rest.


The deced t' ·
a E ~n s net mterest comprising of the following is computed:
. xc1us1ve property of the decedent
b. Share of the decedent in the net co .
mmon proper ties
3. Partition of the decedent's net . t
a. Surviving spouse m erest to t he heirs:
b. Legitimate children
c. Illegitimate children

450
-The Concept of Succession a d E
chapter 13 n state Tax

urviving s pouse and each of the h' ld


fhC _s . te child is e ntitled to . h If c I rcn have one share each. Each
;11cg1t 11n~ a a s hare.

~,ration 1 .
Jlh'· HrchiC Ri ch, a widower, died leaving P45 000 000 t t ~ h . tw
~1r children, Haro ld and Al . , . '. es a e or is o
Jegi1J111,1te ex, a nd two 11leg1t1mate children, Elon and
Jl1hll•
state sha ll be partitioned as follows·
rhee .

H.cif.S ~ Partition lnhe[itan1::e


tnd lwo H,irold (legitimate) 1.0 1/3 X P45,000,000 P 15,000,000
Afex (legitimate) 1.0 1/3 X P45,000,000 15,000,000
Elon (illegitimate) 0.5 .5/3 X P45,000,000 7,500,000
er-s and John (tllegidmate) ~ .5/3 x P45,000,000 7,500,QQ0
Total 1U P 1:5,000,000

Illustration 2
b re his
:in. Don Uyong died leaving behind his widow, Mrs. Uyong, his legitimate children,
Max and Rey, and illegitimate children, Joan, Sylvia and Eunice, as heirs. Mr. and
Mrs Uyong had the following properties:

Exclusive properties of Don Uyong P 18,000,000


t shall Exclusive properties of Mrs. Uyong 16,000,000
second Net common properties 36,000,000

The net estate of the decedent shall be computed as follows:


mmon Exclusive properties of Don Uyong P 18,000,000
Share in net common properties (P36M/2) 18,000,000
to the Distributable estate P 36,000,000
nd the
The estate shall be pa rtitioned as follows:

Heirs Share Partition Inheritance


Mrs. Uyong (spouse) 1.0 1/4.5 x P36,000,000 P 8,000,000
Max (legitimate) 1.0 1/4.5 x P36,000,000 8,000,000
Rey (legitimate) 1.0 1/4.5 x P36,000,000 8,000,000
Joan(illegitimate) 0.5 .5/4.5 x P36,000,000 4,000,000
Sylv_ia (illegitimate) .5/4.5 x P36,000,000 4,000,000
0.5
Eunice (illegitimate) .5/4.5 x P36,000,000 4,000,000
0.5
TotaJ P 36.000,000
.4..5.

451
The total properties
t of Suq::essioo an~ E~tate Tax
Chapter 13 -The Co.ncep , .. 'f
,=.i~ ~
l

t . :·• ,;,f_'::· -:
. ·ng spouse after the partition shall be· -,•:':- ;
· Ofthe sul"VlVI • ;l;,'f.:
~~,,, . . ' '
, :, ;: .
~..;· ~ -7,
,.,

. . · -~,.
· · · . fMrs Uyong P 16,0oo,ooh ,~ ---1:
Exclusive properties o . . (P36M 121
. net common properties
Share m 1· 1 ia,ooo ooo ·-
Share in the distributable estate ~ 1
Total properties £,gnoo~ .".
••
ta Disgosition
Heirs under Testamen_ fY h heirs shall be the following:
In testamentary successwn, t e
1. Compulsory heirs . ecedent in h is will
2. Other persons specified by the d

Legitime , rty h · h h
Le itime is that part of the testator s prop e . w ic e cannot dispose
be~ause the law has reserved it for certam he irs who_ are, the refore, cau of
compulsory heirs (Art 886, Ibid). The excess properties of the decedent~
called "free portion".

By means of a last will and testam~nt, _a testat?r can d~signate the free
portion of his estate for additional heirs 1rrespect1ve of t~eir r~lationshipto
him but he cannot exclude or disinherit compulsory heirs without a Valid
basis under the law.

Disinheritance and Repudiatio n


A decedent can actually disinherit an heir on certain grounds . Similarly,
heirs may repudiate their sha re in the inheritance of the decedent.

The rules on legitime, free portion, disinheritance of ~m heir or repudiation


of inheritance are matters of law which arc irrclev<rnt to estate taxation.
Hence, these topics will not be emphasized in our discussion. Readers with
particular interest in these matters are advised to consult Title /Vo/ Book/II
of the Civil Code.

The determination of the estate tax docs not requll"t' prior identification of
the heirs. Once a person is dead, the estate ol the decedent is simply
determined and reduced by deductions allowl•d by J,nv. Then, the estate tax
is computed ,out ?f the net estate. Neither does the v.tl,dity or invalidity of
the decedent s will nor the absence of an heir ,1 ttect est<l le taxation. In fact.
~he e~tate tax is due even if the decedent does not have rel<1tives who will
mhent the property.

Furthermore the determ 1·nat:1· f h . ·b .-.ble


• ' c on o t es 11.11 col t•.1rh heir 111 the d1stn U1-<1
estate 1s done only after all -h- - , - estate
tax, had been deducted. < c t1iges to the twredit ' 1rv - L' s t ' ill' ' 11ulud1ng

452
"'" 13 - rhe Concept of Success
c~apter ion and Estate Tax
ersons in succession
her P 1 .
011• (]tee - a person w 10 m gifts of
/,cg personal pro , . .
1 ,viii perty 1s given by virtue of
1
' ee. a pe rson who m gifts of re 1
,,
fJt!' • a property 1·5 ·
z. ,.,. ,~cutors - a pe rson named by the d, given by virtue of a will
3- ·X.~visions of hi s will ccedent who shall carry out the
pl
AdJ11illtSl'.ators - a person a ppointed by
th
4 distribution of t he estate of the decedent e court to manage the

ESfAfE TA~TION ta ·
te taxat10n pe r ms to the taxation f
£Sfa rties of the decede nt to the heirs oh th e gratuitous tra nsfer of
prope upon t e decedent's death.
state taxation is governed by the law in force at th .
£. ti1 The estate tax accrues as of th d e time of the decedent's
dea · . . e ecedent's death and the accrual of
ca:< is d1stmct from the obligation to pay the s U h
the · k ame. pon t e death of the
decedent, s uccess1? n hta es place and the right of the State to tax the
privilege to transmit t e estate vests instantly upon death.

Hence,
Between January 1, 1998 to On or after January 1,
Decedents who died December 31, 2017 2018
Shall be _qoverned by The NIRC The TRAIN law
-
Readers who are co~cerned a bout estate tax of decedents covered by the
NIRC are hereby advised to consult the old edition of the book. .

NA TURE OF ESTATE TAX:


1. Excise tax- estate tax is not a tax on the property but on the privilege to
transfer property through death
2. Revenue or general tax - estate tax is intended as a revenue or fiscal
measure
3. Ad valorem tax - estate tax is dependent upon the value of the estate
4. National tax - estate tax is imposed by the national government
5. Proportional tax- estate tax is imposed as 6% on the net estate
6. One-time tax - estate tax applies to a person only once in a lifetime

Classification of Decedents for Taxation Purposes


1. Resident or Citizen Decedents- taxable on properties located within or
outside the Philippines . .
2. Non-resident Alien Decedents- taxable only on properties located m
..
th e Philippines, . "bl sonal property when the
except mtangi e per
reciprocity rule applies

453
Chapter 13 -The Concept of Succession and Estate Tax

THE ESTATE TAX MODEL

Gross estate p xxx,xxx


Less: Deductions from gross estate xxx,x~
Net taxable estate p XXX.XJQC

"Gross Estate" pertains to the totality of the properties owned


decedent at the point of his death. by tie

There are two concepts to be discussed under gross estate:


a. Exclusions in gross estate - those properties or transfers excluded
from estate taxation by law
b . Inclusions in gross estate - those properties which are to be includ
part of the taxable gross estate edas

The concept of gross estate will be extensively discussed in chapter


s 13-A
and 13-B.

"Deductions" generally pertain to reductions in the inheritance of the heirs


such as obligations of the decedent, and losses of property durina
administration, but also include exemptions from the estate tax under th;
law. Deductions are extensively discussed in Chapter 14.

"Net taxable estate" is the net properties of the decedent after all pertinent
deductions allowable by law that is subject to tax. Note that the net taxable
estate is not equivalent to the hereditary estate before estate tax because or
exclusions, exemption rules and deduction criteria imposed by the law.

The determination of estate tax, including other pertinent tax reportorial


requirements, are extensively discussed in Chapter 15.

454
13 -The Concept of Succession and Estate Tax
c~a Prer
f.ER t3: SELF-TEST EXERCISES

~ Questions
. .
tJSSI 00 . ?
piSC tis success10n .
Wha .
1. h tis inheritance.1
z. W arnerate a nd discuss each of the types of succession.
3- sn; tis a will? Discuss briefly.
4. inuarner~te and explain each of the e lemen_ts of s uccession. .
s. B •eflYdiscuss the types of compulsory he1rs and their order of success10n.
6. Br:eflY discuss the sha ring of the inheritance under intestate su ccession.
1 r .. ?
· What is a leg1t1me ..
8. What is estate taxat10n?
9· Enurnerate a nd expla in the nature of estate tax.
:~· What are the two classifications of decedents? Discuss their estate taxation.
. Jllustrate the estate tax model.
12.

rrue or False 1
1. succession is a mode of acquisition of property s imila r to donation.
2. Inheritance refers to the property which will be transm itted to the heirs.
3. The right to succession is transmitted from the moment of death of the
decedent.
4. Adecedent with a last will and testament is said to be intestate.
s. The succession over properties of a decedent who prepared a will but
covers only a portio n of his esta te is called mixed succession.
6. A person who prepa red a will is refe rred to as the executor.
7. A person who di ed with a will is said t o be testate.
8. The successio n by operatio n o f law is called intesta te s uccession.
9. The will may be pre pared by the heirs of the decedent.
10. The will may be pre pared a fte r the death of the decede nt.
11. A testator can designate a ny he irs even if in violation of his legitime.
12. In intestate s uccession, t he s urviving spo use is treated as a leg1t1mate c hild
qualified for a s hare w hile illegitima te chil dre n are a llowe d ha lt - hare each.
13. The computc1 Li o n of the es tate requires und e rsta nding of th e leg1time.

True or False 2
I. A debt can be inherited by heirs.
2· Only testa mentary di pos itions a re subj ect to estate tax.
3. Successio ns whe ther testa mentary, intesta t e, or mixed a re subject to estate
tax.
4
· The decedent's successors in interest are referred to as the hei rs.
5
· With a last will and testa m e nt, th e deced e nt can name any person which he
Wants as hei r.
6
· In testamentary dis pos ition, the he irs mus t always be rela tives of the
decedent.

455

~-1 ~
Chapter 13 -The Concept of Succession and Estate Tax

7 _ The secondary compulsory heirs shall inherit only in default of c


0
heirs. . . . nclllrj
8. The concurring heirs shall m~ent together with the primary h . ~l
their default, the secondary heirs. . . eirs, Ot
9. In default of concurring heirs, relatives m the collateral line u ~
degree shall inherit. p to the.
10. In default of compulsory heirs, the government shall inherit the i~

the decedent. estqte


01

Multiple Choice - Theory 1


1. IT is a mode of gratuitous acquisition of property out of the genera .
person sity of 1
a. Succession c. Loan
b. Donation d. None of these

2. It is a mode of gratuitous acquisition of property by the death of ad


a. Donation c. Inheritance ecedent
b. Succession d. Testate

3. The donor in a donation mortis causa is


a. dead. c. either living or dead.
b. living. d. in a coma.

4. Which is not a form of succession?


a. Testamentary c. Intestate
b. Compulsory d. Mi xed

5. It is succession in the absence of a will.


a. Testamentary c. Mixed
b. Intestate d. None of th ese

6. It is succession by will and operation of law.


a. Testamentary c. Mixed
b. Intestate d. None of these

7. It_is ~ritten document which sets forth how the decedcnl's property will be
d1stnbuted after death.
a. Will
b. Testamentary disposition
c. Intestate disposition
d. Mixed

8. Which is not an element of succession7


a. Decedent t
c. Esate ·
b. Estate tax d H .
. e1rs

456
- The Concept of Succession and Estate Tax
ter 13
cn"P. he property, righ
. ts a nd obltgat1ons
. . of the decede nt not extinguished by
It ,s t
9. 1.5 death.
11 Estate c. Codi cil
.i, Esta te tax d. Legitime
b,
are the successors in interest of the decedent
JO·
~~
Heirs
'
c. Dev1sees
.
"· Estate d. Legatees
b.
. a do nee of a real property in a last will a nd testament
It JS •
11· Heir c. Legatee
~: oevisee d. Successor

_rT is a do nee of personal property in a last will and testament.


12 Heir c. Legatee
a.
b. oevisee d. Successor

Who is not a compulsory heir?


13· a. An adopted child
b. Children of a deceased child
c. The surviving spouse
d. A brother

14. Who will inherit among the followi ng?


a. A legitimate child c. Grandchildren
b. Parents d. First cousin

15. Which of the following heirs will not inherit?


a. Illegitimate child c. Nephew
b. Surviving spouse d. Legitimate child

Multiple Choice - Theory 2


1. Which will not inherit from the following group?
a. Grandchild from a deceased son of the decedent
b. A daughter of the decedent
c. Grandson from a living daughter of the decedent
d. An illegitimate child of the decedent

2· Who may not inherit under an intestate succession?


a. Grandparents c. First cousin
b. Grandchildren d. Best friends
3
· Who will be disinherited from the following?
~- Wi~o_w c. Illegitimate children
· Leg1t1mate children d. Parents

457
Chapter 13-The Concept of Succession arid Estate Tax

4. Who shall inherit from the following group?


a. Legitimate child c. Gra nd parents .
b. Parents d. Brothers and sisters

5. Who shall inherit from the following potential heirs?


a. 1st cousins c. Niece sand nephews
b. Brothers and sisters d. Second cousins

6. This is a person appointed by the court to manage the distribution of H.


estate. ~ie
a. Administrator c. Guardian
b. Executor d. Trustee

7. This is a person handpicked by the decedent to implement his will.


a. Administrator c. Guardian
b. Executor d. Trustee

8. There are several estate tax laws in the history of estate taxation in th
Philippines. Which applies to a particular decedent? e
a. The estate tax law in effect when the properties of the decedent are
being accumulated
b. The estate tax law in effect at the date of the decedent's death
c. The estate tax law in effect at the date the last will and testament was
prepared
d. The estate tax law in effect upon determination of the legal heirs
9. Which is correct regarding estate tax?
a. A privilege tax c. A final tax 16.
b. A national tax d. A fiscal tax

10. Statement 1: Succession will not effect until and unless the estate tax is paid.
Statement 2: Estate tax is payable even in the absence of relatives who may
inherit the estate.
Which is correct? 17.
a. Statement 1 c. Both statements
b. Statement 2 d. Neither statement

11. Statement 1: No estate tax is due if the net taxable estate is negative.
Statement 2: Once there is death, the estate tax is always payable.
Which is incorrect?
a. Statement 1 c. Both statements 18.
b. Statement 2 d. Neither statement

12. Statement 1: A resident citizen is taxable on his estate wherever situated . .


Statement 2: A non-resident citizen is taxable only on his estate situated in
the Philippines.

458
-The Concept of Succession a d E
cnaPter 13 n state Tax
. his correct?
w111c
statement 1 c. Both statements
a. Statement 2 d. Neither statement
b,
,nent 1: A resident alien is tax bl
3 State . a e only on his estate situated in the
1 · pnilippmes.
,nent 2: A non-resident alien ·
State . . . is taxable only on his estate situated in
t11e Ph1hppmes.

Wllich is correct?
a. Statement 1 c. Both statements
b. Statement 2 d. Neither statement

14. The recipro~ity ex~mption on intangible personal properties situated in the


Philippines 1s applicable only to a
a. Non-resident citizen
b. Non-resident alien
c. Resident alien
d. All of these

is. Which is correct?


a. Estate tax is determined per piece of property transferred to the heir.
b. Estate tax is collected by the local government.
c. Estate tax is a property tax.
d. Estate tax is an excise tax.

16. A decedent died intestate with Pl,000,000 net estate. If he has four
legitimate children and two illegitimate children, how much shall each
legitimate and each illegitimate child respectively receive?
a. PO; PO c. P 200,000; Pl00,000
b. P 100, PS0,000 d. P 166,667; P166,667

17. A married decedent died intestate leaving behind Pl,500,000 of his


separate property and P6,000,000 common properties with his surviving
spouse. If he has t hree children and one illegitimate child. How much shall
each legitimate child receive?
a. P 1,666,667 c. P 666,667
b. P 1,000,000 d. P 333,333

18. In the immediately preceding problem, compute the total properties of the
surviving spouse after partition of the properties.
a. P 3,000,000 c. P 4,666,667
b. P 4,000,000 d. P 3,666,667

459
Chapter 13-A- Estate Tax: Gross E5tate

CHAPTER 13-A
ESTATE TAX: GROSS ESTATE
Chapter Overview and Objectives
~;~;;~i;·~~:;;~~:·;~:~;;;·;;~·~;;;~~d to comprehend: ·············-----.
1. The concept of gross estate
2. The procedures in determining gross estate
3. The list of exempt transfers
4. The list of taxable transfers
. The treatment of mortis causa transfers made for in su ffi cient
5
consideration
6. The rules of valuation of gross estate

GROSS ESTATE
Gross estate consists of all properties of the decedent, tangible or intangibl
real or personal, and wherever situated at the point of d eath. e,

In the case of a non-resident alien decedent, gross es ta te includes O 1


properties situated in the Philippines except inta ng ible personal prope~
when the reciprocity rule applies.

Summary of rules on gross estate


Residents or NRA without NRA with
Citizens reciprocity reciprocity
Prooerty location ➔ Within Ab.rQad. Within Qu.tsid.e. Within Qu~ide
Real properties ✓ ✓ ✓ X ✓ X
Personal properties
- Tangible ✓ ✓ ✓ X ✓ X
- Intani2ible ✓ ✓ ✓ X X X

Illustration
A decedent died leaving the followi ng property:

Location
Philippines Abroad Total -
Real property p 2,000,000 p 3,000,000 P s,000,000
Tangible personal property 1,000,000 500,000 1,soo,000
Intangible personal property
800.000 1.200.000 2,000.000
Total
p 3,800,000 p 4.700,000 P 8,500.{W

460

t
ter 13-A-Estate Tax: Gross Estat
cnaP e
Estate:
ross . .
G decedent 1s a:
1ftl1~ sident citizen, non-resident c·t·
l'e. .d 1· . t izen or .
J. Non-res1 ent a 1e n without recip . resident alien _ p 8 5
b- ~,on-resident alien with recipro :ocity - P 3,8OO,000 ' OO,OOO
C. I' City - p 3,000,000

RocEDURES IN ESTABLISHING GROS


p inventory count of exis ting propertie~ ESTATE.
1· Adjustments for exempt transfer at the point of death
2. s a nd taxable transfers
inventory of Properties
To establish the amount of the gross estat .
the decedent and t h eir fa ir values at
established.
;;,:npomt
m~entory of the properties of
0/death shall first be

Date of death

- t
(Inventory count)
)

If the list of properties


. l existing at the point of death is known, th e 1·1st 1s
·
simply d rawn d irect y.

However, if the inventory is prepared as of a later date after the decedent's


death, the inventory must be worked back to establish the list of properties
present at the point of death.

lllustration
Adecedent died on June 30, 2019. An inventory was not immediately prepared
because of the funeral of the decedent. An inventory count of his properties was
drawn only on July 15, 2019.

On July 15, there were properties which had a total fair value of PS,000,000.
Pl00,000 of this represents income earned after death while P400,000
represents income earned before death.

Atotal of PSOO' OOO was pa1·d ,,'or funeral expenses and judicial expenses
.d .
of the
h" d th
estate. A total of PZ00,000 obligations of the decedent was pa1 smce is ea .

the gross estate shall be recomputed as:


p 5,000,000
~roperties as of July 15, 2019 100,000
I\~:_:
Increase in properties si_nce dea th K + p2ooKJ 7 00,000
p · Decrease in properties smce death (PSOO ) p 5.600,000
roperties existing at the date of death {Gross ESt3te
461
Chapter 13-A-Estate Tax: Gross Estate

Note:
1 Properties representing income earned before death properly form P
. h . art of th
estate of the decedent because these were present at t e point of death e gr
2. Properties representing income accruing after death must be excluded sin °'·l
were not yet present at the point of death: . ce th~
3. Expenses or obligations which were paid since death must be added b '
these were present at the point of death. ack Sinct

THE GROSS ESTATE FORMULA:

Inventory of properties at the point of death p xxx,xxx


Less: Exempt transfers
Properties not owned p xxx,xxx 1ne r
.r,-1
11S/JJ'
Properties owned but excluded by law xxx.xxx XXX.)(~
Inventory of taxable present properties p xxx,xxx fhe ti
Add: Taxable transfers xxx,xxx does'
GROSS ESTATE P xxx,x,g cotM
ote i
EXEMPT TRANSFERS naked
1. Transfers of properties not owned by the decedent - AsubJ
One cannot transfer properties he or she does not own. Properties not
owned by the decedent are not part of his/her donation mortjs causa
These properties must be excluded in gross estate even if they transfer
to other persons at the point of death.
111ust1
2. Transfers legally excluded- Mr A 1
There are properties that are owned by the decedent at the point of grand·
death. These properties natu rally form part o f his/her donation mortis as tidu
causa to the heirs, but are exempted by t he law from estate taxation.
Hence, these are excluded from gross estate. Pr
These referred to as exclusion in gross estate.

Transfer of properties not owned by the decedent


1. Merger of the usufruct in the owner of the naked title The d1
2. The transmission or delivery of the inheritance or legacy by the U1de1<.1
fiduciary h eir or legatee to the fideicommissary tran\/t·
3. The tra11smissio11 from the first heir, legatee, or donee in favor 0/ Thede
another beneficiary, in accordance with the de ire of the predecessor
4. Proceeds of irrevocable life insurance policy payable to benefida,Y The t1
other than the estate, executor or administrator anoth,
5. Properties held in trust by the decedent IUu,tr,
6. Separate properties of the surviving spouse of the decedent
7. Tran~Jer by way of bona fide sales
462
~ cnaP
rer
H-A- Estate Tax: Gross Estate

..ocr of usufructin the owner of tiie na, ked title


,11e,,, .
f~e
tiOO
1lt1str3 _ d ·n
I Ju ne 2011. In his will h d ·
\ 1\ d1C operty over 10 ' e evised an agricultural land to f3 who
\r;II :h~
l1sc P\ransmission of ~:ars and thereafter, to C. Subsequently, died
' '. 1ting ,n ,e e pro perty to C.
8
r~~ll .
pre~essor Curren[5cedent C

(Usufructuary) (Owner of naked title)

The transfer of th e devise from 8 to C is referred to in law as the 'merger of the


,r, ct in the owner of the naked title."
~rn .
The transfer f ~om th e usuf~uctuary,_ B, to the real owner, C, upon the death of B
does not constitute a don~t~on mort1s causa as it is a mere return of the property
ro the real owner. Hence, it 1s excluded from gross estate.
Note that the t~ansfer from Mr. A, the predecessor, of the usufruct to B and the
naked title to C mvolves transfer of ownership. It is a donation mortis causa of Mr.
Asubject to estate tax.

The transm1ss10n or delivery of the inheritance or legacy by the


fiduciary heir or legatee to the fideicommissary

mustration
Mr. Adied leaving an inheritance consisting of several real estates to his favorite
grandson, C from his favorite son, B. Because C was a minor, Mr. A appointed B,
as fiduciary of the inheritance. Before transferring the property to C, B died.

P1t i•-ss_o_r_ _ _ _c_u_r_r•1r1c-ed_e_n_t_ __ _ _


C
(Fiduciary heir) (Fideicommissary)

The delivery of the inheritance upon the death of B (fiduciary heir), to C


(ftdeicommissary) shall not be included in the gross estate of B because the
transfer does not involve a transfer of ownership from B to C. B is merely a trustee.
The delivery is a mere return of the property to the real owner, C.

The transmission from the first heir, legatee, or donee in favor of


another beneficiary, in accordance with the desire of tqe predecessor
Illustration

463
Chapter 13-A-Estate Tax: Gross Estate

In his will, Mr. A devised a piece of land to B as the first heir a d


as the second heir. B subsequently died transmitting the n th ereafter
accordance with Mr. A's will. Property to ~oC

[;]
Predecessor C
urrel;lce
td d t i~
en

(1 st heir)
C

{2 nd heir)
I
The ~ans/er from B to C is referred to as transfer under a special
~ppomtment The same is not B's donation mortis causa. The transfer
1
:a we,.8 01
0

ts merely an implementation of the transfer which was originally m rn toe


predecessor A. andated by

The same rule applies even if B were given the power, solely or in conjuncti .
others, to appoint the second heir to the property from a list drawn by pre/n 'With
A. ecessor

In all previous illustrations, assuming B trans ferre d th e pro pe rty during h'
15
lifetime to C, the same shall not be subject to d onor's tax because there is
·
gratmtous trans f er o f owners h.1p. no

Proceeds of irrevocable life insurance policy payable to beneficiary


other than the estate, executor or administrator
The proceeds of life insurance policies w hich a re irrevocably designated by
the decedent to the beneficiary are no longer owned by the decedent at the
point of his/her death. They are ow ned by t he beneficiary des ignated by the
decedent. Hence, these shall not be included in gross estate.

The proceeds of life insuran ce policies w hich are revocably designated by


the decedent to any beneficia ry a re owned by the decedent at the point of
his/her death. He nce, the proceeds a re included in gross estate.

If the decedent named a be nefi ciary witho ut indicating whether th e


designation is revocable or irrevocable, the designation is presumed ~o ~e
revocable. However, if the decede nt did no t replace the beneficiary until his
death, the designation shall be deemed irre vocable exempt from estate tax.

Estate executor or administrator as beneficiary the


' . . · · trator,
If the bene fici a ry designate d 1s the estate, executor or admints f rhe
0
roceeds of life ins ura n ce is included in gross estate rega rd1 ess .dered
P ~ . . are cons1
designation of the benefi cia ry because t hese bene f1c1anes
extensions of the interest of the decede nt.

464
r C11~pte
r 13-A- Estate Tax: Gross Estat
e

arv of Rules: Proce~rls o ff trr 1


rflln . ' ,, n.~urm ,cu
tl ----- - --

,f,d,ll r l) P'iignatlon or'hc·~dh:iary ~


B011 t
. •l t ,1d1111 n1-, t 1,1 ot , _Q I t' \l'U!_I C
_)r Ut:J:JJJ.._(UJJi±
~
, - - /Jrr••uuclli.1
~~-- HJ,c·e.
l•
l'"
l1Hl1tdf' I 111· I IH It' -
Il
Olt , r pJt ·.:_._--------l.-.__;l11cl u d c --
Exclude

0111<;,1 r.-1tion . •
fubod d1l'd. His h e irs coll ected the foll .
r owing pro ceeds o f life insurance
roJ c,r'
/,.XA. re,·ocahly designated to wife p 800,000
Manuhfr. irrrvoca b ly d esigna ted to daughter 600,000
onhft'. revocably d esignated to Mr. Tu bod's estate 700,000
fhtlAm, irrevoca bly designa ted to Mr. Tubod's executor 400,000

The proceeds of ins ura nce policies to be included in gross estate shall be:
.@ revocably designated to wife p 800,000
unh fe, revocably d esignated to Mr. Tu bod's estate 700,000
Ph !Am, irrevocably designated to Mr. Tubod's executor 400,000
Total P 1,900,000
1>,ote:
1 Only the proceeds of ins urance policies that are revocably designated are included in
gross estate.
?.. However, if the be ne fi ciary is the estate, executor or administrator, the proceeds are
included in gross estate wi thout regard of the designation of the beneficiary as revocable
or irrevocable.

Properties held in trust by the decedent


Properties held in trust by the decedent at the point of his death are not
owned by him. These are excluded in gross estate because these will not
form part of the decedent's donation mortis causa to the heirs.

illustration
The following properties were identified upon the death of Mr. Ubaldo:
P 800,000
Car, registered in the name of his brother
200,000
Merchandise, consigned to Mr. Ubaldo
2,400,000
House and lot
150,000
Motorcycle, borrowed from a friend
4,000,000
Boarding house held as trustee
Taxicab , 1,000,000
Tax·icab franchise 600,000
. 400,000
Clothes, books, equipment, and other personal belongings

465
Chapter 13-A-Estate Tax: Gross Estate

The gross estate of the decedent shall consist of the following:


House and lot fU,,t'ffiJtlJ.U p Z,400,000 ~ ~
Taxicab l ,000,000
Taxicab franchise . 6 00,000
Clothes, books, equipment and other personal belongmgs 400,00Q
Gross Estate P 4 ,4 00,0()Jl
Gross estate shall not include properties held in trust by the decedent induct·
other properties he does not own. Hence, the following shall be excluded fr~
gross estate.
Car, registered in the name of his brother P 800,000
Merchandise, consigned to Mr. Ubaldo 200,000
Motorcycle, borrowed from a friend 150,000 1
Boarding house, held as trustee 4.000,00Q
1
Total exclusions in gross estate P 5.150.000
g
Separate properties of the surviving spouse 0
{I
Spouses have their separate properties and commo n pro pe rties. Common
properties are owned jointly by the s po uses while sepa ra t e or excJusive
properties are sole ly owned by eithe r of them .

The separate or exclus ive pro perties ol the h usband .,,re referred to as
"husband's capital" while tha t ol the will' ,s referred to s "wife~
paraphernal."

The wife's para phe rna l s ha ll not be included 111 the gross estate of the
hus band upon his death s ince these will not form part of his donntton mortis
ca usa . Simi la rly, th e husband's capital sh.ill not be included in the gross
estate of the wife upon her death on the same basis.

The gr oss estate of a ma rried decedent mclud0s the Sl'p,tt".ltt' propert1es of


the d eced ent a nd their common properties with the surv,vmg spouse.

Illustration
An inve nto ry of the properties at the po111t ot de,llh ot t-lr. C,tbilt r~vc~tled the
fo llowing:

Exclusive properties o f Mr. Cabili


P 2,400,000
Exclus ive pro perties of Mrs. Cabili
4,000,000
Common properti es of Mr. and Mrs. Cabll,
To tal properties of the spo uses J.000.000
P l ·t.400,000

466

A
13-A-Estate Tax: Gross Estate
chapter
estate of Mr. Cabili shall be:
r11egr055
·ve properties of Mr. Cabili
6>--clLJSI n properties
.
o f Mr. and Mrs. Cabili P 2,400,000
conimo 8,000,000
55
estate
Gro Pl0.400,000
Noie: uounda1y between exclusive and com .
J. The rty rela tions which will be di , _ d ~on properties of the spouses is a question of
prope . scusse 111 the following cha ter
e cornmo n prope rties are jointly ~ ·
z. Th 111 .. . owned
. by th ..
e spouses. l he share of the surviving
°
spouse ~he_n~tt· c m~oln pdrodp~rties is not a n item of exclusion but an it em of deduction.
e it 1s m1 th1a 11y me u e m the gross estate then later removed as an item of
1-{enc ,
deduction from e gross estate.

rransfer by way of bona fide sales


Transfers by way of bona fide sales a re onerous transactions rather than
gratuitous transactions; hence, they are not s ubject to estate tax. Moreover,
ownership over properties sold normally passes on to the buyer
immediately at the point of sale.

Hence, properties transferred by way of bona fide sale or for an adequate


consideration are excluded in gross estate because the decedent no longe r
owns them at the date of his/her death.

Legal exclusions
The following are the list of properties owned by the decedent at the point
of death which naturally forms part of the hereditary estate but are not
subjected to estate tax by Jaw:
1. Proceeds of group insurance taken out by a company for its employees
2. Proceed of GSIS policy or benefits from GSIS
3. Accruals from SSS
4. United States Veterans Administration (USVA) benefits - RA 136
5. War damage payments
6. All bequests, devises, legacies or transfers to social welfare, cultura l and
charitable institutions, no part of net income of which inures to the
benefit of any individual; provided, however, that not more than 30% of
the said bequest, devises, legacies or transfers shall be used by such
institutions for administration purposes
7· Acquisitions and/or transfers expressly declared as non-taxable by law
8· Bank deposits withdrawn from the decedent account during the
settlement of the estate

These properties mus t be removed from the gross estate of the decedent.

467
Chapter 13·A-estat T x· Gross Estate
ot on prop rty acqul ittons ustng exempt benefits 1
Prop rttes acquired using GSIS benefits, SSS accruals, USVA benen f
proceeds of group Insurance and war damage payments are still exernp ~. 1
tong as the heirs or administrators can prove th at the properties t So (

cqulred using the exempt properties.


Were (
(

ote on bequests, device, or legacies to social welfare, culturaJ, a11


charitable Institutions d
The conditional exclusion applies if the do nee i?s.tituti?n uses not more tha
30% of the bequest, device, or legacies for admm1stration purposes. n
The 30% conditional exclusion is deemed satisfied if the donee is
accredited non-profit donee institution._If the do nee is a qualified non-pro~~
donee institution, the same is excluded m gross estate.
I
f
It must be noted that one of the primary requirements for the accreditati e
of donee institutions is that their income does not inure to the benefit of a~n C
private individual and that the level of their administration expenses doe~
1
not exceed 30% of their total expenses.
a
Transfers to these institutions are initially included in the inventory list of \I
taxable properties, but are removed from the list if the do nee is verified as a )
qualified donee institution. f
t
If the transfer qualifies for exclusion, the same is not reflected in both gross s
estate and deduction. It must be noted that there is no item of deduction for !
such transfer under the Tax Code and in the estate tax return.
I
Despite this, bequests, devises or legacies which are restricted by the 1
decedent for administrative expenses of the donee institution (whether
accredited or non-accredited) shall be included in gross estate.

Illustration
A decedent had the following properties:
Family home P 5,000,000
Truck 1,200.000
Cash
200,000
Commercial land
800,000
Other properties
600,000

In hhis will, the decedent designated the cash to be given to a public elementary
sc ool. . The co?1mercial land was also devised to a non-profit charitable
mst1tut1on restricted to be used for pro
gram expenses of the latter.

468


r- c~apter
J3-A-Estate Tax: Gross Estate

estate shall be computed as·


g ross .
fne
·1y home P s,000,000
farJ11
frtJC~
1,200,000
200,000
c~sh roperties
]ler P 600,000
0t tate E 7,000,0QQ
Gross es

No1e: bequests, devises or descent to


on 1Y social welfare, cultural a nd charitable
1, ·nstitutions are exempt.
1
, rs to the government and its i t ·•
frans,e . Th . ns rumentaht1es are not items of exclusion but
2, . of deduction. ey are mcluded in g
11ems . f . ross estate and then separately presented
as deducttons rom gross estate m the estate tax return.

oeposits withdrawn from ~e decedent's bank account


previously under the NIRC, withdrawal from the bank account is prohibited
except withdrawal of up to P20,000 for the funeral expenses of the
decedent.
The TRAIN law allows unlimited withdrawal from the decedent's ba~k
account but requires bank with knowledge of the decedent's death to
withhold 6% final withholding tax upon the withdrawal if made within one
year form the decedent's death (RRS-2019). The 6% withholding tax is a
final tax and is non-creditable. As such, amounts subjected to the 6% final
tax must be excluded in gross estate. However, if such withdrawal is not
subjected to the 6% final tax, the amount of withdrawal must be included in
gross estate.

Illustration 1
The following withdrawals were made from the bank account of the decedent
who died July 8, 2019:

July7,2019 P 200,000 For payment of medical expense


July 9, 2019 300,000 For payment of funeral expense
July 10, 2020 500,000 For payment of claims against the estate
Ending balance P4,000,000
The P300,000 shall be excluded in gross estate. The amount of cash in bank to be
reported in gross estate shall be P3,S00,000.
Note:
I. The P200,000 withdrawal is not part of gross estate as it is expen~ed before the death of
the decedent This is not s ubject to 6% final tax since the decedent is not yet dead.
2· Assuming th;t the bank is informed about the decedent's d.eath, the P300,000 shall be
subjected by the bank to 6 % final tax and shall be excluded in gross estate by the estate
ad,n·1 1·
3 n ster, executor or heirs. • · · b d th 1
. ;he '.500,000 shall no longer be subjected to 6% fi.nal tax sm_ce it is eyon e -year
equ1rement. The estate tax might have even been paid by that time.
469
Chapter 13-A- Estate Tax: Gross Estate

Illustration 2
Mr. Jo and Mr. Kang has a joint account ~ith_ BPI for _the ir busi~ess Ventur
where they s hare equal interest. Mr. Jo died m an accident. A w1thdra\Val E
Pl00,000 is being made against the account Of

Since half of the bank interest will effectively be included in th e gross estate of Mr
Jo, the bank shall withhold the 6% final tax only on l / 2 of the PJoo,ooo
withdrawal.
Illustration 3
Assume the same data in the foregoing illus?"atio n except that Mr. Liam Mado
died and that the account is a joint account with Ms. Mado.

The bank shall withhold the 6% final tax upon the total P100,000 balance since
the entire amount will be reflected in halves under separate and common
properties columns of the gross escate.
It mus t be noted that withdrawal which were not subJected by the bank to
the 6% final tax must be included in gross estate.

The 6% fina l tax treatment s hould be a caveat to heirs or est.tie


ad ministrators. Withdrawal from the decedent'-; a ccounts 1c., no t ad\1 .able if
the decede nt is projected to have a zero or neg.Hive ncr tax.1ble estate.

TAXABLE TRANSFERS
Taxab le transfers are mortis r ,1ui;,1 tr,rn,ft•rs nl propert1t•c; in th C' gut. ~ and
form of inter-vivos trans lers. Thl'ii • ,tr· r •lt.•n l'd to ,,.., tntlu ,wns ,n gros.s
estate.

Types of Taxable Trans fers


1. Transfer in contempl,Hion of death
2. Revocable transfers, including rnnd1t1o nal cr,m ,;ft•rs
3. Property passrng under gcnernl power ot .tppo 1nt111t.•nc

Transfers in contemplation of death


These are donations 111.idc by the 1.kcedt.•n1 tlunng hi-. hl t.·ttrm· wh u:h are
m otivated by the thought of his de,Hh Tht.•s t.• 1r.111slt•r 111tt.•r-v1vth Jrt
usually made by th e decedent 111 a std~• ol t •rnun,ll tlln •ss o r undl•r bt-ht'I or
a n immine nt death. Analogous to te-;t,1111encary dhpthrtl on, rr,101,tt•rs 1n
contemplation of dedth are treated by the ta, la\\ as d o n.w o n morlb l ,1u5d
s ubject to estate tax not, to donor's t.L\.

Tra nsfer in conte mpla tion of death may include ·


a . Tra nsfers of property to rake elte cr in posses to n or nioymenc .1t or
after dea th

470

-----~
chapter 13-A-Estate Tax: Gross Estate

Transfer of property with retenf . .


b- eni·oyment or right over inc IOn of the nght of possess10n or
omeofthepro rty ·id h
Transfer of property with reten . P~ unt1 ~at .
c. conjunction with any pers h twn of th e nght to designate, alone or m
the income there from on, t e person who shall enjoy the property or

Revocable transfers
f h involve tran s iers o f possession over property during
vocable transfers
Re . .
the lifetime o t e . decedent, b u t not transfer of ownership over said
property.. At t h e pomt of death, th e d ece d ent owns the property· hence 1t·
ust
rn . be included
. as part of his . . ' '
gross estate smce the same 1s part of his .
donat10n mort1s causa.

In revocable transfers, ownership transfers only when the transferor waives


the right to rev_oke the transfer. If the transferor dies without waivi ng his
right of revocation, he owns the property at the point of his death. Hence, it
should be included in his gross estate.

Illustration 1: Revocable donation


In January 2017, Mr. Bala transferred a car with a fair value of P 1,200,000 to Mr.
Subas. The car shall be revocable by Mr. Bala until July 30, 2020. Mr. Bala died
on May 30, 2020 when the car had a fair value of P1,1 00,000.

The car shall be included in the gross estate of Mr. Bala at its fair value of
P1,100, 000.

The transfer shall be subject to donor's tax when the right to revoke e.Ypired prior
to Mr. Bola's death or when Mr. Bala waived the right to revoke before his death.
In this case, the property shall not be included in gross estate.

Illustration 2: Conditional donation


Mrs. Mulondo tra ns ferred a house and lot to his son, M,1siu. The tra ns fer of
ownership is cond itional upo n Masiu's p,1ssing of the October 2020 CPA Board
Exam. On June 15, 20 20, Mrs. Mulondo died.

The house and lot shall be included in the gross estate of Mrs. Mulondo at its fair
value on June 15, 2020 since she still owned the property at her death.
Assuming Masiu passed the October 2020 CPA Board Exa.m before Mrs. Mulondo's
death, the transfer shall be subject to donor's tax at the fa ir value of the property
on October 2020.
Assuming further that Mrs. Mulondo waived th~ condition before ~er death; the
~rans/er becomes O transfer inter-vivos which 1s subJe~t to donor s tax. In both
instances, the house and lot shall no longer be mcluded /fl the gross estate of Mrs.
Mulondo upon her death.
471
f
Chapter 13-A-Estate Tax: Gross Estate

Transfer with retention of certain rights


If properties are transferred by the decedent prior to his death but reta·
.
the possession or enjoyment of, or nght .
to mcome 'tis
fr om, th e property, the
same shall be included in gross estate to the extent of the decedent's
interest therein.

Illustration
Mr. Ozamis transferred an agricultural land in favor of his son. He, howev
reserved for himself the enjoyment of a quarter of the land until his death.
land was worth PZ,000,000.
T~:
The PS00,000 (i.e. P2,000,000 x ¼) portion of the land which was reserved by Mr
Ozamis for himself until his death shall be included in his gross estate. ·

Transfer under general power of appointment


Properties subject to a general power of appointment by the decedent shall
be included in the gross estate of the decedent The presence of the general
power enables the holder of s uch power to do with the property anything
which he could do as if the property were his own.

Illustration
Don Kulot died. In his will, he gave Mama Sang a house and lot with the right to
designate the property to whomever heir she wants. Mama Sang eventually died
and appointed Bebe as heir to the property.

Mama Sang had a general power over th e property. The same shall be inc/tided in
her gross estate. If Mama Sang had limited po wer, the same s hall not be included
in her gross estate.

COMPOSITION OF GROSS ESTATE


1. Properties, movable or immovable, tangible or intangible
2. Decedent's interest on properties
3. Proceeds of life insurance:
a. Designated as revocable to any heir
b. Designated to escate, adminis trator or executor as be ne fi ciary
4. Taxable transfers

PRESENTATOIN OF GROSS ESTATE IN THE ESTATE TAX RETURN


In reporting gross estate under BIR Form 1801, the compos ition o f the gross
estate s hall be classified as follows :
1. Real properties - all immovable prope rties of the decede nt excluding
family home '
2. Family home

472


ter 13-A-Estate Tax: Gross Estate
chaP
personal properties - all movable r .
3, . hts or interest in a ny bus· P operties of the decedent, except
ng . mess
Business interests
,f,

JNfEGRATIVE ILLUSTATIONS
rative Illustration 1
toteg d d' d .
Aresident dece ent ie w ith th e following properties at the point of death:
h in bank account p
i:~eivables from friends and relatives l,ggg;ggg
Borrowed car from a friend lZ0,000
House a nd lot . d. h 2,000,000
Motorcycle, reg1stere m t e name of his youngest son B0,000
Total P 4.400,000

The gross estate shall be computed as:


Inventory of present properties P 4,400,000
Less: Not owned
Borrowed car p 120,000
Motorcycle 80,000 200,000
Gross estate P 4,200,000

Integrative JIJustration 2
Mr. A, a citizen decedent, died leaving the following properties:
Cash proceeds of life ins urance designated to
a brother as revocable beneficiary P 1,000,000
Building, properties held as usufructuary 4,000,000
Cash in bank 2,400,000
Agricultural land 3,000,000
House and lot, from Mr. A's indus try 7,000,000
Benefits from GSIS 500,000
Total properties Pl 7,900.000

Additional information:
1. The agricultural land was designated by Mr. A's father in his will to be
transfer red to D, Mr. A's son, upon Mr. A's death.
Z. Mr. A made a revocable donation involving a residential lot to his brother E.
Mr. E paid p 400,000 when the lot was worth Pl,000,000. The lot was
currently valued at P2M zonal value upon Mr. A's death.
3· The heirs withdrew P376,000 cash from the decedent's bank account for
Mr. A's wake, net of 6% final tax deducted by th e bank.

The gross estate s hall be computed as:

473
Chapter 13-A-Estate Tax: Gross Estate
T (

Inventory of present properties P17,9oo,o00


Less:
Properties not owned
Building, held as usufructuary P 4,000,000
Agricultural la nd, under special power 3,000,000
Total P 7,000,000
Properties exempted by law
GSIS benefits 500,000
Bank withdrawal (P376,000/94%) 400,000 7,900,00..Q
Taxable present properties Pl0,000,000
Add: Taxable transfers (P2M - P400KJ 1,600,000
Gross estate Pl 1,600.0QQ

Integrative Illustration 3
An inventory of Mr. D's properties was taken tw~ years after his death. He had
the following properties during the inventory-taking:

Cash (40% from income of properties after death) P 4,000,000


Car (bought for Pl.2M a week before Mr. D's death) 800,000
House and lot (worth P8M on Mr. D's death) 10,000,000
Business interest (worth P6M on Mr. D's death) 7,000.000
Total P21.800.000

The following possible deductions can be claimed by the estate:


Funeral and judicial expenses paid P 1,100,000
Wreck of a fishing boat, one year after Mr. D's death 800,000
Obligations of Mr. D paid from his properties 1,500,000

The gross estate at the point of death shall be established as:

Inventory of property P21,800,000


Add: Decreases in properties since death
Funeral and judicial expense P 1,100,000
Wreck of a fishing boat 800,000
Obligations paid after death 1,500,000
Decrease in value of car {P1.2M - P.BM) 400.000 3.800,000
Total P25,600,000
Less: Increases in properties since death
Cash income of properties (P4M x 40%) p 1,600,000
Increase in value of house and lot
[Pl OM - PBMJ 2,000,000
Increase in business interest
{P7M - P6M) 1 O00.000
I 4.600.0QQ
Gross Estate r21.ooo.0QQ
474
13 _A-Estate Tax: Gross Estate
chapter
rlON OF THE GROSS ESTATE
y,AL(JA ieS su bject to estate tax s ha ll be appraised at their fair value at the
proper~ death. Conceptually, "fair value" refers to the amount at which two
O
poi~t .ndependent buyers and sellers could transact a n exchange.
willing i
tion rules
va1% e fair value of the property as of the time of death shall be the value
1- •nclude in gross estate.
tO I
F ir val ue rules set by law or revenue regulations must be followed.
2- ,: default of such fair value r ules, reference may be made to fair value
3· rules under generally accepted accounting principles.
Encumbrances on the property or decrease in value thereof after death
4
· shall be ignored.

The following sec~ions discuss fair value rules for the following assets:
Real properties
1
2' Shares ofstocks
3: usu/ruct and annuities
4. Other properties
s. Taxable transfers
Real properties
Under the NIRC, the appraisal value of real property shall be whichever is
higher of:
a. The value as determined by the Commissioner of lnterna l Revenue
[zonal value}, or
b. The value fixed by the Provincial or City Assessor.

If there is no zonal value, t he taxable base shall be the fair market value that
appears in the latest tax declaration. Note that the TRAlN law points to the
fair value listed in the schedule of market value - not the assessed value.

If there is an improvement, the value of the improvement s ha ll be the


construction cost per building permit or the fair value that appears in the
latest tax declaration.
Illustration
Mr~. Geogracia died leaving a house and lot as part of her estate. The property
which was encumbered by a Pl,000,000 mortgage had the following fair values:
-
r--... Lot Hous~
~ I value P 3,000,000
Fair v I
~ ue, per assessor 2,000,000 P 4,000,000
~ dvalue 400,000 1,600,000
~ val ue
sa] 2,500,000 4,500,000
475
Chapter 13-A- Estate Tax: Gross Estate
t
The house and the Jot shall be included in gros~ eS ~te at P4,ooo,o00
P3 000 000 . I The independent appra1sal is not used. A.I Qr,d
, , , respect1veiy. ,F h ty so, the
mortgage shall not be offset against the value 01 t e proper ·

Shares of stocks
The fair market value of stocks shall depe nd on wheth er the stocks
lis ted or unlisted in the stock exchanges. are

• Preferred shares are valued at par value .


Unlisted common shares shall be valued at their book value .

For this purpose, RR12-2018 reinstated ~he financial statement method
which ignores appraisal surplus. The Adjusted Net Asset Method under
RR6-2013 is no longer followed.
• For shares which are listed in the stock exchange, RR12-2018 al
reinstated the use of arithmetic mean of highest and lowest quotati;~
at a date nearest the date of death.

Illustration
Mr. Yakal died leaving 1,000 preferred shares and 300,000 common stocks of
MVC Company in his estate. The equity section of MVC in the its latest quarterly
financial statements is as follows:

Preferred stocks, 10,000 shares@ PSOO par P 5,000,000


Common shares, 10,000,000 shares@ P2 par 20,000,000
Share premium - common shares 4,000,000
Retained earnings 12,000,000
Revaluation surplus - PPE 2,000,000
Less: Treasury shares, 100,000 shares 360,000
Total shareholder's equity P 42.640.000
Case 1
Assuming MVC is non-listed company, the book value per share of common
share shall be computed as:

Common shares, 10,000,000 shares@ P2 par 20,000,000


Share premium - common shares 4,000,000
Retained earnings
12,000,000
Less: Treasury shares, 100,000 shares 360,000
Residual net assets
P 35,640,000
Divide by: Outstanding common shares
(10,000,000 - 100,000)
Book value per share 9,900,000
I? 3,60

476
chapter 13-A- Estate Tax: Gross Estate

r,,tr. Yakal's shares s ha ll be valu d


e asfoJlow . h'
s in is gross estate:
preferred stocks (1,000 shares@ PS
ordinary shares (300,000 x 3 _60) OO) p soo,ooo
roral investment 1.080,000
f 1.580.00Q
case 2
Assuming MVC is listed and is tr d d
. d a e as folio .
nearest trad mg ay to the date of death: ws m the stock market at the

MVC Company
8

6 _1 =o __ T --...,

+-· ·- - =l~-t
4

2
I
0

The ~verage price based on the foregoing graph shall be (4 + 6) / 2 = PS. Mr.
Yakal s shares shall be valued as follows in his gross estate:

Preferred stocks (1,000 shares @ PSOO) P 500,000


Ordinary shares (300,000 x 5.00) 1,500,000
Total investment P 2,000.000

Usufruct and annuities


A decedent may transfer usufructuary right to income over property or
right to receive amounts of annuities to his/her heirs. The fair value of such
usufruct or annuities must be included in gross estate.

To determine the value of the right to usufruct, use, or habitation, as well as


that of annuity, there shall be taken into account the probable life of the
beneficiary in accordance with the latest basic standard mortality table, to
be approved by the Secretary of Finance, upon recommendation of the
Insurance Commissioner.

Illustration 1: Annuity contract .. .


Mr. Mairugin, 60 years old, sold his company under a cond1t10n that the acquirer
shall pay his 35-year-old wife and their child P300,000 yearly support payment
for3Q years. Mr. Mairugin died after the fifth payment was made by the acquirer.

477
Chapter 13-A- Estate Tax: Gross EState
. discount rate is 12 % , the annuity sh
Assuming that the appropnat; M Mairugin at the present value of al! be
included in the gross e state O r. . d under the contract. the 25
remaining future payments to be receive

. • mathematically computed as follows:


The present value of an annmty IS

Value of annuity= (1 - (1 +i)·0 ) x Annuity payments

Where:
· d -- 12%
i = interest rate or discount rate per peno
.
n = number of periods = 30years - 5 years = 25 years
Value of annuity = (1 - (1 + 12%)·25) x P 300,000
12%

Value of annuity= 7.843139 x P300,000


= P 2,352,941.73

The annuity contract shall be included in the gross es tate of Mr. Mairugin at its
value of P 2,352,941.73.
Note: To compute for the annuity factor, press the following in your scientific calculator: [(1-
(1 + .12) XY - 25) -i- .12] =

Illustration 2: Usufruct
Don Midas transferred to Aurelius a nd h is he irs usufructuary right over a
PS0,000,000 prope rty fo r 10 years. After 10 years, the property s ha ll be given to
Marcus w ho was designated as t he owner of the naked titl e. Aurelius died just
after the e nd of the s ixth year of the usufruct

The property earns Pl,000,000 a nnu al income. Assume that the applicable
discount rate is 12%.

Upon the death of Aurelius, the us ufruct will be transfe rred to his heirs for the
four years (10 y rs. - 6 yrs.) re mai n ing un expired term of the usufruct.

The fair value of the usufruct to be included in the gross estate of Aurelius shall
be the present value of the a nnual inco me on the property for 4 yea rs:

Va lue of us ufruct = (1 - (1 + 12%)-4) x p 1,000,000


12%
Va lue of us ufruct = 3.037349 x Pl,000,000
= £ 3.037,349
Note:
1. Th e transfer of the property from Aureliu . . _. der of the
usufruct is not a merger of th f s to his heir for the remain
e usu ruct to the owner of the naked title.
478
ter 13-A-Estate Tax: Gross Estate
chaP
.45surning Don_ Midas g_ranted Aurelius' heir a li fe usufruct, the fair value of the
2-usufruct to be 1_n cluded_ in th e g ross estate of Aurelius shall be determined by taking
into considera twn the hfe expectancy of the heir.
The subsequent transf~r of the property from the heir upon his death to Marcus (i.e.
3· merger of the us ufruct m the owner of the naked title) is exempt from transfer tax.

other prop_e rties_ .


for propert1_es which the l_a w or re~enue regulations has not fixed va lua twn
Jes, valua ti on s ha ll take in to consideration fair value rul es under gen erally
:~cepted accounting principles (GAAP).

Additional Guidelines in Determining Fair Values


• For newly purchased proper ty, the fair value may be its purchase price. If
not newly acq uired, the fair value shall be its second-hand value.
• For pawned properties, the fair value may be r eestablished by grossing-up
the pawn value by the loan-to-value ratio.
• For property fixed in monetary terms s uch as a loan or receivable, th e fair
val ue is the amount fixed in the contract including accrued income ther eto.
• For foreign currencies, the fair value shall be its Peso valu e translated at the
prevailing exchange rate a t the date of death.

f)Justration 1: Used properties


Mr. Bantay died leaving a used car as part of his estate. Mr. Bantay bought the
car at P400,000. Brand new units of the same car model sel! at PS00,000. The
car sel!s at P250,000 if sold as is at the point of death.

The used car shall be included in the gross estate at P250,000.

IJJustration 2: Pawned jewelry


At the point of death, Mr. X has a piece of jewelry w hi ch was pawned with
Munting Pawns hop for P90,000. Munting Pawnshop mainta ins a 60% loan-to-
appraisal value.

The fair value shall be computed as ?90,000/60% = Pl 50,000. The Pl 50,000 fair
value shall be included in the gross estate. The P90,000 loan shall not be offset
with the value of the jewelry but should be presented as an item of deduction from
gross estate.

Illustration 3: Loans receivables


On June 30, 2020, Mr. Bombay d ied with the following outstanding receivables:
• P 50,000 non-interest bearing loan to Mr. A, given April 1, 2020 and is due
in 3 months
• P 300,00 0 loan, bearing 10% interest to Mrs. B, g iven January 1, 202 0 and is
due in one year
• AP 20,000 non-interest bearing loan to Mr. C, due March 3 0, 20 2 0 but still
unpaid

479
Chapter 13-A-Estate Tax: Gross Estate c:raf

~
. ded in gross estate as:
The foregoing loans shall b e me1u
p so,ooo
Loan to Mr. A Vi'~
300,000
Loan to Mrs. B f d th l·
Accrued interest on loan to Mrs. Bat th e date O ea 1s,ooo
Overdue loan to Mr. C 20.0Q_Q
p 385,0Q,Q
Amount to include in gross estate
Note:
1. Interest receivables accrued on the loan to Mrs. 8 from J!nuary l , 2020 to the da
death on June30 2020. This is computed as P 300,000 x 10 ¼i x 6 mo nths/ 12 rno te Of
2.
, , . · I <led in
Overdue claims even if due from insolvent persons, a re inc u
·
gross estate I
nths.
5.
taxation claim; proven to be worthless are still included in gross estate then s~ n estate
, ~~~ly 6.
presented as deductions.

1.
Taxable transfers
Taxa ble tra nsfers made without consideratio n a re included in gross estate
at the fair value of the transferred property at the date of dea th.

Taxable transfers made for a considera ti on are va lue d as: Fair value at th
date ofdeath less consideration paid at the date of transfer. e
2.
Hlustration
3.
Before her death, Mrs. Power made th e fol lD\1,/lflg m o rtis cau a tra nsfers dunn
her lifetime: g
4.
At the date of Lr<111~frr_ Fair va lue
Fair value Coo:i1tl~llliW at clcJ.t.h_ s. '
To Alexander P 300,000 P 0 P L00,000
To Bee Jay 200,000 195,000 300,000 6, I
To Cedric 100,000 4 0,000 120,000
To Donnie 150,000 8 0,000 70,000
7.

The a mount of taxable transfers to be included in gross ~st.H1.' ~ha ll be:


8.
EY at death 9.
To Alexander
Coosider,ttio.a
P 200,000 - P 0 P 200,000
To Bee Jay 10.
To Cedri c 120,000 1 1.
To Donnie 40,000 80,000
70,000 80,000 12
13.
Note: 18WQO
1. The tra nsfer to Bee Jay is for a n ad . t
s ubject to estate tax but to i ' equate con s ideration This is bon.1 fide sJJes no
ncome tax at the da te 1 1
2. The tra ns fer to Donnie decreased 1 1
is simply igno red.
° ~e ·i r
n atr value bt.'low tht• rons1derntion. This cram t

480
chapter 13-A-Estate Tax: Gr
oss Estate
cJIAPTER 13-A: SELF-TEST
,,,....- EXERCISES
oiscussio n Questions
. .
1· What is gross estate'>· 0 tstin •
decedent taxpayer guish the ext
. . · ent of g
z. Discuss and illustrate the co . ross estate of each type of
3. Enumerate the list of proprnputat1ona1proccdu
decedent. erty transfers h' res of gross estate
W 1ch ar ·
4 Enumerate the list of p e not owned by the
. b roperty tr c
gross estate ut which ar ans,ers which
5 Enumerate the list of ta ebelxcluded by law are properly includible in
. xa e transi .
6. Discuss the treatment of m . ers.
consideration. ortis causa transfer mad ~ .
7 Discuss the valu ti e or insufficient
. a on rules for real
and other properties. property, stocks, usufruct and annuity

True or False 1
1. The list of properties of the dee d . .
1st
established in determining gros: e:~\:~ mg at the point of death must be
2. Gross estate consists of all real pro pert·tes and tangible I •
wherever situated, at the point of death. persona properties
3. The gross estate of a non-resident citi · I d
wherever situated. zen me u es tangible properties
4. The gross estate of a non-resident alien includes only tangible t·
s1·t ua t e d m
. th e Ph·1· .
1 1ppmes. proper ies
5. The gross estate of a resident alien includes only real properties wherever
situated.
6. The list of properties of the decedent must be counted at the date of death.
7. Properties owned by the decedent which are exempted by law are initially
included in gross estate, but are removed by way of deduction from gross
estate.
8. Properties not owned by the decedent may be included in gross estate.
9. Taxable transfers consist of properties passed to other persons before
death but are still owned by the decedent at the point of death.
10. The motives of the donation shall be the basis of its taxation .. . .
. 1 d t the higher of the fair value and acqms1t1on cost.
11 . Propert1es are va ue a ...
12 N • · valued at acquis1t10n cost.
. ewly-acqmred properties are .b for fair value rules fair value
13. Where the law or regulations do no_t pre;i:~i eles may be sought. '
rules of generally accepted accounting P P
th
14. Listed stocks are valued at eir par ~al~;heir book value.
lS. Non-listed common stocks are value a

481
Chapter 13-A- Estate Tax: Gross Estate

True or False 2 . de before death are included in gross est


1 Transfers inter-v1vos ma . t t ate.
. . d th is included in gross es a e.
2. Income earned after ea h . ·ncluded in gross estate.
3 Income earned before deat 15 1 d .
· f bl. tions and expenses after eath 1s added ba k
4. The payment or o iga c to th
amount of gross estate. e
5 Taxable transfers are included in gross estate. .
· . Id b the decedent as a trustee must be included i
6. Properties 11e Y n gr055
estate. . h t b t ·tt d
. h Id by the decedent wh1c mus e ransm1 e to an h .
7. Propert1es e d d f e1r i
d •th the desire of a predecessor are excIu e rom gross e n
accor ance w1 . . . . d d. State
8. The proceeds of an irrevocable hfe msur~nc~ is 111~ 1~ e •~ gri°ss estate. ·
9. Properties held by the decedent as a fl uc1ary eir are me uded in gross
estate.
lO. The proceeds of life insurance which is revocably designated must be
included in gross estate. . .
11. The proceeds of life insurance is included m gross estate if the beneficiary is
the estate, the executor or administrator.
12. The separate properties of the surviving spouse are excluded from gross
estate.
13. Transfers made for adequate considerations are excluded from gross estate.
14. The merger of the usufruct in the owner of the naked title is a transfer
excluded from gross estate.
15. The gross estate includes only the separate properties of the decedent.
16. The proceeds of group insurance is included in gross estate.
17. The proceeds of GSIS policy and SSS benefits are included in gross estate.
18. Benefits from the USVA are exempt from estate tax.
19. Donations mortis causa to accredited non-profit institutions are exclusions
in gross estate.
20. Transfers in contemplation of death are included in gross estate.
21. If the fair value of the property transferred mortis causa for an inadequate
consideration declines below the amount of the consideration at the date of
death, no amount is included in gross estate.
22. Revocable transfers and conditional tra nsfers are included in gross estate at
the fair value of the property at the date of their transfer to the transferees.
23. Properties passing under special power of appointment are included in
gross estate.
24. The gross estate is valued at the point of death.
25. If a consideration is paid by the transferee in a taxable transfer, the amount
to include in gross estate shall be the excess of the fair value of the property
at the date of death over the consideration given.

482
' chaP
~1.1lf
ter 13-A- Estate Tax: Gross Estate

tple Choice - Theory: Part 1

The gross <'S l at e of res ident or citizen de ,


t. prope,·tles located abroad cedcnts does not Include
a.
b. properties located in the Phi lippines
c. Jntangi~le personal property located abroad
d. Properties not owned

Gro estate means


2· Properties, whether or not o db
a. wne Y the decedent existing at the point
of death '
b. Properties owne_d b~ the decedent at the point of dea th
c. Present properties m the possession of the decedent at the point of
death
d. Any of these

l. As a rule, all deced ents are taxable on world estate, except


a. non-resident alien.
b. non-resident citizen.
c. resident alien.
d. non-res idents.

4. As a rule, the gross estate of non-resident alien decedents includes


a. Intangible personal properties located abroad
b. Real properties located abroad
c. Intangible personal properties located in the Philippines
d. Tangible personal properties located abroad

5. Which has reciprocity exemption?


a Resident alien
b. Non-resident alien
c. Non-resident citizen
d. All of these
. . . ?
6. Which property is covered by the rec1proc1ty exemption.
a Intangible personal property abroad
b. Intangible personal property in the Philippines
c. Tangible personal property abroad . . .
d. Tangible personal property in the Ph1hppmes

· h o f th ese propert·1es may be excluded from gross estate by reason of


7· Wh 1c
reciprocity?
a. Cash c. Paintings
b. Car d.Land

483
Chapt'ef 13-A-Est te T . Gross Esta~
fr th Inventory: llst of properttes in ,
8 Whl \ not deduct d om a~l\g
the .F () to? Id trUSt ' :&&f.t
Propcrtits h a fiduciary helr 4
"t
h Propertt held der eeneral power of appointment t 1i:
Propcrtt held und speclal power of appointment 41'
d Properti held un er a
f . operties ls conducted after the death of
lf ln ntory- kingdedo ctpr d from the inventory list?
decedent. which Is u e
. lncomc ecru Ing after death
b. Expen pald after death
Income accruing before death
d. Expenses paid before death

10. Which is not added to the inventory list of properties in computing groSc
estate7
a. Transfer in contemplation of death
b. Revocable transfers .
c. Transfers under general power of appointment .
d. Merger of the usufruct in the owner of the naked title

11. Which is a correct statement?


a. The merger of the usufruct in the owner of the naked title is a
transmission subject to estate tax.
b . The transfer from the first heir to the second heir in accordance with
the wishes of a predecessor is a taxable transfer s ubject to estate tax.
c. The proceeds of life insurance taken by the decedent for
himself/herself is always included in gross estate if the beneficiary is
the estate, executor or administrator.
d. The truster need not include a property transferred in a revocable trust
in his gross estate.

12. Which of these transfers is subject to estate tax?


a. Transfer of properties for a full consideration
b. Transfer of the inheritance from the fiduciary heir to the
fideicommissary
c. Transfer of the property held in trust by a trustee to the beneficiary
d. Transfer mortis causa for less than full and adequate consideration

13. Which of these properties is subject to estate t ?


P t" h. h ax. re
a. roper ies w ic were initially transferred in revocable trusts but we
revoked by the decedent before his/her death .
b. Pro perties transferred under revocable trust which the decedent failed
to revoke at the time of his /her death

484
.A- Estate Tax Grose; Es c1t
er 13
cr~r
prnprrtic~ tran, frrt d und,,,- co ndltlnn,11 rlnn ,1tion<; wherein th" doncc
c h-1' not rt fulfill d the tc1 me; of do,utton r1t th cl,ll of cl Jth of th e
(irrl'drnt
All of thr"
d
crrds of life lnsura nc dcslgnat d hy th cl ccd rnt to h ic./ h c r chi ld
4 "hr
' P' Olied 111 Rro c; cc;tat c
1 inc1\I
,~ if the de,;1gn<H1 on 1s I vocabl e.
1 f th<' de'-1gna t1 on 1s 1rrevoc.1hle.
b I .
~ ithout regard to the d cs1gn c1t1 on as rcvocabl o r lrrcvoccible.
< 10 all circum lances.
d.
fhe proceeds of life insurance designated by the decedent to his/her estate
15 eluded in gross estate
1• 111
if the des1•gnat1on
. 1•s revoca bl e.
:. ,f the designation is irrev~cabl~.
C.
without regard to the des1gnat1on as revocable or irrevocable.
d. in all circumsta nces.

The proceeds of life insurance designated by the decedent to his wife is


16.
excluded in gross estate
a. if the designation is revocable.
b. if the designation is irrevocable.
c. without regard to the designation as revocable or irrevocable.
d. under no circumstances.

17. The proceeds of life insurance designated by the decedent to his/her


executor is excluded in gross estate
a if the designation is revocable.
b. if the designation is irrevocable.
c. without regard to the designation as revocable or irrevocable.
d. under no circumstances.

18. Which is an inclusion in gross estate?


a Separate properties of the surviving spouse
b. Common prope rties of the spouses
c. Separate properties of the heirs
d. Properties acquired from group insurance
19
· Which is not included in gross estate?
a Revocable transfe rs
b. Transfers in contemplation of death
c. Transfer under special power of appointment
d. All of these

485
T . Gross Estate
Chapter 13-A- Est ate ax.
estate of the husband?
d d in the gross
20. Which is not inclu ~
a. Capital propertie~
b. Common propertte\es
c. Paraphernal proper I
d. All of these
. ross estate of the wife?
21. Which is not included m the g
a. Capital properties
b. Common properties
c. Paraphernal properties
d. None of these
. donations in the last will and testament is exc\uct
22. Which of the following ed
7
in gross estate of the decede~t.
a. Donation to a favorite child
b. Donation to an ex-girlfriend .
c. Donation to a charitable institution
d. Donation to the government

23. Which proceeds of insurance is included in gross ~state of the decedent?


a. Proceeds of life insurance irrevocably designated to the decedent's
child
b. Proceeds of group insurance
c. Proceeds of property insurance
d. All of these

24. If the decedent failed to specify the designation of the beneficiary of his life
insurance policy
a. The proceeds is exempt if he did not changed the beneficiary during his
lifetime.
b. The proceeds is taxable whether or not he changed the beneficiary
during his lifetime.
c. The proceeds is exempt because it is cons idered a revocable
designation.
d. The proceeds is taxable because it is considered a revocable
designation.

25. Which is not an exclusion in gross estate?


a. Accruals from SSS
b. GSIS benefits
c. War damage payments
d. Private retirement benefits

486


_A-Estate Tax: Gross Estate
ter 13
cnaP
. h of the following bequests to
w111c t t t 7 . a social Welf
z6-. subject to es a e ax . are or charitable institution
15 Bequests to be used for a dministrat'
ll· gequests restricted by the deced;e purposes
b- institution nt for program expenses of the
Bequests to accredited non-profit . . .
c. All of these tnS t itut1ons
d.
tement 1: For taxable transfers the 1
27- 5t0the fair value of the property at' the v~ ue to include in gross estate shall
be point of death
swtement 2: For taxable transfers mad i . · .
cl e total fair value of the property at th e o_r an msuffic1ent consideration,
1
estate e pomt of death shall be included in
gross ·

Which is correct?
a. Statement 1 c. Both statements
b. Statement 2 d. Neither statement

Multiple Choice - Theory: Part 2

1. Mr. A devised . in his w ill a piece of land to Mrs . 8 . Mrs. 8 sh aII enJoy
·
usufructuary nght over the property and shall pass the same to Mr. c upon
her death.

Who shall include the property in his or her gross estate upon death?
a. Mr. A c. Mr. C
b. Mrs. B d. Mr. A and Mr. C

2. Mrs. A appointed Ms. B as fiduciary heir over an agricultural land which Ms.
Bshall turn over to Mr. C upon Ms. B's death.

Which is incorrect?
a. The land must be included in Mrs. A's gross estate upon her death.
b. The land must be excluded in Ms. B's gross estate upon her death.
c. The land must be excluded in Mr. C's gross estate upon his death.
d. None of these

3. Mr. A designated his wife as the revocable beneficiary of the proceeds of his
life insurance. Which is correct?
a. The proceeds must be included in the gross estate of Mr. A
b. The proceeds must be excluded in the gross estate of Mrs. A upon her
death.
c. The proceeds must be treated as donation subject to donor's tax.
d. The proceeds must be excluded in the gross eSt ate of Mr. A.

487
Chapter 13-A-Estate Tax: Gross Estate
ts.·. ,t.;,
<

4. , Mr. A designated Mr. K, the ex(?cutor of his estate, as his il't' .


beneficiary to the proceeds of his life insurance. ,evocab1, '
~ 11Jf{
Which is correct? . . '
a. The proceeds of the life insurance pohcy shall be mcluded in the ~
estate of Mr. A. 0ss
b. The proceeds of the life insurance policy shall be excluded in the gro
estate of Mr. A. Ss
c. The proceeds of life insurance is a donati?n subject to donor's tax.
d. The proceeds is exempt from both donors tax and estate tax.

5. Which of the following is excluded in the gross estate of Mr. X?


a. Separate property of Mr. X
b. Common property of Mr. and Mrs. X
c. Separate property of Mrs. X
d. Band C

6. Mr. A made an irrevocable donation in trust in favor of Mr. C. Mr. C died tw


0
years after receiving the donation.

Which is correct?
a. The property shall be included in the gross estate of Mr. C.
b. The property shall be included in the gross estate of Mr. A
c. The property shall be excluded in the gross estate of Mr. C.
d. None of these

7. What is a best way to minimize estate tax exposure?


a. Invest your money in a corporation
b. Donate the properties to relatives before death 14. '
c. Understate your gross estate and do not pay estate tax
d. Invest property in life insurance and make the designation of the
beneficiary as irrevocable

8. Which of these transfer mortis causa will more likely to be included in ross
estate of the decedent? g

Fair value at transfer Selling price at transfer


a. P 100,000 Fair value at death
P 150,000 P 200,000
b. P 150,000
P 150,000 P 300,000
C. P 200,000
P 150,000 P 140,000 15, ,
d. P 300,000
P 200,000 P 280,000
9. Which is excluded in gross estate?
a. Fringe benefits c. GSIS b fi
b D · • . b ene its
. e mm1m1s enefits d. PhilHealth benefits

488
~ r 13-A-Estate Tax: Gross Estate
cnaPte
·ch of the following propertie .
10· wiltresent at t h e point
.
O
f d eath? s IS exclud d fr
e om gross estate even if
P Revocable transfer c. Transfer
,:i. conditional transfer d Pr d by bona fide sale
b, . ocee s of group insurance

wnich may be exe~p~ from estate tax?


11• proceeds of building fire insuranc
b proceed s o f car insurance
,:i. . e
· proceeds of crop insurance
~-- proceeds of life insurance

Which is not included in gross estate?


12· Decedent interest to properties
a.
b. Taxable transfers
c. proceeds fof life insurance
. irrevocably desi·gnat e d to t h e estate
d. Income o properties of the decedent after death

_The decedent owns an agricultural land with the following values:


13
zonal value P 2,000,000
Assessed value 1,500,000
Independent appraised value 3,200,000
offer price of a willing buyer 4,000,000

The agricultural land shall be included in gross estate at


a. P 1,500,000 c. P 3,200,000
b. P 2,000,000 d. P 4,000,000

14. The decedent owns 200,000 shares of Saint Peter Corporation, a listed
company.
Date of death Date of interment
P 23.50 P 23.55
Book value per share P 49.50
P 48.20
Closing price P 49.60
Average trading price
P 48.00

The 200,000 shares shall be included in gross estate at


a. P 4,700,000 C. p 9,600,000
b. P 9,640,000 d. P 9,900,000

15. The decedent had $2 000 in his possession at his death on November 2,
1019. He was buried on November 12, 2019. The following were the
exchange rates:
P 42.50: $1
November 2, 2019 P 43.25: $1
November 2 to November 12, 2019 average rate P 42.75: $1
November 12, 2019
489

---------■ , -
Chapter 13-A-Estate Tax: Gross Estate

At what amount shall the $2,000 be in cluded in ilie gross estate of


'
decedent? th~
a. P 86,500 c. P 85,000
b. P 85,500 d. P 0

Multiple-Choice - Problems: Part 1


1. Mrs. Dely Cado died on November 1, 2019. An inventory of her prope .
was conducted for estate tax purposes on January 1, 2020. On that dat rties
had properties with an aggregate fair value of P7,000,000. This a e, she
includes P300,000 income received by the estate since her death and~Unt
of P600,000 expenses used during her funeral. net

What is the amount of gross estate? 6, J


a. P 7,900,000 c. P 7,000,000
b. P 7,300,000 d. P 6,700,000

2. Mr. Tio died leaving the following properties:


Purchase cost Fair value
Car, registered in his name P 800,000 P 400,000
Family home 2,000,000 5,000,000
Other properties 400,000 350,000
Compute the gross estate.
a. P 2,750,000 C. p 5,750,000
b. P 3,200,000 d. P 6,200,000
7. '
3. ~r. Bacleit~ had the following properties with their respective fair value5 ·
his possession at the date of his death: m

Agricultural land, held in trust p 200,000


Car, registered in the name of his brother 300,000
Motorcycle 80,000
Residential lot 900,000
Other personal properties 70,000
Compute the gross estate.
a. P 950,000 C. p 1,250,000
b. P 1,050,000 d. P 1,550,000
8,
4. The heirs of Mr. Masigasig identified .
existing at the date of his d th A a total P12,000,000 in properties
wake up to his intermenteaH_. t~tal of PS00,000 was used during his
obligations of Mr. Masipag. · IS heirs also used P400,000 to pay off

A_n . investment in the estate of . .


dividends which the heir Mr. Mas1pag also earned PSO 000 in
s reserved for payment of his estate tax. '
490
T 13 _A- Estate Tax: Gross Estate
cnapter
J1at is the gross estate?
W p 12,900,000 c. P12,SOO,OOO
~: p 12,850,000 d. P12,000,000

Adecedent died in Octo~er 20~8. His properties had an aggregate fair value
5. of p1 2,ooo,~OO at that tim ~. His heirs failed to pay his estate tax. In March
2020, his heirs prepared a hst of the decedent's properties which now had a
vatue of P13,500,000 and which excluded a car worth Pl,500,000 stolen in
January 2020. .
What is the gross estate?
a. p 0 c. P13,SOO,OOO
b. p 12,000,000 d. PlS,000,000

6. A resident decedent died leaving the following properties:


An orchard, held as usufructuary P 1,200,000
Amotorcycle, borrowed from a friend 80,000
cash, proceeds of his bank loan 500,000
Aranch, acquired for Pl,000,000 in 1990 2,500,000
Catties 600,000
House and lot 800,000
Compute the gross estate.
a. P 2,400,000 C. P 4,400,000
b. P 3,900,000 d. P 5,680,000
7. A citizen decedent died leaving the following:

Properties, inherited from his father P 1,200,000


Properties, donated by brother 800,000
Cash, from his salary savings 400,000
Cash, income of properties before death 200,000
Receivables, income of properties after death 100,000

What is the gross estate?


a. P 600,000 C. p 2,600,000
b. P 700,000 d. P 2,700,000

8
· A resident Japanese decedent died w ith the following properties:
A house and lot in Japan 2,000,000
Bank deposit, in the Philippines 800,000
A car in the Philippines 1,000,000
A residential lot in the USA 1,500,000

491
Ch pt r 13-A- Estata Tax: Groll Et

Compute the amount to b fndudod In ffJ'Off fft.ltol,..-411


, p0 C, P 2,000,00~
b. P 1,800,000 d, P 5,300,00

9, A non-resfdcnt aJfen decedent dlod Je.wfnB th


A buildIng In Korea 11 s,000,000
A car In Hongkong 2,DOO,ooo
Sh ar es of stocks In Malaysia 4,000,000
Cash In Phlllpplne banks 800,000
Investment In bonds of domestic corpor tJow, 400,000

Compute the gross estate.


a. PO c. P 1,200,000
b. P 800,000 d. P 11,200,000

10. A Korean cldzen residing In the Phllfppfnc1 died In th e Phfllppl


An agricultural land In Kor ea Pl 2,000,000
A house and lot In Korea 4,000,000
A condom i nium unit In the Phillppln<:s 2,000,000
A business interest In the Phlllpp1n J.000.000
Car In the Phil ippines l.200,000
Cash In Japanese banks 1,800,000

What Is the amount lo tnclud In r,ro J


a. P O , f' h.2UO,OUO
b. P3,200,000 d P 2-1.000,000

11. A Fil ipi no citizen died while r c 1d111g In 1f11· US fie h d -h


properlics at the date of his dl',Hh:
Shares of domestic cor por.itions r 2,000,000
Cash in US banks 5,000.000
Car in th e US
.000.000
House and lot in the Ph1l1pp1n •s
,000,000 1
Whal is the ,1mount tu rndud c in •!ros«; t' l ll .
a. p O t . f' h,000 000
b. p 4,000,000 d. I' I 1,ou·u,000
12. A Mextt,111 c1t11.cn dtl'd 111 T 0 k .0 J
· > • .ip 111 Heh 1d th • folio\\ tng prop rt
llou-;e and lot in Mcxirn
llou sc .ind lot in f.ip,tn
Pll 000 000
C.11 111 J.ipan 18 000 000
Sh,ll l's o .f stotk 111 ,1 do111t·s11
Z 000 000
1 1 0 1 po r 1uu 11 I 000 000
l ntL·r cst 111 ,1 Phdtppint• t, t:, ·d hu t.·
II
'00 000
ter 13-A- Estate Tax: Gross Estate
cnaP
Wh at is the amount to include in gro ss estate?
a. p 0 C. p 26,000,QQQ.
b. p 6,000,000 d. p 38,000,000

In the immediately preceding problem wh . . .


13· . 5 estate assuming that th . .' at is the amount to mclude m
g, os 0 e rec1proc1ty condition applies?
a. p C. p 26,000 000
b. p 6,ooo,ooo d. p 3B,ooo:ooo
14. A non-resident alien decedent had th e 1, 0 IIowmg
• •
interests at the point of
death:

Interest in a business partnership organized abroad P 300,000


Shares in a foreign corporation 75% of the business
of which is situated in the Philippines 400,000
Shares of a foreign corporation traded in the
Philippine Stock Exchange 800,000
Claims from resident debtors 1,200,000

Compute the amount of properties considered situated in the Philippines.


a. p 800,000 C. p 2,000,000
b. Pl,200,000 d. P 2,400,000

15. Mr. Masipag died leaving a commercial land as part of his estate. The land
was purchased in 2018 at an acquisition price of Pll,000,000. An
interested buyer tendered a PlS,000,000-offer to buy the property. The
land had an independent appraisal of P16,000,000, assessed value of
P12,000,000 and a zonal value of Pl 4,000,000.

What is the amount to be included in gross estate?


a. P 11,000,000 c. P 14,000,000
b. P 12,000,000 d. P 15,000,000

16. The following relates to the withdrawals from the account of a decedent
who died January 8, 2019:

January 7, 2019 P 400,000


January 9, 2019 500,000
500,000
January 10, 2020
Assuming that the bank was duly notified of his death, compute the
required exclusion in gross estate.
c. P 900,000
a. p 400,000 d. Pl,000,000
b. P 500,000

493
Chapter 13-A-Estate Tax: Gross Estate

17. Mr. Imbestor died with significant stock holdings as follow.s:


~
• 1,000 Globe preferred stocks with Pl'.000 par value per sh;re
• 40,000 San Miguel common shares with par value of Pl oo p
• 80 000 stocks of Carmen Corporation. w1'th par value of PSo er shate
' Per share
PSE closing Price
Book values at date of death
P 1,000 P 1,000
San Mi uel common shares 250 300
Carmen common shar es 45

What is the total amount to include in gross estate?


a. p 13,600,000 C. p 16,600,000
b. P 15,000,000 d. P 17,000,000

18. A decedent died with the following properties:

Purchase cost
Newly purchased jeepney P 1,200,000
An old model Ford Expedition 1,500,000
Land, mortgaged to a bank for Pl,000,000 3,000,000
500 grams pure gold 850,000
The Ford Expedition was refurbished by the decedent making it read!~
saleable at Pl,800,000 in the second-hand market. The land had a zon~
value of P4,000,000 and an assessed val ue of P2,500,000. The Bangko
Sentral ng Pilipinas (BSP) was buying gold at Pl,800 per gram at the date of Mull
death of the decedent. 1.
Compute the total amount to be included in gross estate.
a. p 6,550,000 C. p 7,150,000
b. P 6,900,000 d. P 7,900,000

19. Mr. Margarito died while serving as the m<lllc1ging partner in a business
partnership. He owned 40% of the capital and profits of the partnership.
The partnership had a total capitalization of P 10,000,000, exclusive of
accrued profits of Pl,000,000. Partnership income was su bject to 30%
corporate income tax.

What is the value of the business interest which must be included in lhe 2·
gross estate of Mr. Margarito?
a. P 280,000 c. P 4,280,000
b. P 400,000 d. P 4,400,000

20. A decedent owns 25,000 stocks in a closely-held corporation which had cht
following equity structure at the date of de.1th of the decedent:

494
""" 13.11-Estate Tax: Gross Estate
c~apter

mon stocks (1,000,000 outstanding shar ) P 5,000,000


co:•rional paid-in capital es 2,000,000
Ad t·11ed earnings 1,000,000
Retai , .
1
shareholder s equity P 8.000.000
rota
What is the value of the stocks to be reported in gross estate?
pQ C. p 175,000
~. p 125,000 d. P 200,000

Mr· canuto was hospi~alized on April 8, 2020 and died on April 12, 2020. He
21· was buried_ on Apnl 21 , 2020. Mr. Canuto had $124,000 prior to
hospitalization_. $ 24 ,000 was spent for his hospitalization while $10,000
was used for his funeral.

The following were the exchange rates between the peso and the dollar:

-
peso-Dollar
Aoril 8, 2020 Aoril 12, 2020 April 21,2020

exchange rate P 42.30 P 42.50 P 42.45

Compute the amount to be included in gross estate.


a. P 4,245,000 c. P 5,270,000
b. P 4,250,000 d. P 5,695,000

Multiple-Choice - Problems: Part 2


1. Ms. Ma. Katie Pero died bankrupt. She had the following properties and
obligations at the date of her death:

Cash P 200,000
Agricultural land 3,000,000
Family home 2,000,000
Debts and obligations 8,000,000

Compute the gross estate.


a. (P2,800,000) c. P 5,000,000
b. PO d. P 5,200,000

2· Mrs. Candida died. In his last will and testament, he indicated the following:
P 5,000,000
House and lot, to his adopted son and only heir
1,000,000
Land, to a public school 500,000
Cash, to ABS-CBN Foundation
The legacy to ABS-CBN Foundation was intended for Bantay Bata 163, a
non-profit social welfare program of ABS-CBN Foundation.
495
T
Chapter 13-A- Estate Tax: Gross ESt ate

Compute the gross estate. c. P6,000,000


c. P s.000.000 d. P6,S00,000
d. P 5,500,000
3. The heirs of a decedent received the following ins urance proceeds Upon
th
decedent's death: e

Source . t d to wife as revocab I e b ene r·1c1ary


· ~ P 400.000

Insurance A - d es1gna e
.
. r·
t d to son as Irrevocable bene 1c1ary
·
Insurance 8 - d es1gna e soo,ooo
Insurance C - group insurance proceeds . . 200,000
. ted to executor as Irrevocable beneficiary 400,000
Insurance D - d es1gna

Compute the insurance proceeds to be included in gross estate.


a. P 200,000 c. P 800,000
b. P 400,000 d. P 900,000

4. In his will, Anton transferred a life usufruct in favor of ~endong and 8.


thereafter to Bentong,_ wh? is the owner of the naked title. Cendong
eventually died resultmg m the transfer of the property to Bentong.
Bentong also died a few years later.

Which is correct?
a. The property shall not be included in the gross estate of Anton.
b. The property shall be included in the gross estate of Cendong.
c. The property shall not be included in the gross estate of Bentong.
d. The property shall be included in the gross estate of Anton and
Bentong.
9.
5. D devised in his will an agricultural land with life usufru ct to E and naked
title to F. All parties eventually died. The land had a value of Pl,500,000 on
D's death, Pl,000,000 on E's death and P2,000,000 on F's death.

Which is correct?
a. The land shall be valued at Pl,000,000 in the gross estate of D.
b. The land shall be valued at Pl,500,000 in the gross estate of E.
c. The land shall be valued at P2,000,000 in the gross estate of F.
d. None of these ·

6. Mr. X d~vise~ in his will a commercial land to be given to z, his favorite


gr~ndch1ld. Smee Z was a minor, Mr. x designated Mrs. y as the fiducial'}'
heir ~ho was entrusted the obligation to transfer the property to Z upon
Mrs. Y s death. ·

496
ter 13-A- Estate Tax: Gross Estate
o,aP
Which is correct?
The transfer of the property f .
a. not to esta te tax. rom Mr. X shall be subject to donor's tax
Th e tra nsfer fro m X to z is s ub · '
b. , Ject to estate t
c. The tra ns,c rs from X a nd y are not s u . • ax, not to donor's tax.
The transfer from Y to z is sub· bJect to a ny transfer tax.
d. Ject to donor's tax.
In his will, Mr. Curandang appointed Matit .
7· Matito, however, was bound to tra , 0
as first heir over a business.
h . ·11 M . ns,er the property t
Donna. In 1s w1 , atito designated Donn . .
'th El
o e1 er sa or
a as benef1c1ary of the business.
Which is true?
a. The transfer from Mr Curandang to M ti .
The transfer from Ma~ito t 0 D . a ~o Is subject to estate tax.
b. onna 1s sub1ect to estate tax
c. The transfer from Mr. Curandang to Matit . b" ·
None of these o is su Ject to donor's tax.
d.

8. Mr. Ca?ayo is th e _chief executive officer of Payaman Power Multilevel


Marketmg Corporat10n (PMMC). Mr. Cabayo died in a car crash.

Which must be in~lu~ed in the gross estate of Mr. Caba yo?


a. Proceeds of life msurance taken by Mr. Cabayo with his eldest son as
irrevocable beneficiary
b. Proceeds from Mr. Cabayo's car insurance
c. Proceeds of the life insurance over Mr. Cabayo's life taken by PMMC
with PMMC as the beneficiary
d. None of these

9. Mrs. Malou Phet is the be neficiary of a revocable donation consisting of two


commercial buildings. Mrs. Phet herself was also designated by her father
as a fiduciary heir over a commercial lot which was to be transferred to her
younger sister, Mabaet. Mrs. Phet died in action as a military servicewoman.

Which is correct?
a. The buildings and the commercial lots shall be included in the gross
estate of Mrs. Malou Phet.
b. The two buildings shall be included in the gross estate of Mrs. Malou
Phet.
c. Only the commercial lot shall be included in the gross estate of Mrs.
Malou Phet. ·
d. The buiJdings and the commercial lot shall be excluded m the gross
estate of Mrs. Malou Phet.

497
Chapter 13-A- Estate Tax: Gross Estate

10. Mr. Dino Jones di ed leaving the following properties:

Car, purchased us ing GS IS retire me nt bene fits p soo,ooo


lnt('rest in a joint venture bus iness . . 3,000,000
House a nd lot, separate prope rty of his wife . 2,000,000
Proceeds of insurance, revocably designa ted to wife ~
Total ~
Compute the amount to be included in gross estate.
a. p 2,000,000 c. P 4,500,000
b. P 3,500,000 d. P 7,000,000

11. An inventory total of the properties of Mrs. Tina Taray and her Widoweri
shown below: s
Separate property of Mrs. Taray P 6,000,00Q
Separ ate properties of Mr. Taray 1,500,000
Value of family business 2,000,000
Other properties owned jointly by Mr. and Mrs. Taray 3,000,000
Total P12,S00,00Jl
Compute the gross estate of Mrs. Taray.
a. P 4,000,000 c. P 6,500,000
b. P 6,00,000 d. P 11,000,000

12. On September 30, 2020, Mr. Pogi Nalang died in an accident. A few hours
later, his wife, Maganda Nalang, died of heart-attack upon learning of Mr.
Pogi's death. The spouses had a son who is the only heir to their properties.

The properties of the spouses on September 30 were:


Separate properties of Pogi P 3,000,000
Separate properties of Maganda 5,000,000
Common properties of the spouses 8,000,000
Compute Matikas' gross estate.
a. p 3,000,000 C. P 11,000,000
b. P 7,000,000 d. P 16,000,000

13. In the immediately preceding problem, assume further that Mrs. Maganda
Nalang had the following:

!n
Share the net dist:ibutive estate of Mr. Nalang p 2,soo,000
Share m the net conJugal properties with Mr. Nalang 3,700,000

498


T cnaPte
r 13-A-Estate Tax: Gross Et
s ate
npute Mrs. Nalang's gross
car 5 ooo 000 estate.
P , ' c Pa 7
a. p 7 500,000 · , oo,ooo
b- I d. p 11,200,000
,, Willy made the following tra ,
4 ivir• . ns,ers morr
1. ' Transfer to Jaimee - A revocable t is causa during his lifetime:
valued at_ P2_,000,000 at the date of ;ansfer involving a residential lot
same until his death. ransfer. Willy failed to revoke the
Transfer to Mr. Li - A conditi
' p!0,0 00,000 a t t h e date of transfi
onal transfe. r Of b usmess
· interest worth
before Mr. Willy's death. er. Mr. Li failed to meet the conditions
Transfer to Mark - A conditional tr
' Mark fulfilled the condition befor Mansfe~ 0 ~ a car valued at Pl,500,000.
e r. Willy s death.
The properties had the following fair val
ues at the death of Mr. Willy.
Residenti~l lot P 3,000,000
Business interest 9,000,000
car 1.200,000
Total Pl3.200.000
Compute the amount to be included in Mr. Willy's gross estate.
a. p 3,000,000 C. p 12,000,000
b. P 9,000,000 d. P 13,200,000

15. Mr. Yakusa, a Japanese citizen residing in Tokyo, Japan, died with the
following properties in the Philippines:
• PZ,000,000 car - given to a resident Filipino friend as a revocable
donation; Mr. Yakusa waived his right to revoke the donation on his last
visit to the Philippines.
• P4,000,000 share investment in listed firms held by a Philippine stock
broker
• PS,000,000 interest in a par tnership operating in the Philippines
Filipino non-residents a re not s ubject to estate tax in Japan.

Compute the gross estate of Mr. Yu kusa.


a. PO c. P 9,000,000
b. P 2,000,000 d. P 11,000,000

l6. A decedent invested P2,000,000 in the stocks of ABC Corporation by


purchasing stocks from an investor who sold the shares at P 125 per share.
At the date of the decedent's death, the stocks were selling P134 per share.

499
Chapter 13-A-Estate Tax: Gross EState

· ss estate7 M .tiM ~liq


What is the value to include m gro 34 000 000 oon.o
a. p 2,000,000 fl c. p 1268, ooo' 000 0
b. P 2,144,000 ~ d. p ' '
. ''St
\,~

17. Mr. Mando Rucot owns 20% of the 1 ,000,000 outsta~ding shares of
Corporation, a closely-held corporation. DEF Corporation had a book b~
per share of P120 in its financial statement near~st to the date of dea"a!Qe
Mr. Rucot DEF Corporation had several assets which exceed their fair\': or
by an aggregate amount of P14,000,000. Qe

At what amount shall this investment be reflected in the gross estate Oft.ft
Rucot? ·
a. P26,800,000 c. P 120,000,000
b. p 53,600,000 d. P 268,000,000

18. In 2010, Mr. Cabalde died leaving a will wh!ch directed a ll real estate he
owned not to be sold or disposed of for a period of 10 years after his death
and ordered that the property be_ given to Mrs. Yare after the 10-yeai,
period. In 2010, the estat~ had a fa_1r m arket value of P2,000,000. In 2 ,
022
the fair market value of said estate increased to PS,000,000 and the Bureau
of Internal Revenue (BIR) assessed thereon estate tax based on PS,000,00Q

ls the BIR assessment valid?


a. Yes. The assessme nt of the CIR is correct because on matters of
assessment, he has the authority to determine the value to be assessed.
b. No. The assessment of the Cl R is incorrect. The ~1ssess ment should have
been based on the fair market v.1lue .ll the time of death which b
P2,000,000.
c. Yes. The assessment of the CIR 1s corn'<.:t bt'l\tuse 1t was based on the
valu e at the time of assessment.
d. No. The assessment of the CIR is mcorn•rt bt•t-.1ust• estnte tax Is not
s ubject to any assessment.

soo


""T" cnaPte
r 13.a - Estate Tax: Gross Estate of M .
arned Decedents

;\p'fER 13-B
8
f cf;\ 'fE TAX: Gross Estate 01, M .
~ arr1ed Decedents
)~~ c~aPter overview and Objectives
_____________________ _
lltie ····;;~;i·;·;~~pter, readers are-~~-~~~;~~-;~~----------------------------------------
h or Aft understand the presentation of gross estate O .
lltie 1- prehend the various property . fa married decedent
2 corn regimes applicable t
· master the nature of each property regime o spouses
3· be able to compute the separate and excl .
Mt. 4. der absolute community of r us1ve pro~erties of the spouses
un_ P operty and con1ugal partnership of
gains

i he GROSS ESTATE OF MARRIED DECEDENTS


iath
rear The gross estate of a married decedent is composed of:
)22, 1. The decedent's exclusive properties
·eau 2. The common properties of the spouses
oo.
The gross estate of a decedent is reported as follows in the estate tax return:
r- ----- ------------------ ------ - ---- 1
ed. '
I
Conjugal/ I
.ave , Exclusive Communal Total 1

1 GROSS ESTATE P xxx,xxx P xxx,xxx P xxx,xxx :


h is
: Less: Deductions 1
---- - - - - - - - - - -- - - - - - - - - - - - - - - - - - - - - - - -- - - - - - - - _,
the Note: For single individuals, the "conjugal/communal" column is left blank.

not
PROPERTY RELATIONS BETWEEN SPOUSES
For married decedents, the boundary between separate properties and
common properties of the spouses is important in the determination of the
gross estate of the decedent spouse.

What is a separate property or a common property? . .


Wh_at constitutes a separate property or a common property 1s a quest10n
Which Will be delineated by the property regime of the spouses.

Under the Family Code the prope rty relation between the s~ouses ~ust ~e
~reed upon by the ;pouses before their marriage and 1s set m their
renuptial Agreement"

501
. Gross Estate of Married Decedents
Chapter 13-8 - Estate Tax.

Property Relations of the Spouses .


. t f the spouses s hall be determined basect
The property mte res o h f th 011 tt
agreed property regime. Upon the deat o a s pouse, e prop_erties he •1eit
the spouses shall b e classified as separate or common properties depe11ld.~
. ~
on the agree d regime. ~

Common types of property regimes:


1. Absolute separation of property (ASP) - Technically, all properties
the spouses are separate properties, except those properties Which t Of
. . . tl
may acquire 10m y.
hey

2. Conjugal partnership of gains (CPG} - ~ll prope:ties that accrue


fruit of their individual or joint la bor or fruits of their properties d .as
the marriage will be common properties of the spouses. unng
3. Absolute community of property (ACP) - All present properties ow
by the spouses at the date of celebration of the marriage shall beconed
. l d. f
common properties of the spouses me u mg uture fruit of th .
Ille
. f h . e1r
separate or joint industry or fruits o t e1r common properties.

Applicable property regime in default of an agreement


In the a bsence of an agreement or when the regime agreed by the spouses is
void, marriages celebra ted before August 3, 1988 shall be governed by tl,e
conjugal partnership of gains. Marriages celebrated starting August 3, 1988
shall be governed by the absolute community of property.

Conjugal partnership ofgains ~ Absolute community of property

August 3, 1988

Basic Rules in the Determination of Property Interest


1. Common property presumption rule
In the inventory taking of the properties of the estate, the properties of
the spouses
. are .presumed common properties • unless proven to be
exclusive properties of either of the spouses.

This presumption does not I d ty


u d ASP app Yun er absolute separation of proper ·
n er b, ~r~perties are presumed separate of either spouse unless
proven to e Joint properties of the s pouses.

502
ter 13-B - Estate Tax: Gross Estate of Married Decedents
chaP
consistent classification rule
2,
The sale or exchange of properties do not alter their classification
properties acquired using separate properties are separate properties.
Likewise, properties acquired using common properties are common
properties. There is only one exception to this rule under ACP.

Accruals in value or gains on sale ofproperties


3
· The increases in value or gains on the sale of properties are fruits
subject to the rules of the property regi me agreed upon by the spouses.

CONJUGAL PARTNERSHIP OF GAINS (CPG)


This property relation views marriage as a partners hip of gains.

- Classification
- properties before the marriae:e Exclusive
-Properties derive d during the marriage
. From fruits income and gains Common
. From gratuitous acquisitions Exclusive

All properties of the spouses before the ma rriage are their exclusive
properties because there is no partne rs hip yet.

Fruits, income and gains derived by eithe r or both of the s pouses from the ir
industry or property during the marriage a re common pro perties.

Properties that a re not fruits, income or gains s uch as gratuito us


acquisitions like donations or inheritan ce received by eithe r s pouses during
the marriage are exclus ive properties. Exce ptionally, whe n the p ro pe rty
donated or inherited is des ignated by the dono r or predecessor to bo th o f
the spouses, the same s hall be a common property.

Since common properties begin to accrue only fro m the date o f ma rriage,
this property regime is best described as prospective .

(Before marriage) ~ (During marriage)

- ------- Prospective - - - - - ->


Date of marriage
All Properties here are exclusive! All ro erties here ar~mon
· - - ----i
1---_ _ _ _ _ Exception..:. _ _---:-:---- I
• Grdtu itous acqu1s1t1om, received by
either s ouse

503
of Married Decedents
Chapter 13-B - Estate Tax: Gross ESt ate

A detailed look Duri


e oremarria
Exclusive
Fruit of labor or industr Co
Excl usive
Fruit of ro erties Exe
fnheritance or donation Exclusive
received
*conjugal ifdesignated to both spouses

Illustration 1: Fruits of labor of ei tber or bo~h srou:/~~ershi of .


Mr. and Mrs. Calapan were under the conJuga P . P gains. Mrs.
Calapan is employed in a multinational company while Mr. Calapan is
unemployed taking care of their children.


D unng th e marriage,
· Mrs . Calapan acquired various
. properties totaling
Pl0,000,000 from her salaries. Mr. Calapan also discovered a World War II
treasure in their backyard worth Pl00,000,000. Mr. Calapan also won
P200,000,000 jackpot in the PCSO 6/49 Super Lotto.

The following summarizes the classification of the foregoing properties:

Exclusive of Conjugal Exclusive of


Prooerties Mr. oroperties Mrs.
Salaries of Mrs. p 10,000,000
Treasure 100,000,000
Lotto winning-s 2QO,O_OO,OOO
Total e 310.000.000
Illustration 2: Fruits of properties
Before their marriage on November 1, 1987, Mr. a nd Mrs. Negros had
properties respectively of Pl,000,000 car and P2,000,00 0 r eside ntial lot. Mr.
and Mrs. Negros married without agreement on a prope r ty r egime.

During their marriage, Mr. and Mrs. Negros acquired prope rties respectively
totaling PS00,000 and f- 900,000 from th eir sepa ra te la bo r. Mrs. Negros also
sold her residential lot for PS,000,000 cas h.

Shown below are the separate and common prope rties of the s pouses:

Se pa rate p ro pe rties Conjugal


Mr. Negros Mrs. Ne ~ros p ropertigS
Car (owned before marriage) Pl,000,000
Cash from lot sales
P 2,000,000 p 3,000,000
Fruits of la bor (during marriage)
(PS0OK+P800K)
Total ~
P1,ooo,ooo P 2,000,000 ~
504

_,...._,.~
ter 13-B- ESt ate Tax: Gross Estate of Married Decedents
chaP

~ot\t,e spouse_s are u nder the CPG by default because they married before August 3,
1, 8 and dt~ not agree to a _property regime.
198
fransforrnatwns of propert_ies ~y sale or exchange do not change their classification.
2- Hence, the PZM of th e cash ts still separate property since the PZM lot where it came
f om is a separate property.
~nder CPG, all fruits for~ part of the common property. Hence, the P3M cash gained
3- from the sale of the lot (1.e. PSM - P2M) is a common property.

,uustration 3: Acquisition by gratuitous title


uses Rene and Be?e who were under the conjugal partnership of gains had
tie following properties:
5 0

Rene Bebe

~ ..
1. Donations or mhe ntance received P 100,000 P 150,000
20,000
z. Income of proper~ from No. 1 10,000

~
_ Properties acquired from sepa rate
3 400,000 300,000
industry or labor
4. Property received by donation or 800,000 500,000
inheritance 25,000
Income of property from No. 1 and 2 15,000
s. 40,000 30,000
6. Income of property from No. 3 80,000 50,000
7. Income of property from No. 4

Sell.arate Jl.ron.erties o[the s11Q11ses:


Separate properties
Rene Bebe

Before marriage P 150,000


1. Donations or inherita nce received P 100,000
10,000 20,000
2. Income of property from No. 1

During marriage
4. Property received by dona tion or
800,000 500.000
inheritance
Total separate prope rti es
p 910.00...Q e 620.000
Common properties of the spouses:
&M
During marriage
1. Properties acquire d from P 700,000
P400,000 ?300,000
separate industry or la bo r
15,000 25,000 40,000
5. Income of pro perty fro m 1&2
40,000 30,000 70,000
6. Inco me of pro perty fro m No. 3
7- Income of pro pe r ty from No. 4 8 0,000 50,000 130,000
P 940,000
Total common prope rties
sos
. . - t,

te of Mar.ried Decedents , · - :~,:/j


Chapter 13-& "7,E~tate Tax: ~ross E5t a · · ~ -,;·,c._
r,uG'ALPARTNERSHIPOFG:A[ . ,~;"', ·,
INTEGRATIVE ILLUSTRATION: CONJ n. . rs NS t ··
roperties of Mr. and Mrs. Croc . . .
Mr. Croes died. An in~entory o f the P . s ls lltel> •
~~: . ~
.t ~f1~ s, r~-. ,., ~-~ i.i Mr. Croes Mrs. Cro~
1
Properties accruing before marriage.: ~
Properties inherited before marriage . P ;~~,~~~ P !~~,ooo P 30o,00
Other properties brought into the marriage ' ,OOO 90o, ~
00
Properties accruing during marriage:
Properties inherited during marriage P 250,000 150,000
400,000
Properties as fruit of own labor 140,000 160,000
300,000
Properties as fruit of common labor 2so,ooo
Fruits of:
Properties inherited before mar riage 100,000 50,000 1so,o00
Properties inherited during marriage 20,000 80,000 100,000
Properties acquired from own labor 20,000 40,000 60,000
Properties earned from common labor so,ooo
The following shows an analysis of the properties of the s pouses under the CPG:

Exclusive QCQQerties Common


Mr. Croes Mrs. Croes fuuentu
Properties accruing before marriage:
Properties inherited before marriage P 200,000 P 100,000
Other properties brought into the marriage 400,000 500,000

Properties accruing during marriage:


Properties inherited during marriage P 250,000 150,000
Properties as fruit of own labor P 300,000
Properties as fruit of common labor 250,000
Fruits of:
Properties inherited before marriage 150,000
Properties inherited during marriage 100.000
Properties acquired from own labor 60,000
Properties earned from common labor 50,000
Total
P -850~0 P ZS0&)0 ~
Not~: All properties accruing during marriage a r~ rnmmun pr opertit•s except thost
received by way of gratuitous title.

The gross estate of Mr. Croes shall be


Exclusive property of Mr. Croes
P 850.000
Common property of Mr and Mrs. Cruc~
Gross estdte --i!.l!Lililll
P 1..1 60,UOO
506
,,,,,.-
3-B- Estate Tax: Gross Estate of Ma . d O d t
cnaPte r 1 rne ece en s
oiVTE COM~UN~TY.OF PROPERTIES (ACP)
ABSder ACP' marriage
. 1s. v iewed · o t· the properties
. as a u mon . of the spouses a t
un time of marriage mcludmg fruits of their labor and industries in the
rile
rriage.
,118
·al features of ACP
5peCI
i , Retrospec~ve fe~ture
All prop~rties which t~e spo~ses owned before the marriage which they
brought mto the marnage w ill become common properties.

Exception:
a. Properties of a spouse with descendant/sin a prior marriage
b. Properties for exclusive personal use of either spouse, except
jewelry

2, prospective feature
All properties which the spouses may acquire during the marriage from
their separate or joint labor or industry are common properties.

Exception:
a. Gratuitous acquisition received by either spouses.
b. Fruits of exclusive property
c. Properties acquired for exclusive personal use of either spouse,
except jewelry

(Before marriage) ~ (During marriage)

( - - - - - Retrospective- - - - - - - - - - : - - - - Prospective- - - - - ->


Date ofmarnage

Properties of spouse with - Gratuitous acquisitions received by


descendants in a prior marriage either s ouse

Spouses with descendants in a prior marriage


The absolute community of property does not retroact if there are affected
descendants from a prior marriage. Properties accumulated before the
marriage by a spouse with descendants from a prior marriage are reserved
?Y the law as separate properties. This is apparently intended to p~otect the
interest of descendants which could be prejudiced by the new marnage.

507
Chapter 13-B - Estate Tax❖-Gross Estate of Married .Oe'cedents 1

~
Illustration , ::,()~ i
Ms. Beauty Fool, 20 years I old, married Don Mario Montero . M '1 · ,
Milagroso, a wealthy 65-year old businessman known for his alias 'rMonternay0 .
4M had a child with his deceased wife in a prior marriage. r.•4~:· ~t,t
Ms. Beauty brought into the marriage ~ropertie~ totaling PS0,000. Mr
brought into the marriage properties totaling P?0,000,000. Du/1'4 also
marriage, Ms. Beauty accumulated P300,000 from her salaries. tng the

Mr. _4M can no longer work at his age so he is totally dependent from h
of his properties. His properties earned Pll,000,000 during the marria;/ fruits

The following shows an analysis of the properties of either spouse:


Exclusive properties Common
Mr. 4M Ms. Beauty Properti~
Properties brought into the marriage:
Mr. 4M P70,000,000 P - P
Ms. Beauty so,ooo
Properties acquired during marriage:
Fruits of properties 11,000,000
Cash - salaries of Beauty 300,0QQ
Total P81.000,000 c!:
p===~- p 350,00Q
Note:
1. The ~etrospective rule does not apply to a spouse with a deseend ant m
marnage. . a prior

2. The fruits of properties follow their source prope rty. The fruits 0f I b .
are common properties. a or or mdustry

Summa of rules: S ouse with descendant in a rior marria e

ired
- Fro tions Exclusive*
From Exclusive Communal
From
- Se Excl usive Exclusive
- Co Excl usive Communal
For exclusive use of either 5 ouse
- Jewelr
- Non-·ewelr Excl usive Communal
Exclusive Exclusive
Property for exclusive personal
Unde r ACP, properties for the e~se, e~cept jewelry
s pouses, except jewelry a r cluSive pers onal use of either of the
I
acquire d b efore or duri ~g t~ a way~ exclusive property whether they are
w·11 b ecome common prop e marriage · Jewe Iry a cquired before marna . ge
1
erty. Note that this rule d oes not exist wi th crG.
508

t,
,.. cnaP
ter 13-8 - Estate Tax: Gross E
state of Ma .
. rned Decedents
1.1strauon
Jll nd Mrs. Quezon had the folio .
r,-1r. \ ed before marriage: Wing Personal properti .
A~~ es.
r uxedo of Mr. Quezon
' Wrist watch of Mr. Quezon p 3,000
' car 2,000
' Gown of Mrs. Quezon
800,000
'
ACquired during marriage: 40,000
, Earrings and necklace of Mrs · Quezon
cellphones P 150,000
,' Louis Vuitton bag of Mrs. Quezon 50,000
Clothes, shoes, and underwear of M 150,000
' h r. Quezon
, Clothes, s oes, and underwear of M 60,000
rs. Quezon
80,000
The following are the exclusive and com
muna1properties of the spouses:
Exclusive properties Common
wiuired before marria2e: .Mr. Ouezon Mrs. Ouezoa_ Properties
Tuxedo of Mr. Quezon p 3,000 p
Wrist watch of Mr. Quezon - p
2,000
Car
Gown of Mrs. Quezon 800,000
40,000
Acguired durin2 marria2e:
Earrings and necklace
150,000
Cellphones 50,000
Louis Vuitton bag of Mrs. Quezon 150,000
Clothes, shoes and underwear 60,000 80,000
Total 63,QQQ
ap=-~W!!:i!:!~ e 270J10.Q
Note:
1. The price of the property has no thing to do with the property cla~slnrat1on
2. Note that cars a nd cellpho nes cannot be class1tied as properties tor exclusive personal use
of either spouse.
3. Properties for exclusive personal use of either spouse. other than 1ewelry, are separate
property even if:
a. They are acquired fro m fruits of labor or Industry of either spouse or from fruits of
common propertte'-.
b. They are acqu ireu before or during the mamJge.

SEPARATE PROPERTIES ACQUIRED DURING MARRIAGE


Properties rece ived by w ay of gra tuitous title . ,
S1rn1lar to conjuga l partnership of gains, properoes received by way of
gr t · · heri tance during marriage 1s a
a u1tous title s uch as donanon or m d edent co be fo r both
separate property unless designated by the donor or ec
spouses.
509
of Married Decedents
Gross Esta t e
Chapter 13-B - Estate Tax:

Fruits, Income or Gains


Fruits of labor and indu5try . dustry of either or both spouses
Fruits arising from t h e labor or m i:lte
. o f the spouses.
common properties

Fruits of properties is a separate property. It follows there~


The frmt of separate property is a common property. In short, "Fr 0_re
that the fruit of common property "'ts
follow principal."
. 'fication is unique. Note that compared W'
This qualification on frmt c1assi 1 . Ith
CPG, all fruits are common properties.

Summa of Rules: Absolute Communi


Duril'JJJ.
marria e marri e
Communal Exclusive*
- From Communal Communal

Communal Exclusive
Communal Communal

- ]ewe Communal Communal


- Non- Exclusive Exclusive
*communal ifdesignated to both spouses

Illustration 1
Mr. Ato, a married decedent under the ACP, received the following properties by
gratuitous title:

Before marriage Durin~ marria~e


Residential lot - gift to Mr. Ato P 2,000,000
Commercial lot - gift to Mrs. Ato
P 5,000,000
Income of residential lot 400,000 800,000
Income of commercial lot
Income from own labor - Mr. Ato
1,400,000
200,000 800,000
Income from own labor - Mrs. Ato
300,000 800,000

The following shows a summary of the propert· f h


1es o t espouses:

510
ter 13-8- Estate Tax: Gross Estate O f M .
cha P arned Decedents

Exclusive properties Common


Mr. Ato Mrs. Ato Properties
R sidential lot p - p
- P 2,000,000
c:mmercial lot 5,000,000
me - residential lot
IJlCO . 1,200,000
,.,,,e - commerc1al lot
JnCO1 " 1,400,000
income - own labor (Mr.)
1,000,000
income - own labor (Mrs.)
p 1,100.000
rota! ~===MQ P 6.400.000 P 5.300.000
Note:
A ll properties acquired before marr1·a . . •
1. . . ge are community or common properties. This
includes
• those acquired
·ft d • by own labor or by gratuitous ac · ·t· (R t t· ,1
qu1s1 10n. e rospec 1ve1
z. Inheritance or gi urmg the marriage is a separate property, unless designated for both
spouses.
3. The income of properties follows the classification of their source.
4. The income from the labor and industry of either or both spouses is common property.

Illustration 2
Mr. and Mrs. Negros married on January 10, 1990 without agreeing on a
property regime. Mr. and Mrs. Negros brought into their marriage respectively a
Pl,000,000 car and P2,000,000 residential lot.

During their marriage, Mr. and Mrs. Negros acquired properties totaling
PS00,000 and P800,000, respectively, from their separate labor. Mrs. Negros
also sold her residential lot for PS,000,000 and invested the entire proceeds in
stocks.

Shown below are the separate and common properties of the spouses:

Separate properties Community


Mr. Negros Mrs. Negros Properties
Car p p P 1,000,000
Stocks (gain and principal) 5,000,000
Fruits of labor (PSOOK+PBOOK) 1,300,000
Total !cr====!ao :!::::e====!ao P 7.300.000
Note:
1. The spouses are under the ACP by default because they got married after August 3, 1988
and did not agree to a property regime. . . . . . .
2. Under ACP, the gain on sale of separate prope1:1es_ dur~ng the marriage 1s a fruit which
follows its principal classification. Since the pnnc1pal 1s a common property, the P3M
gain (PSM - P2M) is also a common property.

The rule on jewelry .


Jewelry • th lusive use of either spouse 1s generally a common
1or e exc "f • l'fi h
property. However, jewelry is an exclusive property 1 1t qua 1 es as sue
under other rules of ACP.

511
Chapter 13-B _ Estate Tax: Gross Estate of Married Decedents

It Is a basic rule in statutory construction that sta t utory provisio


interpreted in unison and not in conflict with each . Doubts shall bns 5 ha11 b
in favor of the general provision ra ther than the exceptions. e teso1,..e:

Illustration
During the ma rriage, Mrs. Aguilar recei~ed a gold br~celet worth P4oo
inhe ritance from her grandparents. Using her salanes, Mrs. Aguil '0Oo as
Pl00,000 worth of shoes and clothes and a P200,000 diamond ring
u~. o~
ro:\~~ll&ht
On the other hand, Mr. Aquilar bought a Rol~x watc_h worth P30,00Q from
income of a property which was donated to him dunng the marriage. Usin 1~e
business income, Mr. Aguilar also bought _shoes a~d clothes worth P70,ool his
a platinum necklace worth P250,000 for his exclusive personal use. and

The followi ng shows an analysis of the properties:

Exclusive properties Common


Mr. Aguilar Mrs. Aguilar Properties_
Inherited gold bracelet p - P 400,000 p
Shoes and clothes 70,000 100,000
Rolex watch 30,000
Diamond ring 200,000
Platinum necklace 250,00_Q
Total P 100,000 P 500,000 P 450,000
Note: Properties inherited during marriage are still separate properties, even if they are
jewelry. Fruits of separate properties are still separate properties. Hence, the inherited gold
bracelet and the Rolex watch are separate properties.

Exception to the Consistent Classification Rule


Under ACP, properties acquired for the exclusive use of either of the spouses
during marriage, other than jewelry, are exclusive property.

Illustration
Mr. and Mrs. X are under the absolute community of property. Mr. X had a salary
of P300,000. With his permission, Mrs. X went on a shopping spree and
submitted the following liquidation:
Jewelry P 22,000
Cellphone 20,000
Fashion bag 120,000
Gown and signature dresses 100,000
Total for Mrs. X P 262,000
A pair of shoes for Mr. X 3,000
Total cash s pent 1
P 265,000
Remaining cash P 35,000
512
3-8 - Estate Tax: Gross Est t Of .
chapter 1 ae M arried Decedents
.01,crtics ~hall he cl.issifird .1~ followL"

(-
fflC P'
--- -----

propt•rt i<'S
Exdusivt• of
Mr.
.,.
Common
J>ropcrty
f:xduslve of
Mrs.
l /fl\"l'j I.\ I' 21. .000
I r ' pl111lll' L0.000
h1nll h,ig p J 20.000
J.l' ~ ,,gn:iturc dre..,~es
( ,,1, I1 •
Ap,1
,r of -.,hoes lor 1\1, . X -- p 3,000
100,000

Rt '11•11111
ng c.1s h - a l..1 '1 ' ___ JS,.000
rotal cJ h spent r 3 000
-- £. 77,00U. l20 000
- J>
Additional Illustrations: Absolute Community of Property

Integrated Illustration 1
. pou. es Rene a nd Bebe who were under the abso lute community of property
had the following properties:

&M ~ Toral
Dtfore marriage
1. Donations or inheritance received P 100,000 P 150,000 P 250,000
2. Income of property from No. 1 10,000 20,000 3 0,000

Qynng marriage
3. Properties acq uired from separate
industry 400,000 300,000 700,000
4. Property received by donation or
inheritance 800,000 500,000 1,3 00,000
5. Income of properties from No. 1
and 2 15,000 25,000 40,000
6. Income of property from No. 3 40,000 30,000 70,000
7. Income of property from No. 4 80,000 50,000 130,000

Separate properties of the spouses:


Separate properties
Rene Bebe
Bg/ore marriage
p p
l. Donations or inheritance received
2. Income of property from No. 1
Qy__ring marriage
4- Property received by donation or
800,000 500,000
inher itance
7· Income of property from No. 4 80,000 50,000
Total separate properties P 880.000 P 550.000

513
Chapter 13-B - Estatt! Tax: Gross Estate of Married Decedents

Community properties of the spouses:


~
Before marriru,e
1. Donations or inheritance
Pl00,000 PlS0,000 p
received
20,000
2so,ooo
2. Income of property from No. 1 10,000 30,000

During mauiage lfJi.)


3. Properties acquired from
separate industry P400,000 P300,000 100,000
5. Income of properties from 1&2 15,000 25,000 40,oo 0
7. Income of property from No. 3 40,000 30,000
~
Total common properties .e_1,020.0 00
:!...:!£:

Integrated Illustration 2
A husband who married under the absolute community of property on Jan
1, 2019 had the following transactions in 2019: uary
• Received P 1,000,000 as inheritance
• Used P 100,000 to buy dress and shoes for his wife
• Used P 800,000 to buy a parking lot
• Earned P400,000 compensation income
• The parking lot earned P 120,000 rentals
• Used P20,000 of the rental income to buy jewelry for the s pouse
• Used P30,000 of the rental income to buy gown for the wife

The following depicts the analysis of the foregoing transactions:

Wife's Communal Husband's


Para.nhernal Properties Cqpital
Inheritance P 1,000,000
Dress + P 100,000 + - - - - - - - - - - - - - - - - 100,000
Parking lot - 800K + BOOK
Cash salary P 400,000
Cash rentals 120,000
Jewelry 20,000 ◄ ----- - 20,000
Gown 30,000 ◄- - - - - - - - - - - - - - - - - - 30,000
Total P 130,000 P 420.000 p 970,000

Integrated Illustration 3
Mr. Croes died. An inventory of the prope rties of Mr. a nd Mrs. Croes who were
under the absolute community of property w er e as follows:

514
r !3-B - Estate Tax: Gross Estate of M .
c~apte arned Decedents

· betore
:ne.s--accru mg , marria~: ..Mr, CroQ Mrs, Croes Total
~ ~ies inherited before marriage p
proPe'ries for exclusive pe rsonal use 20 0,000 p 100,000 P 300,000
0 per 50,000 60,000 110,000
~~her properties brought into the marriage 350,000 440,000 790,000

~ accruin~ during marriag~:


rties inherited during marriage
P 250,000 150,000 400,000
proP: rties as fruit of own labor
proP . d, l . 140,000 160,000 300,000
properties acqu1~e 1or exc us1ve use 30,000 40,000 70,000
properties as fruit of common labor 250,000
fruits of: . . .
properties mhented before marriage 100,000 50,000 150,000
Properties inherited during marriage 20,000 80,000 100,000
Properties acquired from own labor 20,000 40,000 60,000
Properties earned from common labor 50,000

The following shows the classification of the properties of the spouses under
ACP:

Exclusive properties Common


M~ Croes Mr~ Croes Properties
Properties accruing before marriage:
Properties inherited before marriage P 300,000
Properties for exclusive personal use P 50,000 P 60,000
Other properties brought into the marriage 790,000

Properties accruing during marriage:


Properties inherited during marriage P 250,000 150,000
Properties as fruit of own labor P 300,000
Properties acquired for exclusive use 30,000 40,000
Properties as fruit of common labor 250,000
Fruits of:
Properties inherited before marriage 150,000
Properties inherited during marriage 20,000 80,000
60,000
Properties acquired from own labor
50,000
Properties earned from common labor
Tota) p 350,000 p 330,000 P1,900,000

The gross estate of Mr. Croes shall be:


p 350,000
~Xclusive property of Mr. Croes
0 1,900,000
tnmon property of Mr. and Mrs. Croes p 2,250,000
Gross estate

515
Married Decedents
ax· Gross Estate of
- Estate T ·
3
Chapter 1 - B JES

A F EXEM PT pROPERT . be included as exclusi"


f es WI 11 d · h ve
CQUISITION . . O f exempt properbut
I
sha ll be exclude mt e cornPut<tt1. Ot
The acquis1t10n _o f the spouses
common properties o 011
of the gross estate.

. t employees. Mrs. Ravena died duri


Illustrat1on both governmen ·ved a lump sum GSIS ben ~g
Ravena are 5 he rece 1 e11t
Mr. and M_rs. fore her death, his was expended before her death
the marriage. Bi' 000,000. p 200,000 of ~hich now has a value of P900,0oo
amountmg to p '. vested in a propertyOO 000 other common properties. l,.
The balance was m h e P4,0 , . I f Pl 800 i•1r.
. from t hese' the hspouses
Aside av prope rties respective y o
separate , ,Ooo and
and Mrs. Ravena also ave
P2,200,000.

. o f the ,spouses s ha ll be computed


The common properties . as:

Investment:
P 800,000
- Original principal GSIS bene~~BOOKJ
_ Income investment (P900K 100,000
Total investment value P 900,000
Other common properties 4,000,000
Common properties . . /J P 4,900,000
Less: Exempt properties (GSIS benefits pnnc1pa 800,000
Taxable common properties P 4,100,000
The gross estate of Mrs. Ravena shall be:

Exclusive property of Mrs. Ravena P 2,200,000


Taxable common properties
Gross estate 4,100,000
P 6,300,000
Note:

1. The principal of exempt properties shall be removed from the reportable gross estate.
2. The exclusion of exempt properties cannot be extended to the income of exempt
properties. Hence, the Pl00,000 income from the investment which was acquired using
GSIS benefits is included in gross estate.

EXEMPT PROPERTIES OF NON-RESIDENT ALIEN DECEDENTS


The exempt foreign properties of a married non-res ide nt alien decedent
must be excluded in gross estate.

Illustration
Mr. Speedy Gonzales, a non~resi·dent M d Mrs
Gonzales had the following properties: exican, died in Korea. Mr. an ·

516
ct,apter 13-8- Estate Tax: Gross Estate of Married Decedents

Mexico_ .Pbilippine.s_ Korea Total


. roperties P l00,000 P 200,000 P 300 000 P 600,000
capital prnal properties 250,000 350,000 ' 1,200,000
paraphe properties - tangible 550,000 200,000 600,000 950,000
fl'lrnon . . . 200,000
co0111100 properties - mtangible - 500,00..Q 150,000 650,000
rota!
co P 900.00Q P.1.250.00J! .e 1.2so.ooo P 3.soo.ooo

rhe gross estate of Mr. Gonzales shall be:


d's capital properties P 200,000
ttusban n properties - tangible
200,000
cornmo n properties - intangible
cornmo 500,000
Gross Estate P 900,00Q

A5sum1•ng that the reciprocity rule applies, the gross estate of Mr. Gonzales shall
be:

Husband's capital. properties


. P 200,000
Common properties - tangible 200,000
Gross Estate P 400.000

517
.G Estate of Married Decedents
Chapter 13-B - Estate Tax. r 055

CHAPTER 13-B: SELF-TEST EXERCISES

Discussion Questions
·
1 . Wh a t is operty regime?
a pr . . he types of property regimes
2 Enumerate and bnefly discuss t . ·
3~ Compare the absolute community of property to the con1ugal Partnershi 6-
gains . . Pai 1.
d d·scuss the list of exclusive properties under
4. Enumerate an i abs01
community of property ~~ s.
True or False 1 . 9.
1. The spouses can stipulate the conj~gal partnership of gains as th.
eit 10.
property regimes even in the current time.
2. The property regime of the spouses may be agreed upon dur·ing the
marriage. .
3. ln default of agreement as to the_property relation between the spouses, the 11.
absolute separation of property is presumed. . 12.
4. Fruits accruing during the marriage are conclusively presumed conunon
while fruits accruing before the marriage are conclusively presumed 13.
exclusive.
s. The absolute community of property applies on fruits prospectively from 14.
the date of marriage.
1s .
6. The conjugal partnership of gains operates retrospectively and
16.
prospectively.
17.
7. Properties for exclusive personal use of either spouses are exclusive
properties under absolute community of properties.
18.
8. All fruits, accruing before or after the marriage, are conjugal properties.
9. Fruits accruing from common properties are common properties under
conjugal partnership of gains.
10. Fruits accruing from separate properties are common properties under
20
conjugal partnership of gains.
11. All fruits before the marriage are conjugal properties.
12. All fruits during the marriage are communal properties.
13. Real properties are common properties under absolute community of
properties.
14. Personal or movable properties are separate properties under absolute
community of properties.
15. Properties received by way of gifts are exclusive properties.

True or False 2
1. Properties ~eceived by way of inheritance are exclusive properties. gal
2. All properties brought into the marriage are separate under the con1u
partnership of gains.
3. Generally, all propert·ies b rought into the marriage are con"IJ1\un 1t}
.. ,
properties.

518
13~8- Estate Tax: Gross Estate of M . d
chapter arne Decedents
roperties of a spouse with a d
The P t· escendant from a prior marriage are
4-. iusive proper 1es.
exc 1·t 0 f inherited pro t'
rhe fru _ per ies are exclusive under absolute community of
5- ropert1es.
P fruit of donated properties ar I •
The . . e exc us1ve properties under conjugal
6- artnersh1p of gains.
~he frui~s of labor of ei th er spouse are exclusive under the absolute
7. con1lnunity of property.
The gross e state. of a _dece_dent includes his separate properties and their
8· common properties with his surviving spouse.
Marriages celebrated after August 3, 1988 are conclusively presumed under
9
· the absolute community of property.
10. In default of an agreement between the spouses, marriages celebrated
before August 3, 1988 are presumed under the conjugal partnership of
gains.
ll. Issues of property regime are irrelevant to a single decedent.
12. In taking inventory, properties are generally presumed common unless
proven as exclusive of either spouse.
13. The proceeds of separate property sold during the marriage can become a
conjugal property.
14. The proceeds of separate property sold during the marriage is always a
separate property under absolute community of property.
15. jewelry is generally considered community property.
16. jewelry inherited during the marriage is exclusive property.
17. The sale of a separate property may produce a separate property and a
conjugal property.
18. The sale of a conjugal property may produce a separate property and a
conjugal property.
19. The properties of a spouse with a descendant from a prior marriage are
communal properties.
20. The properties of a spouse without any descendant from a prior marriage
are communal properties.

Multiple Choice - Theory: Part 1


1. Statement 1: Under absolute community of property, properties are
presumed community unless proven otherwise.
Statement 2: Under conjugal partnership of gains, properties are presumed
conjugal unless proven otherwise.

Which is correct?
a. Statement 1 c. Both statements
b. Statement 2 d. Neither statement

2. In determining the property regime of the spouses, which is given primacy?


a. Agreement
b. Conjugal partnership of gains
519
l ,.., -, • r' !
c. Absolute community of property
d. Absolute.separation of property

3. Which is not a property regime?


a. Absolute separation of property
b. Conjugal partnership of gains
c. Absolute community of property
d. Pooling of interest

4. Which is a conjugal property?


a. Property inherited during marriage
b. Property inherited before marriage Qr
c. Property received from exercise o f pro~ession during marrtage
d. Property received from a d onation dunng marriage

5. Which of these is an exclusive pro perty?


a.
Fruits aris ing from properties inherited during man;
b. Fruits of properties acquired before marriage
c. Properties acqu ired from persona l labor of th wife
d. Properties donated to both of the s po u

6. Which is a community property?


a. Properties received lnh ri tnc:1.• durmg m 1rn g
b. Properties representing (nm-. 01 ommunu • prop nl,
c. Proper ties repr scnting f1 ult from tlon 1H•d prop r
d. Fruits of inhe ritanc durin • m ni 1 , •

7. Wh ich is a commo n prop rt 'lHHlt~r b 1lucc •p IUOn Olp


a. Sa laries of the husb~1nd
b. Bus iness profit of t he w11 •
c. Donatio n or in hcritann• re •iv , h\' t•l~h 1
d. Bus iness interes t Jointly 11n inc• 1 hv h h

8. Which of these d onations or rnh •ill 11


l J)f p •rt
a. Donattons des1gn,H~d hy the donor r
b. A d evise to the husb rnd
c. A donation de ignatl•d by th don r I t th • hu
d d th ,r
d An 1nheritann• trom tht! mothci o: h

9 Which not .1 common propt•r \


1.s
,1, Re\ Ol,1ble don.ttwn lo bot·~,
b I POl!St·
. lrrcnll.1blt' don.lCJon In ru l r rb
t D1rect.:.u1dur \O iblt:dorw·
d All ot the!>~
~ ter 13-B - Estate Tax: Gross Estate O f M .
cnaP arned Decedents
Which is not a sepa rate property und .
10· professional income of the h b er conJugal partnership?
a. . us and
Rental mcome from separate
· propert1es
.
b•
(.
income o f common properties
d. All of these

Vilhich is incorrect regarding fruit O


f
I1- . •age? s separate properties during the
111a1 n ·
Exclusive under conjugal partner h' f .
a. , .. d . s 1p o gains
b Exclusive un e1 absolute commu 111·ty f
· · d o property
c. Exclusive un er a bsolute separation of ro er
d. None of these P P ty

12 _ Fruits accruing before the marriage are


a. Common under conjugal partnership of gains
b. Comm?n under absolute community of properties
c. 1x~~ste under absolute community of properties
d.

13. The incom~ of donate~ properties before marriage are


a. Exclus~ve properties under conjugal partnership
b. Exclusive under absolute community of property
c. Common properties under conjugal partnership of gains
d. A and C

14. Property inheritance during the marriage is


a. Exclusive under conjugal partnership
b. Exclusive under absolute community of property
c. Common properties under conjugal partnership of gains
d. A and B

15. The property inheritance before the marriage is


a. Separate property under conjugal partnership
b. Separate property under absolute community of property
c. Common property under absolute community of property
d. A and C

Multiple Choice - Theory: Part 2


1. Which is incorrect under the absolute community of property?
a. Jewelry received as an inheritance during marriage is an exclusive
property.
b. Jewelry received as a donation before marriage is a common property.
c. Jewelry acquired from income of separate property is an exclusive
property.
d. None of these
521

• *
Chapter 13-8-Estate Taj(: Gross-Estatebf Married Decedents

2. The income of properties acquired from the personal hard Worft of


spouse is • '
a. Exclusive under absolute separation of propertres
b. Common properties under absolut~ community of property ..,
c. Exclusive under conjugal partnership
d. A and B

3. The husband has numerous pricey personaJ apparels. These are


I
a. Exclusive properties under conjugal partnership
b. Common properties under absolute community of property
c. Common properties under absolute separation of property
d. None of these

4. The gain on sale of a separate property during the marriage is


a. Separate property under conjugal partnership of gain
b. Separate property under absolute community of property
c. Common property under conjugal partnership of g In
d. A and 8

5. The gain on sale of common propert1e is


a. Exclusive under conjugal partnership ot R Jin
b. Common under absolute p ration or J')mp ·rt ,
c. Common under absolut" communfrv of pro1 ·rt
d. None of these •

6. Which is not cons1dt,•n,•tl ,l I t prns n . Uw r • Ipl nt p u


a. Acquisition of propt•rt)' 1 I ln1
b. Acquisition of propt•rtv
Odud ry h Ir
c. Acquisition of propt.'rt)• t:,; u ufru u r,
d. All of these ·

7. :'hich is .1 par,1pht.'rnal propNty und ·r b ofu


b. Property inht•rited hy tht hush n I m, UH o pro rt
r Property lnht•ntl•d hy tht• w,tc:
Property earned from s ·p r r"
d Property from the rxcrcJ st f J>r'OJ r •
·o p: or 1 n lh \\
H Wh ,c' h ·is u pJr,lpht•rnal Ito
.1. Prope, ty brought 111'co r~eu, uud ·r th c n u
b. Jncume of p . em 1rn..1 lt• bt ch p
ropt•n1e don.:1t d , •
t Income lroru the c lo the\\ Uc I
I I ~epdr tte If1d
<. nco111t• trom tli , • u~tn r th
t: :.t: p.1ract• lndust lt
, ptef 13
_8 - Estate Tax: Gros Estate of Married Decedents

()'13 h of thrsP is a rapil ;il prorw n y u nder ronfur,al !hHtner hip of gains?
q Wo'~rop<'rtlf'!'- hrnugh t rn to thl' rn,11 n..igc by the w 1f
.J. rrorrrti<'~ mhr11tr<l hy th t" h11c;h,111cl
b. FrtJit<. of pr opr1tic~ do n,1tr d to the h11 c;h,H1d
r Pi ope, tic~ for <' 'elusive US(' of the husband
d
·"" h 1 .1 rap1t.ll property under th absolute community of property?
10 '"'''''r rorrrt.1es before ma, 1,age of the w1tc with a descendant in a prior
' .
a-
mam,1ge
b Fruits of c;eparate prope rty of th e wife
c. propf'rt1c~ fore elusive personal use of the husband
d. Com pen. Jl10 11 income of the husband

Which I excluded 111 gross estate?


11
a. eparate property of the decedent
b. Common property of the spouses
c. eparate property o f the surviving spouse
d. A and C

l2. WhJCh is excluded in the gross estate of a deceased husband under the
absolute community of property?
a.. Fruits of properties inherited by the wife during marriage
b. Fruits of commo n pro perties during the marriage
c. Fruits of properties of the spouses before their marriage
d. Properties inherited by the husband

13. Which is excluded in the gross estate of a deceased wife under the conjugal
partnership of gains?
a. Business income of the husband
b. Professional income of the wife
c. Property received by way of inheritance by the wife
d. Property brought into the marriage by the husband

14. Which will not be included in gross estate regardless of the property regime
of the spouses?
a Accruals from SSS
b. Jewelry
c. Properties for exclusive use of the spouses
d. Fruits of separate properties
15
· Which is excluded in the gross estate of the husband under the conjugal
partnership of gains?
a. Properties inherited by the wife
b. Properties brought into the marriage by the husband
c. Income from properties inherited by the wife
d. Properties acquired by the wife from her own labor

523


Chapter 13-B - Estate Tax: Gross Estate of Married Decedents r
ct
Multiple-Choice - Problems: Part 1

~~1 .
1. Mr. Andrenico brought into the marriage . an agricultural Ia 1-
Pl,000,000. During the m arriage, the a?ncultural land Waslld \vo
Pl,500,000 and was used to acquire a family ~ome. The family h SoJd ~
0
valued at Pl,800,000 at the death of Mr. Andremco. nie \v~

Compute the amount to be include_d in t~e common properti


spouses under the conjugal partnership of gams. es of
the
a. p 500,000 C. p 1,300,000
9.
b. P 800,000 d. P 1,800,000

2. In the preceding problem, compu te the amount to be included .


communal properties of the spouses. in the
a. P 500,000 c. P 1,500,000
b. P 1,300,000 d. P 1,800,000

Case 2 C.
The following problem applies for Numbers 3 through 10. T

Before their marriage, Mr. and Mrs. Bone ti ha d salary savings r espectiv Li
I
PZ,000,000 and Pl,500,000. Mr. and Mrs. Boneti earned respectively P20~ ta°' p
and PlB0,000 income from these savings during the marriage. Mr. and ~I O
Bonetti also earned r espectively P400,000 a nd PS00,000 from th eir sepa rts. T
· d ustnes.
m · ra e

Under the absolute community of property, com pule th e following:

3. Separate property of Mr. Boneti.


a. p O C. p 2,000,000
b. P 400,000 d. P 2,400,000

4. Separate property of Mrs. Boneti.


a. p O C. p 1,500,0QQ
b. P 500,000 d. P 2,000,000 "
5. Common property of the s pouses. 1
a. p 900,000 C. p 3,500,000
b. P 1,280,000 d. P 4,780,000

6. The gross estate of Mr. Boneti.


a. p 1,680,000 C. p 4,780,000
b. p 3,500,000 d. p 5,500,000

524
ter 13-B - Estate Tax: Gross Estate of M arried Decedents
cnaP
der the conjugal partnership of gains, compute the following:
v11
sepo,,rate property of Mr. Boneti .
7. a pO C. p 2,000,000
b: p 400,000 d. P 2,400,000

separate property of Mrs. Boneti.


8· a. PO c. P 1,500,000
b. p 500,000 d. P 2,000,000

common property of the spouses.


9
· a. p 900,000 C. p 3,500,000
b. p 1,280,000 d. P 4,780,000

10. The gross estate of Mr. Boneti.


a. p 3,280,000 C. p 4,000,000
b. P 3,500,000 d. P 4,780,000

case 3
The following problem applies to Numbers 11 through 18:

Lovely, married Andy, a 60 year-old lawyer, who had two children from a
previous marriage.

The spouses had the following properties:


Lovely ~
Before marriage:
Total properties P400,000 P 4,000,000

During marriage:
Income from separate industry 200,000 2,000,000
Income of properties brought to the marriage 80,000 700,000
Inheritance and donations received 450,000 500,000

Assuming the conjugal partnership of gains, compute the following:

11. The separate property of Lovely


a. p 400,000 C. p 850,000
b. P450,000 d. P 1,050,000

l2. The separate property of Andy


a. p 4,000,000 C. p 4,700,000
b. P 4,500,000 d. P 5,200,000

525
111 . 1, 1 d, · ,-,f M:,rrlcd Decedents
<" ,pt 111 1 t.. n 1•.111 11 , 1><: " ••

I :1. 'I'll,, 1•11 1111111111 p1'1lpf•1•t y ol I Ill 1111111 IM


(', ,, 11,::00,000
1, I' :~,1,110,000
h, I' /., 11110,000 (I, I' !i,(ill0,000

I ti. 'l'hl' Hl'Oll:: ,•:11 ,1 1• ol l,ovt•ly


11. I' :1, I :I0,0(1() ,•. 1, :1,1J:w,ooo
h, 11 :1,:i:w ,000 ti. fl 7,IIII0,000 zO·
Ass11111l11g lho nhsol11t ",.,.,111111 u,ilty of' iu·orcrly, compute the following:

t !i. Th t• s (•p:i ,·:ilt' prnpc·,·ty of l.ov<'ly


I) 'ViO ooo . I' H!i0,000
lt. I • r: (JOO
h. P '1•:iO,OOO d. p 1,CJ.JO,

16. Th e st•pnrnrp property of Ancly


a. P '1-,000,000 c. P '1-,700,000
b. p '1•,!WO,OOO u. p 5,200,000
l 7. The common prope rty of Lh c spouses
a. P 2,6110,000 c. P 4,SOO,OOO
b. I' 2,900,000 d. P 5,680,000

18. The gross esta te of Lovely


a. P 3, 130,000 C. P 3,830,000
b. P 3,330,000 d. P 7,880,000 24.
Case4
The following problem applies to Numbers 19 t h rough 26:
Mr. Corn elius died. An inventory a nd analysis of the properties held by his
fami ly Me presented below: 25.

Mr. Mrs.
/!mperties ac:qufred before marria.ge_:
Cometius. Cornelius ISlliJl
26.
Properlies for excl usive pe rsonal use p 20,000 p 30,000 p 50.000
Other prope r ties acquired
Total 28Q,QQQ 1ZQ,QQQ 750,0QQ
f 300.00~ .e SQO.QOO
_ , eaoo.orui
Mui
PrQperties OCfJuired during marriOJM:
Properties fo r excl usive personal use p 1.
Properties from own industry 30,000 p 40,000 p 70,000
Donated properties received 290,000 500,000 790,000
Inherited properties 300,000 300,000
Fruit of donated/inh erited property 400,000 400,000
Total 8Q,QQQ_ QQ,QQQ ~
E. 800,00.Q £ 900.000 ~
526
. ter 13-B - Estate Tax: Gross Estate of M .
c110P arned Decedents
iing the conjugal partnership of gains d t .
Ass11 11 • e ermine the following:
scparat·r property of Mr. Cornelius.
9
I . 9. p 480,000 C. p 530,000
b. p 51 0,000 d. p 700,000

sep,1rr1tc property of Mrs. Cornelius.


20
. a. P 400,000 C. P 800,000
b. P 430,000 d. P 860,000

21. The common prope rty of the spouses.


a. P 1,000,000 C. P 1,540,000
b. P 1,160,000 d. P 1,590,000

22. The gross estate of Mr. Cornelius.


a. p 1,640,000 C. P 2,070,000
b. P 1,700,000 d. P 2,100,000

Assuming the absolute community of property, determine the following:

23. Separate property of Mr. Cornelius.


a. p 480,000 C. P 530,000
b. P 510,000 d. P 700,000

24. Separate property of Mrs. Cornelius.


a. p 400,QQQ C. P 800,000
b. P 430,000 d. P 860,000

25. The common property of the spouses.


a. p 1,000,000 C. p 1,540,000
b. P 1,160,000 d. P 1,590,000

26. The gross estate of Mr. Cornelius.


a. P 1,640,000 c. P 2,070,000
b. P 1,700,000 d. P 2,100,000

Multiple-Choice - Problems: Part 2


1. Mr. Jose ma rried Josephine on February 2, 1988. Josephine died on
February 14, 2019. On that date, the spouses had the following properties:

Car, donated to Mr. Jose on June 14, 2015 P 1,200,000


Investments - inheri ted by Josephine on Feb. 4, 1995
when its va lue w as P2M 2,800,000
House and lot - salaries of Mr. Jose and Josephine 4,000,000
Cash income of car 400,000

527
Chapter 13-B- Estate Tax: Gross Estate of Married Decedents

Compute the separate property of Mr. Jose.


pO c. p 1,200,000
~~ P 400,000 d. p 1,600,000

2. Compute the separate property of Josephine.


a. p o c. P 2,800,000
b. p 2,000,000 d. P 3,200,000

3. Compute the gross estate.


a. P 5,200,000 c. P 7,200,000
b. P 6,400,000 d. P 8,000,000

4. Mr. A died on June 3, 1987, but his_ estate ?ad not paid tax since then. He
had the following properties at the time of his death:
7.
Proceeds of life insurance irrevocably
designated to his son P 2,000,000
Properties for exclusive use of Mr. and Mrs. A 300,000
Properties inherited by Mrs. A on June 1, 1987 4,000,000
Properties from salaries of Mr. A and Mrs. A 1,400,000
Properties which accumulated since June 3, 1987 400,000
Common properties of the spouses used by the
family since Mr. A's death 230,000

Compute the gross estate of Mr. A.


a. P 1,700,000 c. PZ,330,000
b. P 1,930,000 d. P3,700,000
8.
5. Mr. Filan, a bachelor, died leaving the following properties:

Proceeds of group insurance P 150,000


House and lot 1,000,000
Car, registered in his name 400,000
Original investment in a business partnership 200,000

~r.bFilafn_owns 50% interest in the profit of the business partnership with 9.


h is oy nend. The partnership had undistributed profits of P100 000 at the
time of Filan's death. '

Compute the gross estate.


a. P 1,650,000 C. p 1,750,000
b. P 1,700,000 d. P 1,800,000

6. Mrs.,tEdn~iquhez, a glovernment employee, died in a car accident which


resu e m t e tota destruction of th . , .
e1r 1am11y car.

528


r 13-B- Estate Tax: Gross Estate of M .
c~apte arned Decedents
she left the following properties:

eceivable fro_m the car insurance compan


:enefits (receivables) fr_o m GSIS Y P 800,000
ily home, bought usmg Mr. Enriquez' . 1,000,000
farn h . s sa1anes 2,000,000
Jeepney, boug t _us mg ~rs. Enriquez's salaries
Value of the car 1mmed1ately before the accident 700,000
Clothing, shoes, and apparels of Mr. Enriquez 800,000
40,000
Clothing, s~oes, a n_d apparels of Mrs. Enriquez
60,000
Wedding gift, rece ived by the spouses on April 1, 1990
120,000
cornpute the gross estate of Mrs. Enriquez.
a. p 3,620,QQQ C. p 3,680,QQQ
b. p 3,660,000 d. P 4,620,000

7. Mr· X died on November 1' 2020 · He Je ft t h e following


• properties to his
wife:
Land, as birthday gift to Mrs. X P 2,000,000
car, bought from Mr. X's compensation income 1,000,000
Family home 4,000,000
Properties for exclusive use of either spouse 120,000
Assume Mr. and Mrs. X were under the conjugal partnership of gains.
Compute Mr. X's gross estate.
a. PS,000,000 c. P 5,120,000
b. P6,000,000 d. P 7,120,000

8. On June 4, 2020, Mr. Navarro died after 28 years of happy marriage. Mr. and
Mrs. Navarro initia lly started respectively with P2,000,000 and Pl,000,000
properties. Their fruitful marriage accumulated additional PS0,000,000
properties for their twelve children.

Compute Mr. Navarro's gross estate.


a. P 82,000,000 C. P 80,000,000
b. P 81,000,000 d. P 79,000,000

9. Mrs. Vincent died. The properties of the spouses at the time of death were
compiled as fo llows:
Properties of Mr. Vincent before marriage P 2,000,000
Properties of Mrs. Vince nt before marriage 4,000,000
Properties acquired by Mr. and Mrs. Vincent during
2,000,000
marriage from their joint ind us try

Properties donated to Mr. Vincent during marriage 1,000,000


200,000
Income of donated prope rties

529
. Gross Estate of Married Decedents
Chapter 13-8 - Estate Tax.

iously married. His first wife died leav1·h


Mr. Vincent was Prev ,1
descendant. g ~o

Assuming the absolute community of property, compute the gross estat


P 6 000 000 c. P 8,000,000 e.
: ~ P7,200,000 2
c. p 9 , 00,000

10. Mr. Dino Saur died on May 2, 2020. He wa~ survived by his Wife and fo
children. An inventory of the family properties as of the date of his deathUr
IS
as follows:

Lot 1, birthday gift to Mrs. X on June 7, 1987 P 4,000,000


Lot 2, with a small building 800,000
Family home 8,000,000
12,000,000 13- ·
Business interests

Before his death, Mr. Saur transferred an inheritance he received during


marriage to his first born son for PlM. The property had a value of P2M at
that time. Mr. Saur indicated that the transfer was revocable. He, however
failed to revoke the same at the time of his death. The property had a valu;
of P4M at the time of his death.

Mr. and Mrs. Saur were married on February 14, 1988 w ithout a pre-nuptial
agreement.

Compute the gross estate of Mr. Saur.


a. p 20,800,000 C. P 23,800,00 0
b. P 22,800,000 d. P 27,80 0,000

11. Mrs. Henlin died leaving the following properties to her husband:
14. I
Commercial lot, purchased with Mrs. Henlin's salaries P 4,000,000
Residential lot, donated to Mr. Henlin on Janu ary 12, 2007 600,000
Family home A, bough t by Mr. He nlin fro m his salaries
during marriage 3,000,000
Family home B, inherited by Mr. Henlin on July 4, 2002 2,000,000
Mr. and Mrs. Henlin got married on January 1, 200 4 witho ut a prenuptial
agreement.

Compute Mrs. Henlin's gross estate.


a. p 6,000,000 C. p 9,000,000
b. P 7,000,000 d. P 9,600,000

12. ~rs. Chipboy died. Mr. and Mrs. Chip boy had the following properties at the
time of her death:

530
ter 13-B - Estate Tax: Gross Estate Of M .
chaP arned Decedents

properties accumulated in a prior


1. of .
Mr. Chip boy marriage
P 12,000,000
2_ Income ac~um~lat~d from properties in No. 1 1,500,000
3_ Salary s~vm_gsh u_nng the present marriage 6,000,000
4_ Properties, m er~ted by Mrs. Chipboy during
the present marriage
2,000,000
s. Income of inheritance
1,000,000
1t
is Assuming the spouses agreed to a conjugal partnership of gains compute
the gross estate of Mrs. Chip boy. '
a. P 10,500,000 c. P 8,500,000
b. P 9,500,000 d. P 7,500,000

13. Mr. Malinao died. An inventory of the family properties is shown below:
Commercial building, inherited by Mr. Malinao
g during marriage Pl2,000,000
t Cash from rental income of commercial building 1,000,000
Family house, from salaries of Mr. and Mrs. Malinao 1,500,000
Lot where th e home stands, earn ed from buildi ng rentals 500,000
Investments in b onds, from Mrs. Malinao's salaries 2,000,000
Interest income o n investment in bonds 150,000

Mr. and Mrs. Ma linao stipulated the absolute communi ty of property as


their prope rty r egime.
Compute the commo n pro per ty to be reflected in gross estate.
a. P 3,650,000 c. P 15,150,000
b. PS,150,000 d. P 17,150,000

14. Mr. Liwanag died leaving the following properties to his wife:

Common stocks, a t acq uis ition costs P 1,000,000


50,000
Cash dividend incom e
4,000,000
House and lo t*
1,000,000
Agricultural la nd
800,000
Other personal pro perties

Additiona l informatio n : · d M
1. The ho use a nd lo t w e re given by the ch'.ldren as a gift to Mr. an rs.
. d . th ir sil ver wedding anniversary.
L1wanag un ng e db M Liwanag from the proceeds
2. The co mmo n s tocks were purchase Y r. .
. . . h uir ed before the marriage.
of his mhe nta nce e acq d b . h grandfather of Mrs. Liwanag.
3 Th

. I 11 d w as do nate y t e
e agn cu t u ra a n . f Pl 200 000 in the Philip pine Stock
4. The s tocks ha d a fair value O '. '
Excha nge a t th e date of dea th of Mr. Liwanag.
531
Estate of Married Deccdr.:nt:;
,,.
Tax . Gross
tate ·
Ch ap ter 13-B- E5 d r the conjugal partncr:;hfr, <Jf ,, ••
were un e ~,,11n11
Mr. and Mrs. uwanag .
te of Mr. Liwanag.
Compute the gross esta c. p 6,050,000
a. p s,200,000 d. p 6,200,000
b. p 5,250,000
. . the following properties to Mr. Yong:
15. Mrs. Yong died leaving
.ft d by a friend during marriage
p 200,000
Jewelry of Mrs. Yong'. g~ e·ted before marriage
400.000
Jewelry of Mr. Yong, 10 ~n for persona l use of Mr. Yong
Other exclusive propert~es for personal use of Mrs. Yong so,ooo
Other exclusive properties 70,000
2,000,000
Family home
Other family properties 1,200,000

e gross estate of Mrs. Yong under the absolute commun·


Compu t e th 11y 01
property.
a. p 3,470,000 c. p 3,870,000
b. p 3,650,000 d. P 3,920,000

16. Mr. y died leaving the following properties to his family:

Commercial lot, purchased from Mr. Y's GSlS benefit" P 1,000,000


Agricultural land, designated by Mr. Y's father to be
transmitted to a daughter 800,000
Proceeds of insurance policy
irrevocably designated to Mrs. Y 2,000,000
House and lot 1,800,000
Car, registered in the name of his d a ughter 400,000

:ss~~i~i~h; agsolute community of property, compute the gross estate.


• , , 0 C. P 4,800,000
b. p 3,800,000 d. p 5,200,000

17. Mr. Shin, a married .


properties: non-res ide nt alien, di ed leaving the following
• P800,000 car in the Phi!i in . .
• P4,000 000 stocks . PP es, inherite d by Mrs. Sh in d uring marriage
• P2,ooo:ooo land an'~~es_~;ent in th e Philippines, inherited by Mr. Shin
• P2,000,000 business u_, mg in Japan, from salaries of both spouses
investments in the Ph ·1·mt:rest in Hongkong, from fruits of stock
1 1PPmes

Compute Mr Shin'
a. p a · s gross estate.
b. P 800,000 c. p 4,000,000
d. p 6,800,000

532

Didt&t&&
~ 13_8- Estate Tax: Gross Estate of Married Decedents
cnaPter
wnat is Mr. Shin's gross estate assuming that the reciprocity rule applies?
19• pO C. p 4,QQQ,QQQ
ll• p so0,000 d. P 4,800,000
b,
Mr· An~ersen, an American residing in Hawaii, died leaving the following
19, properties: ·

.... Andersen's separate property in America 12,000,000


JVlr•
Mr, Andersen's separate property in Hawaii 8,000,000
Mrs. Andersen's separate property in the Philippines 2,000,000
common property of Mr. and Mrs. Andersen
in the Philippines 7,000,000
common properties of Mr. and Mrs. Andersen
in America 24,000,000

Compute the gross estate.


a. p 0 C. P 9,QQQ,QQQ
b. p 7,000,000 d. P 51,000,000

zo. In the immediately preceding problem, compute the gross estate if Mr.
Andersen were a resident alien.
a. PO c. P 9,000,000
b. P 7,000,000 d. P 51,000,000
,.
Chapter 14 - Estate Tax: Deductions from Gross Estate

CHAPTER 14
Art
Chapter Overview and Objectives
------------------------------------------- --- -- -
--------------------------------
After this chapter, readers are expected to comprehend: ···,,
---~-
1. The general princi ples of deductions against gross estate
2. The classification and deduction rules on deductions
3. The list and limits of items of ordinary deductions
4. The computational procedures of the share of th e sunl i\'i ng p
5. The list and Ii m its of s pecia I deducti ons • ouse
6. The deductible expense of nor1 -residc nt c1licr1 decede r1ts
•'

DEDUCTIONS FROM GROSS ESTATE


There
th h ·are charges whi ch naturally diminish the ,1r11otint ot· the h
· in t•rt a
e e1rs. Hence, the law allo1;vs dcd t1<..-t1ons fron, grr> • l~~t ilc 1 1 nee ·
these _charges, the law also a llo, cen.1111 d ·d ucl1on . 'rn .
1ncent1ves from gross estate.
d1 ion •
t ' nature ·
i":it •

Presentation of Dedu ctior1 s in th e F.~c.,tt~ 'J'.1 Rc t\~r,,

( lltlj U~JI/

GR OSS ESTATE
l vu 1wl111al
p
_ l ulaf -
Less: Ordir,nry Deduct.Ion~
Clai n1s clg.tin:.l the t.'S(.l ll.:
Cl.11n1s c.1ga111!->t 1n,olv1.•n c pcr,un
lJnp.1id n1or1g,,gl's
'<. \
~ropl'l~ly prl'Vlllllsly l,ll(l'tl X\ :'I: X
1 r.1ns lt.'r 101 publ1t. ust' '\X ~
Others
X..\:S,X.
Estate Lifter dt•<ltit. uon l' '<Xl\ 11. p p
Lc~s: Special De,luctions
l·a111ily ho1uc
)( :( '(
Stand.1rd deduction
Others X l'.X~

Net !:~late ~,. _g


Less: Sil are of Che survi . p \ '(,!I.\:( ...
I
NE1' 'l'AXAl3LE ES'l'ATE v1119 spou e 1/. \.\:\' A
e ~l,\~

Not e:
~: For a single dert>de nt th
The "Co111u9u l/L'on1 • e rolu1nn co1nn1on
munur roluntn t~ lt'rt bl prkopt>tttt":s w1II b left bl u1k
Jll It th~ d
t'Ct:Ut"(\( ·~ lll!,1~

534
chapter 14 - Estate Tax: Deductions from Gross Estate

SSIFICATION OF DEDUCTIONS
cLAordinary Dedu~tions
,A. special Deductions
9· Share of the surviving spouse
c.
ordinary ~eductions conceptually ~nclude items which diminish the amount of
h
the inheritance,: T _e ~nly ex_cept1on here is the deduction for "Property
revious/y taxed which Is a tax Incentive but is classified as ordinary deductions
fn pursuant to the estate tax form.

special deductions are items which do not reduce the inheritance but are
nonetheless allowed by the law as incentive deductions against gross estate in
the determ ination of the net taxable estate.

Share of the surviving spouse pertains to the interest of the surviving spouse
in the net conjugal or communal properties of the spouses. This portion is not
owned by the decedent and will not be transmitted by the decedent as part of
the inheritance; hence, it must be removed in the taxable estate.
of
to GENERAL PRINCIPLES OF ESTATE DEDUCTIONS
of
t. The substantiation rule
As a rule, items of deduction must be supported with documentary
evidence such as receipts, invoices, contracts, and other proofs that they
actually exist or occurred to establish their validity.

2. Matching principle
As a rule, items of deduction must pertain to properties that are part of
the gross estate. They must be proper charges thereto.
Examples:
a. Obligations of the exclusive properties of the surviving spouse cannot
be claimed as deductions because said properties are not included in
gross estate.
b. Losses of properties before the death of the taxpayer are not deductible
because the properties are no longer part of the gross estate of the
decedent at the date of death.
c. Separate obligations or losses of exclusive properties of the surviving
spouse cannot be deducted against the gross estate.
3, "No double classification" rule
Items of deduction cannot be claimed simultaneously under several
deduction categories.
Examples: .
a. A family home which is destroyed by any casualty during the
settlement of the estate cannot be simultaneously deducted as a "family
home" and a "casualty loss."

535
• •' • . ~ I
-,! .,
' ...

. .

Chapter 14- Estate ►T-ax:, Deductions frrom .G11oss Estale


• ·~
·~. '
,;
_- .
• I I

? '.
.'

-·~
b. Losses claimed in the income t:ax'return ·af the estate <Za'nnot-be tla·
again as deduction in the estate tax return. .,,....,.... · ~ llll~
,, !'
. '. "'
(.WF,,
4. Default presumption on ordinary deduction
. t;h ~fuh~'1.'t 1,
"', l
tl
.

In the case of married decedents, ordinary deductions are presumed t


against the common properties unless proven to be an exclusive Prope~he
either spouse. This is in line with the rule that properties are conini Of
properties unless proven to be exclusive. on
ORDINARY DEDUCTIONS
Under current usage, the following are deemed ordinary deductions:
1. Losses, Indebtedness and Taxes {Ll'TJ
2. Transfer for Public Use
3. Vanishing Deductions

LOSSES, INDEBTEDNESS AND TAXES (Un


Losses
These pertain to losses of properties of the estate during the settlement of
the estate. These may arise from casualty such as fire , storms. ship\vreck,
robbery, theft or embezzlement when sucl1 losses are not compensated for
by insurance.

Date of death l>t•,1d ll11 • of return (lluS1


I ___ I
I --:,-----..........-,,,,.;;,,.,~) just I
Non-deductible here! , Ded11ctlble If Jo e occt,rrt.•d here , ibl Ml'N
ro11o,
'¥ ----------------- ----- -- ~
( 1 }'l'.lf) 2019
Points to Remember: l. ,i
1. Loss must be a st1stai r1ed cast1alty loss. P1
2. The loss must occur during the :scrtler11er1t t>l Ll1 • , t,1tr. tip ttl the J!
deadline of the estate tax retL1rn.
3. The loss must not be conct1rrcntly clai,11(•(1 i11 tl1i~ 1r11,, >n1 • t, x rt tt1111. 21
Illus tration 1 I
Mr. Y died in a fa tal car cra~ h on Noventl> ·1 2• .!() l c1 1'h
properti es were identified by hi s e<.t,1tc .1cln1 ir1islrc1tr11 •
follo\111ng IL> c ol
'
t
r
Losses up to tl1e po111t cJJ cleatJ1·
Valtte of car totally destroyed Llur1r1g the Cl'd~l1 r> 1,_00.000
J)ilferdge lc)ss l) ll n1erchJr1ll1se re\' t'.tleti h}'
tl1e pl1vs1cal i11,,e11tc)ry count 011 Octobt•r '1 . ~{l l c 00.000
i ,().',St:~ SIIJ(t' l)}t! ciec1t}1 OJ lilt! tlt!(t;'(/t'f/{ "
F1 1t' It)~~ l>r1 .in 1n~ured bttiltilr1g t)ll l)~l l:111l>cr lS 20 1l l . ()00 ooo
'l'lit>tl t>t l)t•r:,011..11 \ dluc1blt'S ot ~Ir ) 011 la11udr1 l 10- U 180 Olll

536
14 _ Estate Tax: Deductions from Gross Estate
chapter
Of
cash robbed from Mr. Y's residence on February 14 2020 620,000
va 111e fa n uninsured car destroyed by a storm on March 1 '2021 800,000
yal 11 ~~ loa,1s receivable from a bankrupt customer ' 100,000
unptll
ducti ble loss shall be:
fhe de
n theft of personal valuables p 180,000
Loss o
on robbery 620,000
Loss, deductible loss p 800.000
rota

A loss incurredb e fore or at t he date of death is non-deductible.


Note:
1
· • The value of the car destroyed on the crash is non-deductible because the
car will no longer be included in the gross estate of the decedent.
• If the property is compensated for by insurance, the proceeds of insurance
is included in gross estate, but still no loss deduction is allowed because the
same does not affect the hereditary estate.
• The pilferage loss on inventory will not be deductible since the inventories
were lost before death and are not part of the gross estate.
z. The fire loss on the building is not deductible because it is insured. The loss is
not actual but temporary which will be recovered by the estate.
3. The loss on an uninsured car caused by a storm is non-deductible as it is
beyond 1 year from the date of death.
4. The unpaid loan receivable from a bankrupt customer is a deductible loss but
under a separate category "Claim against insolvent person."

Illustration 2
Just before filing the return in June 15, 2020, the estate administrator noted the
following losses in the estate of Mr. Wong, a businessman who died June 30,
2019:
1. The $100,000 in Mr. Wong's savings account. He purchased these dollars at
PS4/$. The Peso is trading P53/$ on June 30, 2019 and P52/share on June
15, 2020.
2. Mr. Wong had an office equipment with book value of P400,000 on June 30,
2019. The executor sold this for P350,000 on March 10, 2020 to settle claims
against the estate.
3. Mr. Wong's vault containing P300,000 inventories of precious metals was
stolen on August 15, 2019. This was claimed as deduction in the income tax
return of the estate for 2019.

None of these losses is deductible.


The $100,000 cash shall be included in gross estate at PS3/$. The decrease in
~he value of the $100,000 cash from PS3/$ to P 52/$ is not deductible since this
is an unrealized market loss. The loss in market value from PS4/$ to PS3/$ shall
not be deducted because the same is already deducted in the amount of gross
estate.

537

-------- ---
. f m Gross Estate
Chapter 14 - Estate Tax: Deductions ro
. loss of pS0,000 during the settlement of
The office equipment l1ad a realized -deductible. the
state but tl1is is 11ot a casualty loss; hence, non
. ualty Joss during the settlement of Cl
The P300,000 theft loss is a rea lized ca~. ary Joss in th e income tax retu e
th j,,C
estate but the same is claimed as an or in rn Of or
the estate. r•
pl
Claims against insolvent persons_ "of loss but is presented
· · · 1 t persons 1s a ,orm as
Cla ims against inso ven The deductible amount of l . a IJI
separate item of deduction in the tax return. t f 1. c a1in fl
. h verable amoun o c aim.
against insolvent persons 1st e unreco cc
Illustration 1 OOO f om Mr Kumag Th I l·
00
Mr. Kugar died with a total receivable of PZ ' r · · e atter
was adjudged bankrupt by the court with only PSOO,OOO total assets but With
PZ,000,000 in total liabilities.

Mr. Kugar would be expected to recover only PZ00,000/ PZ,OOO,OOO x P800,ooo


or P80,000 from Mr. Kumag. The claim from insolvent person shall be P200,ooo 4
- P80,000 = P120,000.
Assuming that there is zero recovery, the entire amou nt of claim shall be 'f
presented as a deduction. Either way, the PZ00,000 claim m ust be included in
gross estate.
R
Illustration 2 T
Mrs. Shelly died leaving a P 500,000 promissory note from Dye Company a F
bankrupt company undergoing liquidation. The note was secured by a small 1
piece of land with current value of P300,000. The fiduciary of Dye Company
estimates a 40% recovery for unsecured creditors.

Mrs. Shelly also loaned Dye Company P 20,000 in a written instrument which
prescribed a few years prior to her death.

The claim against insolvent persons shall be computed as: (


,
Recoverable amount P 380,000 (
Less: Total claim 500,000 •(
Claim against insolvent person P 120,000
The recoverable amount is computed as:

Total claim p 500,000


Less: Fair value of collateral
300 ,000 p 300,000
Unsecured portion p 200,000
Multiply by: Recovery r atio
Recover able amount 40o/q 80,000
p 38Q.OOO

538
-- -------
_ Estate Tax: Deductior1s from Gro'-s [ t t
ter 14 ., •s a e
cnaP
p20 000 \' ,JtV<.'d loan \ h,c h pr '''- 11 I 1 11
;t,r ' lf lI < lt•c ,., 01 il <l,11n1 .1~.11114,f 111.,olvrnt JJerson
110 tt • no longer ,lll '-'' 0 1tr,11 e , 1ghl ,11 tlii· iJ<>11Jt of , 1. ~
~1 11,; (, I I. 11
"'• l

·n
Ii I I '
~rion of l.ossr
(13:t , , ,n< f11cf111g cl,11111~ ,1~c11n \ t rn,<1lv<'11t IJP1 «.one;, sh,111 IJr r l<c1<.c,ific•cf t>ac;cd
• p1 , ,c.,, tv c/a r;\1/1c a t10,1 R / " ·1·1 I ·
1
, , '"
th<' < 1 • . '' e. 1<' o~ c1t ~c11,1rc1tc prt>pC'rty 1s
trd J\ a c
i rd11 < t,orl ag,i,,ist se11c1r,1tt' pro11er ty. 'fl1e lc,ss of corr1mon
'~ ._
1
111
~',, rerl) rs 11rcsented a c1 clccluct,on agt1,nc,t co,nnl<>ll property.

,ration
11 Ill f M X
fh<' e tale o ~ ~s. · •1 niarr,cd clccc~Jent under the reg1n1c of absolute
, ,,,,unrl)' of Jll 011~ rty, sufferccl th e f~llow,ng lcJsscs:
00
p~OO 000 c,1sh f, om the family business were robbed from Mrs. X during the
1
· night of her n1urder.
z. A car \\'1th PB00,000 value which Mrs. X inherited during the marriage from
hr~ grandrr1other was stolen at the time of her wake.
3_ A (e,v n1onths after Mrs. X' death, the P2,000,000 house which was inherited
b) ~ir. X JUSt before their marriage was totally guttered by a fire
4. p 300,000 worth of personal belongings of Mr. X were destroyed by the fire.

The foregoing shall be reported in the estate tax return of Mrs. X as follows:

Exclusive Communal Total


Robbery loss p 500,000 p
500,000
Theft loss p 800,000 800,000
Fire loss - house 2,000,000 2,000,000
Total p BQQ,000 p 2,SQQ,QQO P 3,3QQ,QQO

Note:
The P300,000 fire loss on personal belongs of X is non-deductible since these are separate
properties of the surviving spouse and are not part of the gross estate. The house is communal
property since the same is inherited before the marriage.

Claims against the estate (Indebtedness)


The word "claims" as used in the statute is generally construed to mean
debts or demands of a pecuniary nature which could have been enforced
against the deceased in his lifetime and could have been reduced to simple
money judgments {RAMO 1-80).
Claims against the estate or indebtedness with respect to property may
arise out of contract, tort, or operations of law.
Unpaid mortgages are claims against the estate but are separately reported
under the category "Unpaid mortgage" in the estate tax return.

539
. .
_,. --. - ----- . .-
~ - -- . ,• Ii',,· ~ - •~ • L • r/ >rs•,";'-;;:'•:'·,,' ••·• ,'Ir
,!' ~ • l ' 'f- ~·"' !~ ...,-~,.., 1~-l : . . f
..;.~-.t '·•
' ... •. . . f:' '"'-,~~!\--st'· . ' ,
• .t...-.t;,'\....... ~- ,•'.i•"'••f~. ..,(.,- •

Chapter 14 - Estate Tax: 1@educt1ons ; ,, ·''. · .....-:. t<·,:. ·::JJ/i'f;·. ~l


gainst the estate:· A .. -~-........~M~~:.:': · '· :;:~i··~t~t: 1

Requisites of deductibility of cIaims a f h d · d ~ -1~.. · ~ 'f· ~rr•;
°
1. The liability represents a personal 0 ?ligation t e ecease existing ;a f th. :1~ i .',:if.
time of his death except unpaid medical expenses . ,..,. t e · :: •, ';~:
2. The liability was contracted in good fai th a nd for . adequate
1

fu)J ,~;J • ·i.~· ~t!


consideration in money or money's w orth i , . .· . ·, w:' :
3. The claim must be a debt or claim \Yhich is valid in law a nd enforceable i t~~;
court; , · ' d b· h n ~~11
4. The indebtedness must not have been coo d one Y ~ e creditor or the P'
action to collect from the decedent must not have prescribed.
d'J'
Classification Rules for Claims against the Estate
1. Family benefit rule
If the obligation was contracte d or incurred fo r t h e ben e fit of the farnn
t h e claim shall be classified as deduction again st co~mon prope~'.
Otherwise, the property classification r ule s ha ll be applied.
Examples:
a. A mortgage which was contracted for the education of the children of
the spouses shall be deducted against common properties even if the
same is constituted against a separate property of either spouse.
b. An unpaid real property tax on the family home shall be deducted
against common property even if the family home is a separate ,
property of either spouse. II
c. Obligations constituted for the medication or other support expenses of I
I
I
any family member shall be considered deductions against common I
properties. II
I
I
2. Property classification rule I
\
Claims follow the classification of the relevant property.
Examples:
a. A mo_rtgage or unpaid _t~xes on property inhe rited or acquired before
marriage shall be_class1f1ed following the classification of the property
based ~n t~e app~1~able family regime of th e spouses.
b. An obl~gat1on ar1s1ng from exclusive property shall be considered as
deduction. fromf exclusive
. . accrued or was used for
prop ert·ies un 1ess it
th e bene fit o the family.

Special rules on certain claims agai·nst th e estate


1. Unpaid mortgage I
I
I
This includes mortgage upon . I
I
property whe re the ' or a ny indebted ness, w ith res pect to I
I
1
t1ndiminished by such va ue of th e dece dent's inte rest therein,
I
I
I
I
estate. m ortgage or indebtedness, is includ ed in gross I
I
I
I

540

L
14 - Estate Tax: Deductions from Gross Estate
cnaPter
· n1
111usrrat10 .
A decedent had a fami ly home wo rth Pl,500,000 which was encumbered by
am O rtgage. Details about the mortgage were as follows·.
Mort~age A
original amount P 900,000
Less:
paid before death p 200,000
paid after death 400,000
present balance p 300.000
The family home is a common property of the decedent and his spouse. The
proceeds of the mortgage were used for the family.
A deductible mortgage, just like other obligations, must have been inct1rred
before death and remain unpaid at the point of death. Hence, the allowable
deduction for "Unpaid mortgage" shall be the balance of the mortgage at cite
point of death :
Mort~a~e A
Original amount P 900,000
Less: Paid before death 200,000
Balance at the date of death _ Jllil.OQQ

Presentation in the t.ax return:


,----- --------------------------------------------------------------------------,I
,'
'
t:3,, lu:.11't: l:v11111zun I
/ Gross estate
I
P · P 1,.'i00,000 i
I

:I De d uct1ons:
. :I
/ Unpaid ,nortga_qe P • /' 700.000 f
L------------------------------------•--------------------•-------------•-------4
Note:
1. The valul' of tht• propt•rtv 11nd11111n1,ht·d hy tht.· mort.:agt• 1,; tnt.lu,lt•d 1n ~rn c (.It·.
2. Only 11101 lg,1~e, \\'hlrh \\'t'r,· run:.tllull·d durttit,: the: l1lt·c11nc u l ch,· le, t."\lt•nr \,·hie h
ren1l1i n u11p.11d ,ILtht• t1111t• 01h1, tft•.1th .irt.• tlt•dultJble.

Illustration 2
During the n1arriage. Nl r. Y 1nt1t•r1recf. ro111rr1t'rl 1,1I lt>t ,,·1th ,1it,r1,1I v,1luc ,,t
P4,000,000. When one c>f h1, rhtltlrl'n got ,1tk. ht• n1t>rt~.1gt•cl tht.· prur>erty
for P2,000,000. tir \V,ts ,1blt• tt> ,,,iv P l U(l.000 unr1l h1'i dt•.1th.

Presentation in the t.1x rt.•turn·


:·------------------------------------------------------------------------------~
! E,lcl,1s1te_ _!&!Jl'll£J...._ l
: Gross estate P 4,000,000 I
: :
:I Deductio11 •• ,
I

l Unpaid 111urc_qa_qe P · P 1,600.0()0 i


~------
-----------•-------•-•----w-•---•-••-•-••-••-••--------------- ____ ,. -------· f

Note: l'he n1ortgagt' shall bt> prt>~ente<l under comnion prop ny b~i.: u~<-• th pro t't'd o f
lhl· \.t01t' 1-. U~t' tor tht' be11et1t ut Cht' r..u111lv

541


. ns from Gross Estate
Chapter 14 - Estate Tax: Deductio

2. Unpaid taxes income tax, business tax, and property


Tl1is inclt1des taxes such ash death of the decedent and Which ta~
which have accrued as of t e Were
unpaid as of the time of dea th. . .
. hat on]y obligations ex1st1ng at the Poin
It must be emphaSIZed t . . including taxes which are t Of
death are deductible. Obl1gat!ons fter death are n ot deductibl sefttled
before death and those accruing a e roni
gross estate.
Hence, the fol lowing taxes are non-deductible:
a. Tax on income earned after dea th
b. Property taxes accruing after dea th
c. Business taxes accruing after dea th .
d. Estate tax on the transmission of the estate to the heirs

It must be noted also that as contemplated in RRZ-Z003, "Claims against


the estate" are restricted to private c]aims enforceable against the
decedent's estate.
Although taxes are claims against the estate, taxes should be reported
under a separate category, but since there is no separate category for
taxes in the estate tax return, the same shall properly be included under
the category "Others."

3. Accommodation loan
An accommodation loan is one contracted by a person in behalf of
,
another person with the contracting person merely representing in
behalf of the other person who will be the beneficiary of the loan
proceeds.
j
Accommodation loan are presented as a receivable in tl1e gross estate I
I
I
and is presented as a deduction. However, if th ere is a legal impediment I
I
I
to recognize the same as a receivable, it may not be included in the gross I
I

estate. Likewise, it will not be presented as an obligation. '


t
Illustration 1: Claim against the estate - unmarried decedent J
The heirs identified the following obligations of Mr. Natoy, a bachelor, who died
on September 1, 2019:
Personal loan condoned by the creditor p
Balance on the purchase price of a car, paid b the he ·r 400,000
on September 28, 2019 Y 1 s

Prescribed promissory note 200,000 4


Bank loan 100,000
Interest on bank loan, P30,000 accrued as of 300,000
September 1, 2019
s
50,000
542
r 14 - Estat e Tax: Deduction s from Gross Estat e
chapte
ductible "clr1in1s a,qai11st t/1e est<ite" sJiall be:
r11c de
. ·d bt1lance o n purcl1ased car at poiJlt of deJtli p 200,()00
(fJl p,l I
30(),() 00
111k /oJll ·
H, • , 1 payalJlc accru ing as of date of death 30,000
11 Ie1 t • . .
I dcd ucliblc c1a11n s aga inst tl1c eslale
rotil 1 5.3Q..UOO
sentation in the estate tax retu rn :
/,re.. ----- ---------------------- ----------
,---- --- --------------------------------------,I
; E'xclu.sive Common :
; C/oirns agai11st t/1e estate p 530,000 p o i
''
.------- ---- ------------------------ -------- --------------------------------- -------'
,11ustration 2: Claim agai~st the estate-married decedent
The executor of Mr. X compiled the fo llowi ng obligations:
Obligation of the exclusive properties of Mrs. x P 500,000
Unpaid funeral expense 100,000
Unpaid medical expense 200,000
Obligations accrui ng after death 150,000
Obligations of family before decedent's death 300,000
Obligations of the separate properties of Mr. X 600,000
Unpaid mortgage on family properties 1,000,000

The deductible "claims against the estate" shall be:


Obligations of family before decedent's death P 300,000
Obligations of the separate property of the decedent 600,0 00
Total deductible claims agai nst the estate P 900.000

Presentation in the estate tax return:


r·----------------------------------------------------------------------------------
1 I
: Exclusive Common :I
/ Claims against the estate P 600,000 P 300,000 :
I I
I I

~----------------------------------------------------------------------------------~
Note:
1. Obligations are pres umed common unless established as exclusive. Obligations of t he
surviving spouse are not deducti ble because exclusive properties of the surviving s pouse
are not included in gross estate. (Matching rule)
2. Unpaid funeral expenses are non-deductible because they are debts accruing a fter death.
3. Unpaid medical expenses are likewise non-deductible even if they are in curred prior to
death because Congress in creased the standard deductions from PlM in the NIRC to PSM
under the TRAIN law in lieu of deductions for funeral, judicial and medical expenses.
4· Obligations accruing after death are obligations of the heirs and are not obligations of the
decedent. They are deductible against the share of the heirs in the hereditary estate and
are not deductible from the amount of the hereditary estate. Hence, these are not
deductible.
5
· An unpaid mortgage is presented under a separate classification "Unpaid mortgage".

543


I 1'ro, tl Gross Estate
Chapter 14 - Estat \ TJ ; o,,,i t u;t tlll!•

• , ,,1,,,.-.· ,,11 ,,1,,st tl,e Estate


S11bstantiatfo11 Rt•t111fr•,,,,,,,,,,.-. <Jll ·1
1

1. Sin,ple lc)an :1n<I .1clv.1nr1·~ . ,, 1 lu· <luly n<>l:tri1.eci at the time ~he hidl'btl•(I
11111
a 'fhe debt in:-tr11111t•11 t •' , ·tiry r1t>I C or co11tract t>f loa ex . 11 l•)l~
. I1 ' I ' I p1'1Hll 1:,;:,; . ' ' 11 ( ('Pt I
I •
was incurrt'<l. su r ·:,; • 1 1 t l,i t1 s where notar1 zat1011 ts 11ot P'll'L 1,,,
loc111s grantl'<l I >y I'111 ·•1111·1
· ·ti .lust
· , 1."1 1 ,ncla . l i11stitt1t1on-
. Ie11d er Ill II
l
10
. I 1
,llry 111 t 11 ' •
bt1si11ess pract,cl' JH • 1 f · Ill the crccli to r as to 1e unpaicl b·,t·i
t1
b. A duly notari1..t•1 I Cert 111rat o1~ ', _tt,11, l'hnc of death. , , llto O(
i I11 t 1·· r 1,.;t •1s (l t
the (lebt. inr h1< Ing , • ·r· , rrt'tl lt<J r to lend tJ1e amount at the tin, ,
O 1 1
c. Proof of t1na11ci:1I l'ap:ir it y ~ti it ·•s t ;1ttclited bala11ce sl1eet witl1 il dct~ 111t•
1oan was gra11t ecf , •·ts, Wt' II '·1.s ·I1tswing t . •
tlic unpaid balance ot the dccc
. ~ ,1 1lc11
schedule of its rec:civab c ~ <> 1 1 <1unt.

debtor 1 , . , ·utccJ 11y the adn1i11istrator or executor of ll


d A staten1ent t111cier ont cxc< ·
1 d f l I 'f • le
· estate reflecting
. the d'. 1~1> 0~·it·ion t)f l'llC procee s o t 1ef thoar1d 1 sa1cl loan wUs
. . t 111.t!C ('•3) yu•irs
contracted w1tJ1111 .... prior to the deat11 o
r ·
e ecedent;
2. Purchase of goods or servi ces f d .
a. Pertinent doct1n1e11ts evic.icncing tlie pttrcliase O goo s or service_s as duly
ac know Ie dged, exe Ct lted , ancl sigriecl by tJ1e decedent a11d tl1e creditor, sue!,
.
as: .
• Sale of goods - sales invoice/delivery ~ece1pt
• Sale of services - contract for tl1e servi ces agreed to be re11dered
b. Statenierit of accotint given by the creditor as dt1ly received by the
decede11t-debtor
c. Duly notarized Certification from the cred itor as to tl1e t111p aid balance of
the debt i11cludi11g interest as of the tirne of death
d. Certified true copy of tl1e latest at1d ited bala11ce sl1eet of the creditor with a
detailed schedt1le of its receivable s howi11g tl1e t111paid bala11ce of the
decedent-debtor

3. Where the settlement is made througl1 the Court in a testate or intestate


proceeding, pertinent doctrments filed witl1 the Court evidenci11g the claims
against the estate, and tl1e Court Order approvi11g tl1e said cla i111s, if already
issued, in addition to the documents m entio11ed it1 tl1e preceding
paragraphs.

TRANSFER FOR PUBLIC USE


Transfer for public use includes the amou11t of all beqtiests, legacies, devises
or transfer to or for the use of the Governmer1 t of tt 1e Repttblic of the
Philippines, or any political subdivision thereof, for tile exclusive public
purposes. These must be indicated in the decedent's last will and testament.
Illustration
Mr. A devised in his will tl1e following properties:

Commercial la11d, to a public school


Land and building, to a gove rnment-owned and P 2,0 00,000
controlled corporation (GOCC)
Total l.OQO.QQO
£ 5,000,0QQ
544
14 - Estate Tax : DedlJctions fro111 Gross Estate
chapter
Pp ,ooo,ooo
m~st _be included in gross estate. Only the PZ,000,000 can be
f/l . 5d as transfe , fo, public use. GOCCs are corr1r11ercial and are not for public
c/t11ll1e
r11oses.
P''
p£RTY PREVIOUSLY TAXED (VANISHING DEDUCTION)
pRO_ are insta11ces where properti es are tra11sferred between persons in
r helteperiods of t11ne
. . a series
causing . of transfer taxation.
shor
Exan1ple: .
' .he deatl1 of the decedent 1s preceded by a donation inter-vivos
a. 1The deatl1 of the decedent is preceded by a donation mortis causa
b.
case !: Donation before death

A Donates to - - )- B - - )-
1'
donor's tax
,1'
estate tax

case 2: Series of deaths

A ----> B ----> C ---~ D ----> E

t 1'
estate tax
t
estate tax
t
estate tax
t
estate tax
estate tax
Note the series of double transfer taxation in both cases. Due to this, a
deduction for property previously taxed is allowed by the law against gross
estate to mitigate the impact of successive transfer taxation. This deduction
is commonly known as "Vanishing Deduction."

Requisites of vanishing deduction:


1. The present decedent must have died within five (5) years from date of
death of the prior decedent or date of gift.

Donation/Succession
~ - - - - - - - - - 5 years - - - - - - - - - 7 x
Non-claimable here! Claimable if received in this period.

Z. The property with respect to which the deduction is claimed must have
been part of the gross estate situated in the Philippines of the prior
decedent or taxable gift of the donor
In short, the property must have been previously subjected to a transfer tax.

545
from Gross Estate
Chapt er 14 - Esti1t e Tax : DcdLictlons
t irl cntifled as th e same property receiv
3. Tl,<> pro11rrty ,11t1st ,e I, ceived in exchange th ect fro
prior llccc<.i{'ltt or donor or the one e ereof h1
I'f the property transform ed j t
l)edtlction is , till clai n1 alJlc even n anoth° er
k111d t1I 11roJ>erl .

4. 1'he e tate tc1xcs on tl1e tra11 s1nissiondof the . priJr e~tat~dor the donor'
ta or, the gift ,nust have been finally eterm1ne an pa1 . s
Tt1e basis of tl1e vanishing deduction is to_miti~ate the impact of doub1
. . .· not be claimed 1f the donor's tax e
ta ation. v~1111sh1ng dedu ~t1on can or estate
tax \VJS not paid in the prior transfer.
5. No va 11 i, hing deducti on on the property or the property given in
exct1ange thereof was allowed to the prior estate
This rule applies in the case of a series of dea th s. If .th e prior
. estate clairn ect
h.
vanishing deduction the second estate cannot claim vanis ing deduct·
1

because the purpose of vanishing deduction is to mitigate double taxatio~~n

The double deduction with vanishing deduction


The purpose of vanishing deduction is no other than to minimize the burden
of double transfer taxation that could occur when a decedent dies soon after
receiving properties that are previously subjected to transfer tax.

This noble gesture from the government should not be construed as permit
for tax payers to abuse claims of deduction. In principle, vanishing deduction
can be claimed only if there is an incidence of double transfer taxation.

Despite the absence of a rule prohibiting double deduction using vanishing


deduction, there is no good reason to claim vanishing deduction if the entire
value of the property is already claimed under:
a. Casualty losses
b. Trans fer for public purpose
c. Family home

The property is effectively excused from taxation by being deducted under


the afore mentioned categories. There would be no double taxation to occur.
Hence, further claim of vanishing deduction should be disallowed.
IIJustration 1
Mr. A died on June 3, 2019 with the following properties in his gross estate:
Property Mode of acouisition Date of acauisition
Condo unit Purchase ulv 1, 2018
Car Donation July 3, 2016
Residence Purchasei Aueust 5, 2018
Comm ercial building Purchase2 June 1, 2019
Aericultural land Inheritance Anril 1, 2014
546
_ Estate Tax: Deductions from Gross Estate
ter 14
chaP
ote: . rnooeY inherited from his father ~ho died on July 15, 2017
~ using oneY received by way of donation on December 25, 2011
l· sing rn
z. LI
erties received by way of donation and inheritance within 5 years from
011/J pro~! death can be claimed with vanishing deduction.
the date
acquired by way of purchase may qualify only if the money or property used
fh0 5e rchase or exchange came from donation or inheritance received by the
in thde p~ within 5 years from his or her death.
dece en
hat the commercial building was purchased using donated money, but \'vas
Note_t d beyond 5 years from June 3, 2019. The agricultural land ivas also
recei~eted beyond five years from June 3, 2019.
inher1
onfv the car and tl1e residence can be claimed \vith vanishing deducdoris.
Hence, .,

(ltustration 2
Within 5 years prior to the death of Mrs. Y, she received a hou .:ind lot ,
. heritance fron1 her grandparents. The house \vcts tJn1n urecJ "1n<l was cot..illy
:estroyed by a fire before Mrs. Y's death.
Mrs. y also received a villa as int1cr1t.1nce fro n1 h~r r.1tt1cr who e c~tatt• t,
remained unpaid at th e elate of Mr-; Y, tli·.1tJ1 . ~1,, Y .11 1,0 rcrt•,v~<I ~1 P100,000
cash donation fro n1 her 111utl1cr o tlorlof ·~ t.1, \1. . ,~ J>.1itl 0 11 ci1e don,1t1on
because it is tax exc1r1 pl.
The estate of Mrs. Y ca,, ct,,,,n vt1n1.;h1,,.,, tl,·tl11cc,,1,1 ,,,1/v Jvr tl1t· lot c, 1,-on1sh1,1.Q
deduction ca11 be clc1in1l'<I \VtClt r,·,,,c·cc c,, ci1c· ht>tl\t' bc.·t·t1t1,·e ,c ;~- ,,u tonJ,·r f"lrl of
the gross estate J1avi11,q bt:e11 cJcscrvr,· I be-Jut t· ,,,. r·~ dt·,1cl1.
No vanishing de<luction ts c/c1imc1l,lt· /or cht· ~•,lltt a~ cht• eJtt1tt· tt, o/ th, 1>rror
1
decedent is still 11nµ c11<I. Nu vc1n1.\l1t11_q clt'li11tr1 on ,~ ul~v t:lu1rr1c1t,J,· c,, cht· l lOl>.tJOO
cash as the sarne "'''s nc, c prt, v1ut1'i◊' t<J.~t•,J. ·1·h~rt: ,~ ,,u clot1blt: tc1-cac,o,1 ,n tlt1 , c,1 ~-

Procedural Co111puta ti or1 : Va nistting Oettut·tio11

1. Deterrni11e th e 1nit1t1I \1 a/11t·.


The initial vali,e 1s tltt" t.11r 111 .1rkt·t v.,ltit' l>t tt1t' f1r,>pt·r ty tt tt1e d .1te of
the 11rst tr,111s fer ti.e. tl.,1tt• l>t i1r1 t,r tft' ct·tll•t1{'-; lic.,tt1 l>I' t1cttt• c>t g1tt ) or tht:
fair value at tl1e <.J.,1te o t tle«th vvt11cl1e\ t'I t:) lt>\vcr.

lllt1stration
The lullo\-vtng rel,ites to ,1 property th. t \\"3 dun~ttcd to the d \.~den t.

Upon ct_on;it1Qll UpQll ~illh Ql U.t.~~deol


~} l)l)U,000
Zonal value •' 1.~00.uoo
Fair value per assessor 1,100,000 l,000,000

547
Chapter 14 - Est ate Tax: Deductions from Gross Estate

Tl,c re~\>Cctivc fair val tic at tl1ose dates shall be the hi gher:
tt i!
Upon donation h for
1ligl1 cr P 1,200,000 P 1,000,0
dvt
grc
11cncc, tl1c i11itial valt1e sl1all be the lower Pl,000,00 0.
1111.
2. Dctermi11e tl1e initial basis.
~r

Tl1e i11itia l basis is tl1e initial value re?uced by a ny indebtedness


0
J>ropcrty wl1icl1 was assumed an d pa id by the present d ecedent bn the
ca:
l1is or t1er dcatl1. efore
otl
This is computed as: Gr
p xxx,xxx All
Initial value
Less: Indebtedness assumed and paid before death XXX,X2()(
p -
Initial basis xxx,xxx

3. Determine the fina l basis.


The final basis is the initial basis reduced by a proportion of oth rt
ordinary deductions (i.e., LIT + transfers fo r public purpose) which
initial basis bears over the gross estate o f the decedent.
t~: In
LE
This is computed as: In
LE
Initial basis p xxx,xxx
Less:
(Initial basis/Gross Estate) x (Losses, indebtedness, Fi
taxes and transfer for public purpose) xxx,xxx M
Final basis !:::p=~xx~x~,x~xx!2 V

4. Determine the vanishing deduction. T


T
The vanishing deduction is the final basis multiplied by the following
vanishing percentages:
G
If the decedent died within Vanishin ercenta e
L
1 ear from recei t of the ro 100°/o
2 ears from recei t of the 80°/o
3 ears from recei t of the 60°/o
4 ears from recei t of the 40°/o
5 ears from recei t of the 20°/o
More than five ears QO/o
. ertieS
If m o r e than o n e property qualifies for vanishing deduction, the prop
shall be grouped and totaled o n a per-year basis.

548

--
r 14 - Estate Tax: Deductions from Gross E t
, .pte sa~
(JIO
. veeau se of these
. decreasing ded u cti' on percer1tages that the deduction

;, 15pro pertY previously taxed is referred to as ",,
'ci ,an1s• h1ng · ,, Al so
• D ed uct1on,
I !f rnese year 1Y percentages, properties qualified for vanishing s ho uld be

dv ed annual.
b(ltJP
tradon 1 - Basic procedu res
111 115H, a bache Ior, d ,e
,ir . d w,'th th e 'iolJowing properties and allowable deductions:

Value upon Value
inheritance at death
P 1,200,000 P 1,000,000
received as inheritance 3 years ago
car . 9.000.00Q
other properties P 10,000,000
Gross estate
AJl ,vable ordinary deductions: 300,000
0 p 500,000
· f.-iortgage on the car 1,400,000
Indebtedness and taxes 300,000
Transfer for public use P 2,000,000
Total ordinary deductions before vanishing deductions

The vanishlng deduction shall be determined as follows:


P 1,000,000
Initial value (lower ofPl,200,000 and Pl,000,000) 200,000
1.,e5s: Mortgage assumed and paid (500,000 - ?300,000}
P 800,000
lnjtiaf basis
Less: Proportional other ordinary deductions
Initial basis/ Gross estate x other ordinary deductions 160.000
[PB00,000/Pl0,000,000 x ?2,000,000} p 640,000
Final basis 60o/o
Multiply by: Vanishing percentage (3 years) p 381,QQO
Vanishing deduction

Tax return presentation


These shall be presented in the estate tax return as follows:

Exclusive Communal Total


-P 10,000,000 P - Pl0,000,000
Gross estate
Less: Ordinary deductions 300,000
300,000
· Mortgage 1,400,000
1,400,000
· Debts and taxes 300,000 - 300,000
· Transfer for public use 384,000 - 384JOOO
· Vanishing deduction

Illustration 2 - Integrative application


Mrs. Z died on July 1, 2019 Jeaving the followi ng properties upon her death:

549
1 - uctfGn ftom Gro Estate ..
t
C
P i~oo,00o
a,ooo,o0o
4,000,000
1,000,000
2,000,000
7,000,000
r 1
am,ms dJ following deductions :
I
otncr pro rtl P
J
t , fncJu Ive of Pl 00,000 funeral expense 400,ooo
ntt,nrgill:C on tb ranch 900,000 ,
600,000
rui1a1u,onaJ fnfonn don:
• I
... ,ch had a (air value o( P2,400,000 in the gross estate o( her fatJi J
• d I subjto Pl,000,000 mortgage at that time. er 1
• Th ord, rd had a fair value o( P2,SOO,OOO on December 18, 2017.
•.•·, Z mortsa,ed the orchard on January 1, 2018 for Pl,500,000. Psoo oo
of th mortgage was paid before her death. ' O
• 1
Z designated rn her will to donate the commercial land to , I
O\' mm nt agency for public use.
1

R qui~d: D termine the vanishing deduction. I


1
Th ros estate shall be computed first as follows:
,
I
Ranch Exclusive <

Orchard p 2,000,000 - _Communal- Tota/


•'

-
Commercial land p 3,000,000 P 2,000,000 ,
f-·am,I}' home 1,000,000 3,000,000 ,
Other properties 1,000,000
Gross estate 2,000,000 2,000,000
p 3,QQQ QQQ 7,000,00Q 7.000,0QQ
- - P 12,QOO OOQ Pl s.000,000 I
ote-: Th rest house is an exclusive pro erty f M I
O
exduded u, gross estar:,. P r. 2, the surviving spouse; hence, it i5

Ti1e otr1er ordinary deduction shall al b


so e computed, as follows:
Casualty losses
- Exclusive - _Communal
Cla1r11s aga,rist the estate Total
Ur1r,a1d r11ortgage P 400,000 p 400,000
11·a11sfer f,Jr JJul,Jic Pllt pose p 800,000 800,000
600,000
Total oti1er orclina1 y cleduct1or1s - 1.000.000 600,000
1,000,00Q
E l ,q_QJtJJQQ g .-L_?._00,000 p 2,800.00Q
sso
_ Estate Tax: Deductions from Gross Estate
14
cnaPter
~ote: . ti against the estate is P900,000 - Pl00,000 = P800,000.
_ c1a1~house is not part ~f the gross estate since it is a property of the Mr. X, the surviving
1 Res . hence, not qualified for vanishing deduction.
2· sp~u~~rnrnercial land_ s~all be cl~imed through transfer for public use; hence, not
fh . ble again as van1sh1ng deduction.
3· c1a1rna

ishing deduction for the ranch and the orchard shall be compute as
rne van
foJloWs·· Exclusive Communal Total

. •al val ue P 2,000,000


~ 0 werof P2M & P2.4M) P 2,000,000
P 2,500,000 2,500,000
· orchard (lower of P2.SM & P3M) _ _ __
P 4,500,000
· Total P 2,000,000 P 2,500,000
0 100,000
. Debts assumed and paid 400,000
Less.
~ P 1,600,000 P 2,500,000 P 4,400,000

Less: Pro-rated deduction 170,667


Ranch: (P1.6M/P15M x P1 .6M) 170,667
200,000 200,000
orchard: (P2.SM/P15M x Pl.2M) _ _ __
p 1,429,333 P 2,300,000 P 3,296,667
• Final basis. •
8 0 o/o
'•• 60%
Multiply by:
!•
vanishing deduction p 857.600 P 1.840.000 P 2.697.600

Note:
1. The mortgage on the orchard is a new indebtedness of Mrs. Z. It is not a passed-on pre-
existing debt. Deduction for mortgage or indebtedness payinents pertains to mortgage or
indebtedness on the property assumed and paid for by the decedent
2. The ranch is June 30, 2017 to July 1, 2019 or 2+ years; hence, up to 3 or 60%.
3. The orchard is December 18, 2017 to July 1, 2019 or 1 + years; hence, u to 2 or 80%.

I Tax return presentation


These shall be presented in the estate tax return as follows:

Exclusive Communal Total


Gross estate p 3,000,000 P 12,000,000 PlS,000,000
Less: Ordinary deductions
- Casualty losses - 400,000 400,000
- Claims against the estate 800,000 800,000
600,000 600,000
· Unpaid n1ortgage
- Transfer for public use 1,000,0 00 1,000,000
857,600 1,840,000 2,697,600
Vanishing deduction
C

Note to Readers:
Ordinary deductions are allowable to all types of decedents, whether they
are residents, citizens, or non-resident aliens.
"'r- --_ _ _ _ _ _ _ _ _ _ _ _ _ _ _ __j

551
• A- rn Gross Estate
. Deductions Io i
Chapter 14 - Estate Tax.
11
SPECIAL DEDUCTIONS . I deductions: ft
The following are considered spec1a 0
1. Family home
2 Standard deductions 1
3: Benefits under RA 4917 jl

FAMILY HOME 11 . g house, and the land on which it .


Family home includes the dwe ind/ r members of his family reside is
°
situated, where t he decedent . anof the locality. as
certified by the Barangay Captain

st •t ted on the house and lot from the time .


The family home is deemed co_n i u ·dence and is considered as such for it
· II ·ed as a fam1 1Y re st r a
1s actua y occupi . . . t ally resides therein tArts. 152 and 1
long as any of its benef1c1ar1es ac u 53,
Family Code).
. d ~ -1 home the residence shall be characterized by
To be cons1dere 1am1 Y ' b f b ·
permanency. It 1s . the place to which, whenever a sent or us1ness or
pleasure, one still intends to return.

For purposes of availing of a family ho_m e deducti on to the ~xt ent allowable,
a person may constitute only one family hon1e {A rt 161, Ibid).

Requisites for deduction offamily home


1. The family home must be the actual residential home of tt1e decedent
and his family at the time of his death, as certified by the Barangay
Captain of the locality where the family horne is situa ted
2. The value of the family home mt1st be in cluded as part of the gross
estate of the decedent; and
3. The allowable deduction must not exceed th e lowes t of fai r market
value of the family home as declared or inclL1d ed in gross estate, the
extent of the decedent's interest therein, or P 10,000,000.

Not only marri~d decedents can claim family home. A s in gle decedent vvl10 is
a head of a family can also claim deducti on for fa mily home A single who is
not a head of a family is not legally allowed deduction for f~mi ly home.
A head of a family is an unmarried or le II h
parents or with one or more broth g_a Y separated man or wom an with one or botd
' ers or sisters or with gnize
natural or legally adopted children livin with ' one or more legitimate, reco_ ·ef
support, where such brothers or siste g h' a nd dependent upon h im or her for their chi
of age, unmarried and not gainfully rs or c 1ldren a re not more than twenty one (Z l) years.
I
regardless of age are incapable of self~mp oyed or where such children, b rotl1ers or s1st~r~f
the beneficiaries mentioned in Artie! :~~port because of mental or physical defect. or any o
and dependent upon the head of the; ·1 of the Fan1ily Code who is living tn the fan11ly honie
am1 Yfor legal support.

552
14 _ Estate Tax : Deductions from G
ter ross Estate
(IlaP
atton 1 . . ,
5
111 tr ent <lied leaving a fa1n 1ly home With a f .
~
4(icrc.dl· •s dcatl1.
. • 0! 1I
air value of Pl 7,000,000 at the
J tll
1
11
viilg shall be ded uctible for family h
l
i 1J o\ on1e unde h f
,-nc ,< de 11 t cases: r eac o the following
i cpcn
1111
Assumin th
Exclusive e am;/ home is
Common Exclusive
property of the
property of property of the
decedent
the spouses surviving
spouse
f family home P 17,000,000 Pl 7,000,000

lt \•1
•·Jue. o1 by % owned 1 000/Q
P 17,000,000
~1ult1dP )01t's i11terest 50o/Q 0%
a oeee e P 17,000,000 P 8,500,000 p
;3 0
I
P 10,000,000 Pl0,000,000 p
Lin1it 0
·iy
faOl l
home deduction p10., ooo., ooo es,soo,ooo p 0
Note: When the family home is an exclusive property of the surviving spouse, none of it is
reflected in gr~ss estlate. Hence, there should be no deduction for family home in accordance
",th the matching ru e.
e,
iuustration 2
Mr. Ti died leaving a family home consisting of a lot valued at P4,000,000 and a
house valued at Pl 1,000,000.
It
y Required:
Determine the amount to be included in gross estate and the deductible family
s home under each of the following independent cases:

t Case 1 Case 2 Case 3


Lot Exclusive of Mr. Ti Common
House Common Exclusive of Mrs. Ti Exclusive of Mr. Ti

•)
Solution:
1. Case 1
Gross Estate % owned Familv

home
Lot - SP - decedent P 4,000,000 100% P 4,000,000
House - CP 11.000.000 50°/o 5,500,000
To be reported in gross estate r
1s,ooo,ooo
P 9.500.000
Decedent's interest
Limit Pl0.000,000

Deductible family home P 9.500.000


✓ •

553
r
Gross Estate C
Tax: Deduc t ·ons
, from
Chapte r 14 - E5t ate
J
z. Case 2 cross Estate % ownect. Fi tni/ h l
-p 4 000,00
,
0Q 50°/o
p 2,000,000 l
l,,ot - CP . . ·n spoL1se
0% '•
SP - s11 rv1\fl g
I-lou se - tate 0
To be reported 111. gross es ·
Deccde11t's in terest
Lin1it

Deductlble fan1ily home

3. Case 3 % owned Fam;/ hotr,


Gross Estate
p 4,000,000 50% P 2,000,000
Lot - CP
11,000,000 100% 11,000,000
House - SP - decedent
To be reported in gross estate p 15.000.000
Decedent's interest P13,ooo.QQQ
Limit P 10,000,000 ~

Deductible family home e10.ooo~


3
Illustration . home was gu tted by fire which resulted
Mr Yoo's family . in his death. The family
ho~e had a value of PZ0,000,000 at the time of the fire.
There shall be no deduction for family home as the p~operty was already I

destroyed at the point of death. Neither shall the loss be claimed as casualty loss
pursuant to the matching rule.

STANDARD DEDUCTION
A deduction in the amount of PS,000,000 shall be allowed as an additional
deduction without the need of substantiation. The full amount of PS,000,000
shall be allowed as deduction for the benefit of the decedent.
In order to simplify tax administration of the estate tax, the TRAIN law
adjusted the standard deduction of PlM under the NIRC to PSM in lieu of
the funeral expense, judicial expense and medical expense which were
previously deductible in the old law. In view of this these expense
deductions are no longer allowed under the TRAIN law. 'fhey are deemed
included in the increase in the standard deductions.
f:Jote on unpaid funeral, judicial and medi'c
u ·ct f a1expenses
npa1 uneral and judicial expenses c t .fy as i·ndebtedness
because they are incurred after rathe th abn~o qua 1I
1
r an e1ore, death.
Medical expenses are incurred rio · al
expense logically qualifies as . pd br to the decedents death. Unpaid_ medic d
in the increased amount ofm e tedness, however, it is deemed include t
technically be deducted under t~tandarct deductions. Therefore, it canno
e contemporary rule.
554
ter 14 - Estate Tax: Deducti
chaP ons from Gros E
s state
£NEffTSUNDER RA 4917
8 rstJant to RA 4917 wt1ich took effi
pti ~fit or terrn ination benefit rec . ect on June 17 19 67 th .
beJ1 t h · eived by em 1 ' , e ret1rement
. bject to at ac ment, 1evy, execution p oyees of private firm . t
su , or any tax whatsoever. s is no
·suant to the NIRC which took fc:
P11 l h . e ,ect O J
received by the e1rs from the decedent's e: ~nuary 1, 1998, any amount
ath of the decedent-employee in a d Poyer as a consequence oif the
de d d . 1 R
ccor ance w·th
uowed as a e uct1on provided that th epublic Act No. 4917 is
~ocluded as part of the gross estate of the admount of the separation benefit is
1 e ecedent.
rnustration 1
In 2018, Mr. W resigned from his e 1
retire1nent pay from his employer's mp _oyment and received a PZ,000,000
1
pt,000,000 in the stock market and us~~ ~~:e benefit plan. Mr. W invested
car In 2019, Mr. W died leaving the car h" h 0th er Pl,000,000 to purchase a
his.investments with a value of Pl,SOO,O;Q_ 1c now has a value of P800,000 and

The amount to be included in gross estate shall be:


Car P 800,000
Investment in stocks 1,500,000
Total inclusion in gross estate P 2,300,000

The deduction for benefits under RA 4917 shall be nil. The NIRC qualified the
exemption of b~neftts received _as ~ consequence of death (i.e., death benefits)
rather than retirement or term1nat1on benefit received during the lifetime of the
decedent

Illustration 2
Mr. H, a bachelor, died in a car accident. His heirs received a Pl,500,000
termination pay from his employer on account of Mr. H's death.
The Pl,500,000 termination pay shall be included in gross estate and shall
likewise be presented as a deduction against gross estate.

SHARE OF THE SURVIVING SPOUSE


The share of the surviving spouse is one-half of the net conjugal or
community properties of the spouses. Needless to say, only married
decedents have this deduction.

After deducting the allowable deductions appertaining to the conjugal or


community properties included in the gross estate, the share of t~e
surviving spouse must be removed to ensure that only the decedents
interest in the estate is taxed (RR2-2003).

555
Chapter 14 - Estate Tax: Deductions from Gross ES t ate I
1,111.s 1.s a11 tJnequ 1.vocal dee 1aration
· that the amount
. of the .deduct·Ion f 1
,
0
s l1are of the survivi11g spouse a d mi·ts to adJustment 1n . th e dectu r tt:1~ \'
classification to ensure tl1at only the intereSt of th e decedent 1s taxect. C:tiCl~

lllt1stratio11 f th · h'
Using the sa1ne inforrnation in the Illustration 2 ° e vanis ing deduction ,
sl1are of tl1e surviving spouse shall be computed as follows: 'the 1
Exclusive Communal
Gross estate p 3,000,000 P 12,000,000
Less:
Ordinary deductions
- Casualty losses - 400,000
400,000
- Clairns agair1st the estate - 800,000
800,000
- Unpaid mortgage 600,000 600,000
- Transfer for public use 1,000,000 1,000,000
- Vanishing deduction 857,600 1,840,000 2,697.Q.Q_Q
Total p 542,400 p 8,960,000 P 9,502,400
Less: Share ofsurviving spouse •

- 2 4,4Bo, 000
Special deductions
- Family home
- Standard Deduction .' .
Statutory deduction vs. Actual share of surviving spouse
It must be noted that the deduction for "Share of s urviving spou~e" is a
statutory deduction and is different from the actual share of the surviving
spouse in the conjugal property. The statutory deduction is simply
computed as half of the net common properties in the estate tax return.
Illustration
A married decedent died with the following gross estate and allowable
deductions:

Separate properties of the decedent P 1,200,000


Common properties 3,800,000
Gross estate P 5,000.000

Actual estate expenses and deductions:


Funeral expenses P 500,000
Judicial or estate administration expenses 400,000
Claim against the estate - separate properties 400,000
Claim against the estate - common properties 600,000
Unpaid mortgage on separate properties 100,000
Unpaid mortgage on common properties
400,000
Loss of common properties
150,000
Transfer for public use
100,000
556

....,, i '· ~
~r··
,..... ·

~
;< • •

''
1
rer 14 - Estate Tax: Deductions f
cnaP ram Gross Estate
, hing dedtiction on common prop . .
tfl n1s . ert1es 200,000
.1, 1ion1e
1
'F;;n1d~rd deductions 1,000,000
SIJl 11 ' 1,000,000
rat1.itory deductio11 for survivirig
r~ce . ; tatutory ded uct1o
S .· spouse
n for tJ1 e share of the s . .
rn · estate tax rules as follows: ttrviving spot1se shall be computed
t.J . Jllg

- Exclttsive Common Total


ross Estate P 1,200,000 P 3,800,000 P 5,000,000
G . ordinary dedttctions
Less.
Claims against the estate 400,000 1,000,000
600,000
Mortgage 100,000 400,000 500,000
LOSS 150,000 150,000
Transfer for public use 100,000 100,000
vanishing deduction 200.000 200,000
et estate before special deductions p 600,000 P 2,450,000 P 3,050,000
Divide by: 2
Share of surviving spouse e 1.22s.ooo
Note:
1. Funeral and judicial expense are no longer deductible in computing taxable estate.
. As a rule, transfer for public use is presumed deductible against exclusive properties of
2 the decedent as married persons cannot dispose common properties without the consent
of the other spouse. ·
. Exceptionally, if transfer for public use or vanishing deductions pertain to common
3
properties, the applicable deduction is against common properties.

The actual share for surviving spouse


The actual share of the s urviving spouse would be computed from ilie actual
properties that remains of the common properties as follows:
Exclusive Common Total
P 1,200,000 P 3,800,000 P 5,000,000
Gross Estate
Less: Ordinary deductions 500,000 500,000
Funeral expenses 400,000 400,000
judicial expenses 600,000 1,000,000
400,000
Clai ms against the estate 400,000 500,000
100,000
Mortgage 150,000 150,000
Loss 100,000
100,000
Transfer for public use 0 0
- P 2,350,000
Vanishing deduction p 600,000 p 1,750,000
Net actual estate 2
Divide by: p 875.000
Share of surviving spouse
Note: turn is very different with the actual
. 1·n the estate tax re
1. The amount of statutory deduction
share of the surviving spouse.
557
Chapter 14 - Estate Tax: Deductions from Gross Estate

t x re turn fune ra l expe nse a nd juct· .


2. 'rhough no n-deductible in the estate_ a rt to e~tract the net pro per t ies or'c1a1 e)(lle
woul d be ded ucted to th e con1111on P1ope Y . h f d f h the Sh Ilse
. - against t e commo n un o t e spo ,,olls
Funer.ll nnd jud icia l ex pe nses a re c11a1ges < • • · ncen tive deduct·
1 Uses. es.
3. Vnn is hing d eductio. n 1.s not co n,s·1de red as 1t 1s a n ion anct does
p hysically dirnin ish the prope rties o f the e st ate. llot

NOTE TO READERS: THE ORDINARY AND SPECIAL ~LASS/FTC~ TION


The classit1catio11s "ordinary deduction · " and "special deduction"
. are actua
a bsent 1n · tl1e NIRC. The NIRC merely listed the ·ctdeductions . allowabJe ltyt
res1·d en ts or c1·t·1ze11 s a n d tl,ose allowable
_ .to non-res1
. . a.11ens. The orct·111a o
ent
and special classificatio11s exist only 1n the illu st rative guidelines of RRZ-20 ly
03 •
Ordinary and special in their usual meaning _
As previously rnentioned, ordinary dedu~tions ~ertarn to_ those ch_arges that
norrnally reduce the }1ereditary estate while special deductions perta1r1 to th
tl1at do not diminish the estate but are allowable deductions under the law. ose faJ11il)
se11ef
Evolution of Ordinary and Special Classification in the current usage otner
Under current usage, however, ordinary deduction per~ains to_ deductions Gross
claimable by any decedent: citizens, residents or non-re~1~ent aliens. Special Less: '
deduction pertains to deductions claimable only by c1t1zens or residents LI
excluding non-resident aliens. B
EstatE
Hence, vanishing deduction which is a mere tax incent ive is classi fied as an Less:
ordinary deduction even if it does not di mi nish the hereditary estate because it F
is claimable by any decedent. The new estate tax form, Form 1801, made this s
classification official.
Net e:
Benefit under RA 49 17 is commonly treated as a special deductio n becat1se it is Less:
normally deductible by citizens or residen ts and is least likely to be availed of NE1' '
by non-resident aliens.
IllusI
Note on the classification of benefits under RA 4917 The t
Death benefits under RA 4917 may be indicated as an ordi nary deduction or a
special deduction under the category "Others" in either classification. Fan1i
BenE
This can be made without defeating the law. Regardless of the classification
Othe
used for RA 491 7 death benefits, the share of the surviving spot1se mt1st adjust
to ensure that only the interest of the decedent is taxed as declared under RR2· Gros
2003. Less

If the decedent is single, there is no tax issue on which classification to use. In Esta
the case of married decedents, however, the fo llowing approach must be Less
followed: -
• If RA 4917 death benefit is classified as an ordinary deduction
The a~ount of benefits must be included in conjtigal or communal
properties of tl1e spouses bt1t is removed in full under ordinary deductions.

558
r 14 - Estate Tax : Deductions f
chapte ra m Gross Estate

!l.A 4917 death be11efit is classified us .


, r1ifl1e an1ou nt of the l)er1 efits rn ust b . a spec,a/ deduction
. f e I r1 c Iud •d · .
·opcrt1es o t I1e spoL1ses. Howeve th e in co nJt1gal or community
~~/ 7 .5/1all 011/y he one-/1alJ· of itsr~al:ed~d~c~ion for Benefits under RA
deducted tl1rougl1 tl1e deduction cate or ·,, fhi s 1s because the other half is
g Y, Share of the surviving spouse".
rratior1 1: RA 1917 death ber1 efits a .
rd1
111~15 ,ie a rn arried decede nt died with s an ° nary deduction
A~<:> 11/ hon1e of Pl,200,000 other c R~ 4917 death benefits of P800,000, a
f.1 111 ~~~00 exclusive pro pert/es Los onJ_tigal properties of P2,800,000 and
r~~ .' t conjugal properties wer ~ P?O~eoso, 01nd ebtedness, and taxes chargeable
3
gains , .

rJ,e net taxable estate sl1all be determined as follows:


Exclusive Conjugal Total
Fan1ily hon1e p - P 1,200,000 P 1,200,000
Benefits under RA 4917 800,000 800,000
other properties 500,000 2,800,000 3,300,000
Gross Estate p 5 00,000 P 4,800,000 P 5,300,000
1,e5s: Ordinary deductions
LfT 700,000 700,000
Benefits under RA 4917(others)____ 800,000 800,000
Estate after Deductions P 500,000 p 3,300,000 P 3,800,000
Less: Special Deductions
Family home {Pl,200,000 x ½ since family home is conjugal) 600,000
Standard deductions 1,000,000
Net esta te P 2,200,000
Less: Share of surviving spouse (?3,300,000 x ½) 1,650,000
NETTAXABLE ESTA TE P 550,000
Illustration 2: RA 4917 death benefit as a special deduction
The taxable net estate shall be determined as follows:
Exclusive Conjugal Total •
Family home p - P 1,200,000 P 1,200,000
Benefits under RA 4917 800,000 800,000
Other properties 500,000 2,800,000 3,300,000
Gross Estate p 500,000 P 4,800,000 P 5,300,000
Less: Ordinary deductions
LIT 700,000 700,000
p 500,000 P 4,100,000 P 4,600,000
Estate after Deductions
Less: Special Deductions
400,000
Benefits under RA 4917 {PB00,000 x 50%} . .
600,000
Family home (Pl,200,000 x ½ since family home 1s con1ugal)
1,000,000
N Standard deductions
P 2,600,000
et estate
2,050,000
~ess: Share of surviving spouse (P4,100,000 x ½)
P 550.000
ET TAXABLE ESTATE
559
Chapter 14 - Estat Tax: Deductions from Gross Estate .....,....t~ l ~ .
Bot h t.Teatments result In the same net taxable
f
estate. Despite thi
cial ded ct· s, 8eri
O
under RA 4917 is best presented as part spe u ion hecaUse ite/1~
deduction prescribed by special law. , Is~

RULES ON CLAIMABLE DEDUCTIONS PER DECEDENT CLASSIFICAl't


0~
r"'m~ ~~~= IIO~~~~r •iiR:;e~sl"1dr;e;nffoorrccltlti~z~e;nl**~ ]Nko~n~-~re~s;iidde;,n~t;:-:a:-;:I.~
• ✓ ✓ le
Ordina deduction , ..,. ✓
,
Special deduction None, except
standard deducti
✓ ons
•includes resident citizen, non-resident citizen and resident alien

DEDUCTIONS ALLOWED TO NON-RESIDENT ALIEN DECEDENTS


It should be emphasized that non-resident aliens cannot claim the spe .
deductions. Non-resident aliens can claim only the following deductions: Cia\
1. Prorated Losses, Indebtedness, and Taxes (LIT)
2. Property previously taxed (Vanishing Deductions) Tl
3. Transfer for public purpose bt
4. Share of the surviving spouse
Cl
5. Standard deduction
L<
Ti
Prorated LIT
p
The claimable deductible amounts of LIT of non-resident aliens are pro-
rated as follows: T
p
Philippine gross estate h
X Losses, indebtedness, and taxes tl
World gross estate

The prorata treatment will normally res ult in items of losses, indebtedness, II
A
and taxes being deducted at an amount different from their a ctual costs or
value. However, this is the legal treatment specifie d under Section (B)(l) of e

the NIRC. I!
'
II
Illustration a
An unmarried non-resident alien decedent died with the following gross estate V
and deductions details:
1
Philippines Abroad* Total - a
Family home p
Other properties estate · P 1,200,000 p 1,200,00°
4.000.000 4,800.000 s,soo.filill I
Gross Estate l
P 4.000.000 P 6000.000 PlOQO- •
I
560
_ Estate Tax: Deductions from Gross Estate
14
c~apter
Philippines Abroad* Total _
ainst the estate **300,000 2,100,000 2,400,000
.,,,,,5 ag .
11
c1a1 • properties 400,000 800,000 1,200,000
es on 700,000
Loss er to public use 300,0000 400,000
fraJ1S~
1 peso equivalents ** Pl 50,000 is unsupported.
Note:
. elaxing the matching rule with respect to deductions abroad, the total
W1111 et'.ble amount of LIT items must first be determined in the usual way
dedUC I • •
. to residents or citizens:
si1111 1ar
against the estate {P300K - Pl SOK+ 2.1M)
.
5
P 2,250,000
Claims on properties (P400K + PBOOK) 1,200.000
Losse
Total World LIT P 3,450.000

Tile Philippine gross estate ratio shall be computed as follows:

Philippine Gross Estate -- P 4,000,000


- = 40%
World Gross Estate P 10,000,000

The deductible amount of each LIT to be presented in the estate tax return shall
be computed as:
Claims against the estate (P2,250,000 x 40%) P 900,000
Losses on properties {Pl,200,000 x 40%) 480.000
Total Deductible LIT P 1,660,000

Property previously taxed (Vanishing Deductions)


The same vanishing deduction shall be deductible provided that the
property subject to vanishing deduction is included as part of gross estat e.
In other words, the property subj ect to vanishing deduction must be within
the Philippines at the date of death.

Illustration
A non-resident alien died with Philippine gross estate of P4M and foreign gross
.. ~state of P6M. The compt1ted allowable pro-rated LIT against Philippine estate
~s Pl,660,000. The Philippine gross estate inclt1ded a PlM property which was
inherited 2 years ago when its value was worth Pl.2M. The foreign gross estate
also includes a P2M property which was inherited 3 years ago when it was
valued at P2.SM.

:;e vanishing deduction of the non-resident alien decedent shall be computed


Initial I
Le va ue (PlM or 1.2M whichever is lower) P 1,000,000
ln·s~: Indebtedness paid 0
1ttal basis
P 1,000,000
561
----- -------- · ·-
. [)r,,1,Jcllons from Gross Estate
Chapt r 1.4 - I ,\.,ll t I ,1x . , ., _,.
1 1

Initial h~1sls IJ l ,OOo,oo0


I. "ss·
. .. · · t >1·1) t' 1\t•<I <l(•<lt1rl 11111
· •· t'r + ·r r PP)
( l11il 1.1I l1,1s1~/ l,l1 ili1•1•i11t (,I•.) X () .
1r1'1M/ 4MJx ,,,,,,<,<J,(JtJ<J/ -- ; lS.o~oo
1:inal l)asis P ~)l\S,ooo
Mulllt1ly t,y: V;111lsl1l111~ \l •rc1•11tar~c for up to 2 years - -- UQiyQ
Va11ishing ll l!clu cll<) IIS p 4<,u,ouo
Note: No vanl -:hlnp d1•d11 rt lo11 ra n h , rlal n1cd with respect to th e properties loc~\lcd 1b
1
· • gr oss estate·
because these ,u·c 11ol lrH l11tlcd \11 the I, , IIl lpp1nc ' road

Trans fer for p11hlic Purp<,se


'fransfcrs f<)r pttl)lic ptiri)<>SC hy n?n-resident alie_n_dc~eclc11ts arc clcducuble
only if the 11 r<>pcrtics l)cir1g tra11slcrrcd to the Ph1l1p11111e govcr11111cnt is I)
of the gross cst;1lc. ll mL1Sl he situated 1n · t he Ph·11 1· p111.ncs al the tin,cartof
transfer.

Cons istent with the 'N<J double deduction' rule, va 11isl1i11g dccJuclio11 cannot
he claim ed simLtltaneclusly with this deducti on. It is i11 tl1c i,,t ,rcst of the
decedent to claim tra nsfer for public purpose i11slcac.l of va 111sh111g deduction
tc, maximize the tax benefit of deduction.

Share of the surviving spouse


'l'hc dcduclihlc s l1arc of tl1c st1rvivi11g sp<)t1s' c)f a 11 011-rcsitlt'nl ,llicn
dcc.:cclc11t shall be compL1tcd ot1t of 1>\1 il 11 >1)111c cc>11jt1g,1I or ccln\n1u11t1l
prc,pcrtics us ing the sa me prc>ccdL1rcs as previc>L1c;ly cl1sc:l1ss •tl. -
Standard Deductions
In view <)f the removal of the proratctl ft1ncrdl c111 cl jt1tlici,1l 'X lll'I\Sl' 101 non·
rcc;idc11t ali en dcccdc11ts, tl1c 'l'llA I N law c1llc>ws ~, st,111tl,,rcl clctlutt1on of
P5()0,()()0 for no11-rcs idcnt alien dcccdc11ts. •

lllustr~tllon
A nc,r1 rc!->i d c11t alien died leaving th e f<) llc>wl 11g grc>ss cst,llt• :

~llUPlLl ·s Al)1'()1ltl 'l'owl


J•:xc:1usl vc• rr<> l)t•rty P J,()()() ,0()0 1>I (),()(l() ,(l(l(l p I i,000,000
( '.C) l l"l lllUlli1 1 p l <>1 >c rty _ 7...uuu.uuu l 3,UUl),000 10 .uuo.uoo
'l'c, \al (;r,,s ... l•:~ld lC P10.uuu.uuu l'JJ.UUU,000
l' L J.UUU.UUO
l'rc,rat<•cl I.I'!' I' I ,2() (),()()() lj l,tl(l(),llllU p 4,000,000
'J'r,Jll',ll'r tc, l'hlliJ>pl ne g<>vcrnn1cnt
lr1v1)lvlt11~ 1·xt lt1'iivc 1>r<>pcrly (l() () ,()()() 8()(),()()() 1,400.00°
Vanl•,hl11g ,l1•cl11c:.t1c,n <> n
lC>JlllrlUll,tl r1 l) J)C I ly 400,000
I\ ()(),()()()
-
562

- -- ------ - - - - - - - - - - - -
,,,.
' ·,-v:~ ,1 ! -l - E tate Ta : Dedl1ctio,1s fron1 Gross Estate

- Exclt.LS.i\'f.'- . CO 111111,ll.lli\L :r·o,tal


. tJtt'
\ ' ' P 3,l) 00,000 P 7,000,000 Pl 0,000,000
, il1,·t1 )Jl~
: • ~i,ttt i l IT
~-l' ~tt'r t\)f rltllllic tl e
1,200,000 1,200,000
600,000 600,000
\ '.tl\lSlt 111~ (ie(i tl cti o 11
~ ·, t,itt' ,lfter de-dt1ctions . 400.QOQ 400.000
P 2,400,000 P 5,400,000 P 7,800,000
'
. ·,,n1ndard ded11ction
~
• 500,000
...• <"'~t.ltt~
P 7,300,000
· . ~·t,urr of the surviving spouse (PS,400,000/Z)
, .\.\BLE I ET EITATE 2,700,000
I
P 4.600,000

t .. rtional Requirements on Deductions of Non-Resident Aliens


'\ c.ltlU :tion s hall be allowed in the case of a non-resident alien decedent,
1
the executor, ad n1inis tra tor, or anyone of the heirs, as the case may
.._ ~ -

~ includes in the r etl1rn the value at the tirne of the decedent's death that
n.irt

of his gross estate not situated in the Pl1ilippines .

)tltlARY OF DEDUCTION RULES

·'
Residents or Non-resident
Citizens aliens
~sses YES
Cairns against the estate YES Pro-rated
bdebtedness YES amount
-
,a.'=es YES
Transfer for public use YES YES
\'anishing deductions YES YES
Family ho me YES '
NO
Standard deductions . YES YES
Benefits under RA 4917 YES NO
. Share of the surviving s pouse YES YES

563
. 0 duct ions from Gross Estat e
Chapter 14 - Estate Tax. e

CHAPTER 14: SEl.,F-TEST EXERCI SES

Discussion Que5tio rls .. . s of deductions.


1 E11u111erate the tl1ree class1f1cat1o n d d t·
. I .
2 Discuss the ge nera prii, cip e . . I s of estate .e uc ions.
3.· Et1t1n1erate
· .
th e 1te1ns o f ord'ina ry deductions.
.
. f ci·at deductions.
4. E11umerate the 1te1ns o spe
f l ·ms against the estate.
5. Differe11tiate 1nortgage rom c a i . .
.
6. Discuss t h e concep t of a deductible loss 1n. estate .taxation.
7. Discuss the items of deduction of non-resident a lien decedents.

True or False 1
1. Items of deduction must be supported by documents.
2. As a rule, deductions are allowed if they are taken from gross estate or are
chargeable to gross estate. .
3. Double deduction is not allowed in estate taxation.
4. For married decedents, deductions are presumed common unless proven to
be exclusive.
5. Vanishing deduction is a special deduction.
6. The share of the surviving spouse is ½ of the husband's exclusive property.
7. Ordinary deductions normally result in reduction in the hereditary estate.
8. Special deductions do not reduce the hereditary estate of the heirs.
9. Special deductions are allowed only to resident or citizen decedents.
10. Non-resident decedents cannot claim the full amount of ordinary
deductions.
11. Obligations of the surviving spouse are deductions against gross estate.
12. The deduction for share of surviving spouse does not apply to unmarried
decedents.
13. The estate tax liability of the decedent is deductible in the computation of
the net taxable estate.
14. The vanishing deduction is applicable only if the previous estate paid estate
tax.
15. The vanishing deduction is applicable only to properties inherited by the I
decedent within five years before his death.

True or False 2
1. An unpaid funeral expense may be deducted through claim against the
estate.
2. Non-resident decedents cannot claim standard deductions.
3. Non-resident
. . alien decedents cannot claim deductions for the share of the
surv1v1ng spouse.
4. Only taxes and obligations accruing after death are deductible from gross
estate.
5. The loss of separate properties of the decedent is not deductible against
common properties of the spouses.

564
ter 14 - Estate Tax: Deductions f
cnaP rom Gross Estate
f he 1oss of separate JJropertics of th . . .
6, ~inst gross estate. e surv1v1 ng spoL1se is not deductible
~~e sta11dard deduction is claimable b .
7. \,\,ithotit tl1 e ,1e~d t(J prove c11title1nent toy a res1de_11t or citizen decedent
th
A fa n1 ily ho111 e 1s a clai111able deduct· 1 e ciedt1ct1on.
8: Standard deduction up to Pl 0,000,0~~ ~:::, ticPS0_0,000. .
9 Non-re 1dent alien decedents cannot . clain:ied as special deduction.
1
10· purpose fo r properties located abroaI aiin deduction for transfer for public
Losses of property before the death of th d
11' Non-resident alie11s cannot deduct s e . ecedent a~e deductible.
12· ded uctions. pecial deductions except standard
Non-resident alien decedents can c1aim.
13 indebte ness, taxes, and transfers for pub! ' only a proportion of lo sses,
· d
. . d d 1c purpose.
Res1der1t a 11en ece ents can claim deduct· f & •
14. . d d . . ion or ,am1ly home
va11ish1ng e uct1ons rs applicable only . .·
15· Philippines. on properties situated in the

Multiple Choice - Theory: Part 1


1. Which is correct regarding deductible obligations of the estate?
a. It must always be notarized.
b. It must be incurred before filing of the estate tax return.
c. It must be paid before the filing of the estate tax return.
d. It must be incurred before the death of the decedent.

2. Which of the following losses is not deductible?


a. Losses of properties compensated for by insurance
b. Losses arising from fires
c. Losses arising from theft or embezzlement
d. Losses arising from storms or shipwreck

3. Which of these do not diminish the hereditary estate of the decedent?


a. Judicial expense c. Medical expense paid before death
b. Transfer for public use d. Funeral expense

4. Which of the following decedents cannot claim special deductions for family
home?
a. Resident citizen c. Non-resident alien
b. Resident alien d. Non-resident citizen

S. Which is not considered in computing the share of surviving spouse?


a. Claim against the estate c. Standard deducti~n
b. Transfer for public use d. Vanishing deduction

565
d tions from Gross Estate
Chapter 14 - Estate Tax: De uc
I hare in common properties, funeral
6. In compt1ting the actua s e)(Pen
. t ses
deductible aga1ns f the surviving spouse at~
a. Separate property o decedent
b Separate property of th e
c.· Cornmon property of the spouses
d. Share of the surviving spouse
. 7
. . is an incentive ded uct1on.
7. Which of the following V ishing deduction
Family home c. an
~~ Standard deduction d. All of these

8. . bl net estate claims against the estate are d d


In ~omput1ng taxa e ' e Ucijbi,
against
A. exclusive property of the decedent
B. common property of the spouses

Which best completes the statement?


a. A only c. Either A or B
b. B only d. Neither A nor B

9. Which is not an ordinary deduction?


a. Family home
b. Vanishing deduction
c. Share of surviving spouse
d. Casualty losses of estate properties

10. Which ordinary deduction is least likely deducted against common


properties?
a. Losses of conjugal property
b. Claim against the estate
c. Communal properties previously taxed
d. Transfer for public use

Multiple Choice - Theory: Part 2


1. Losses of separate properties of the su rvivi 11g spo use are deductible again5t
a. Separate property of the decedent
b. Common property of the spouses
c. Both A and B

d. Neither A nor 8
1
. . . .
2. Wh 1ch of the following losses 1s claimable as deduction against gross estate-
a. Losses claimed in the income tax return of the estate
b. Losses occurring before the death of the decedent
c. Losses occurring after six months of the decedent's death
d. Losses of separate properties of the decedent

566
ter 14 - Estate Tax: Deductions f
chaP rom Gross Estate
Which is deductible by a non-residen .
J. Transfer for public use c M d. t alien decedent?
a. · e ical exp
b Family home d F enses
. . uneral expenses
The maximum allowable amount of ta nd
4· p 200 000 p s ard deduction is
a. ' c. 5 000 000
b. p 500,000 d. p 1'0,00'0,000

Losses may be deducted if


s. a. paid. c ·
· ~ncurred within 6 months before death.
b. unpaid. . d. incurred within one year before death.

6· What
N
is the maximum amount of deductibl
e casua 1ty losses?
a. one c. Pl,000,000
b. p SOO,OOO d. 10% of gross estate

7. The allowable deductible amount of family home is


a. 5% of gross estate c. P 5,000,000
b. P 2,000,000 d. P 10,000,000

8. If ~e family home is a conjugal or community property, what is the


maximum allowable deduction?
a. 5% of gross estate C. p 5,000,000
b. P 2,500,000 d. Pl 0,000,000

9. The allowable amount of standard deduction is


a. 5% of gross estate c. P 500,000
b. P 200,000 d. P 5,000,000

10. Which statement is incorrect?


a. Claims against insolvent persons are presented in gross estate and
deductions against gross estate.
b. Claim s against insolvent persons are presented separately from losses.
c. Non-resident alien decedents can claim deductions for losses,
indebtedness, and taxes.
d. Properties subject to mortgage are presented in gross estate at an
amount net of the mortgage.
11 · Which obligation is deductible against gross estate?
a. Bank loans acquired for the medication of the decedent
b. Income tax of the decedent before death
c. Sole obligation of the surviving spouse
d. Real property tax of the separate property of the surviving spouse

567

--- - - - - -
... ' .
'

I W'htmof
lm»tne tu patd
Income tax of th
Real pn,petty tu accruing after death .
.. 1') '
Real property ccru1n1 before d ath,
n cl im prorated amounts for the foll
•.._. on•·.,,.< dent altvn decclG n
OWJna
dedudlo t
c. Indebtedness
d . Vanishing deductions "u ~
I . _,..,,rh I not requisite of vanishing deductions? "
ff th property is acquired by inheritance, the prior estate must h~
paid the estate tax. e
b. tr the property is acquired by . inheritance, the prior estate must ha\'e
not claimed vanishing d ed uct1on.
c. The decedent must have acquired the property by way of inheritance or
donation.
d. The decedent must have acquired the property by purchase.

15. Van1sh1ng deduction is allowed if the property subject to vanishing


deduction is acquired
a. one year before death.
b. more than one year before death.
c. within five year before death.
d. more than five year before death.

16. A decedent died in a wild fire which totally gutted his home. Which is
correct?
a. lf the property is ins ured, the insurance reimburs ement is included in
gross estate and the loss is reported as a deduction.
b. If th e property is not insured, the insurance reimbursement is included
in gross estate and a deduction for loss is claimed.
c. No deduction is allowed with or without insurance reimbursement. •

d. With or without 1 eimbursement, a loss is claimable.

Multiple-Choice - Problems: Part 1

1. The heirs of the decedent co mpiled the fo llowing accrued taxes:

Before d eath After d eath_


P 40,000 p
Real p r operty tax
80,000 110,0 00
In come tax 400,000
Es t1mated estate tax

568
rer 14 - Estate Tax: Deductions f
cnaP rom Gross Estate
cornpu te the deductible taxes.
a. p 120,000 C. p 5 10.000
b. p 230,000 d. p 630,000

r hc exect1tor of the decedent ideritified th .


2. e following losses of properties:
Reported losses
LO ses ~ncurred within 6 m onths s ince death p
Losses incurred beyond 6 months since death 60,000
30,000
unreported losses
Losses incurred within 3 months since death p 10,000
t,.osses incurred beyond 6 months s ince death
20,000
What is the deductible amount of losses?
a. p 60,000 C. p 90,000
b. P 70,000 d. P 120,000

3. The following losses of properties occurred during the settlement of the


estate of Mrs. Undoy:

Losses of separate properties of Mr. Undoy p 60,000


Losses of common properties 40,000
Losses of separate properties of Mrs. Undoy 80,000
Compute the deductible losses from gross estate.
a. p 60,000 C. P 100,000
b. P 80,000 d. P 120,000

4. In the preceding problem, what is the proper depiction of the deductible


loss in the estate tax return?
Exclusive Common
a. P 60,000 P 40,000
b. P 100,000 p 0
C. P 80,000 P 40,000
d. P 120,000 p 0

5. The following properties were lost soon after the death of Mr. Fredo:

- Inventory stolen the decedent's business office P 300,000


- Jewelry, inherited a years ago by Mrs. Fredo 500,000

Upon d 1scovery,
· t h e even t was 1·mmediately
. reported to the BIR. The estate
executor claims the inventory theft in the income tax return.

569
. f om Gross Estate
. Deductions r
Chapter 14 - Estate Tax. .
ss against gross income of the Mr
f
·bJ amount o o I .
What is tJ1e deducti e
Fredo? c. p 500,000
a. PO d. p 800,000
b. p 300,000
. roblem, what is the proper presentatj
6. Jn the immediately preced;n;n~ on or
the loss in the estate tax re u .
Common
Exclusive p 0
a. p 300,000
p 0
p 300,000
b. p 300,000
c. p 500,000
p 0
d. p 500,000

7. Mr. Garno) donated the following properties in his last will and testament:
Cash - to Takusa, a social welfare institution p soo,ooo
Land - to Benguet State University at purchase cost 1,000,000

Additional information:
1. The donation mortis causa to Takusa was restricted for program
expenses. None of it would be used for administrative purposes.
2. The lot devised to Benguet State University had a fair value of
Pl,600,000 at the decedent's death.

Compute the deductible amount of deductible transfer for public purpose.


a. p O C. p 1,500,000
b. P 500,000 d. P 1,600,000

8. The following relate to the estate of a decedent:


Gross estate, (PlM is exclusive)
P 3,000,000
Expenses and obligations:
- Funeralexpense
- Judicial expense 200,000
- Indebtedness and taxes 300,000
- Losses 150,000 •
250,000
Compute the deduction for th
. a. P 550,000 e share of the surviving spouse.
b. P 575,000 C. p 800,000
d. P 1,oso,000
Multiple-Choice - Probl
ems: Part2
1. A decedent died le .
deductions: 1 the following properties and
av1ng an estate w·th

570
r
hapter 14 - Estate Tax: Deduct·
C ions fron,
Gross Estate
Gross estate, (P2 M is exclu .
. SlVe)
Expenses and obligations:
_ Fu11eral expense p 4,000,000
_ Judicial expense
200,000
_ Indebtedness and taxes
100,000
_ Losses
250,000
150,000
What is the share of the survi .
p 2 000 000
1 1
ving spouse?
~- p 1 650 000 C. p S50,0QQ
. ' ' d. P 650,000

2. Tl1e following data pertains to the estat f


e O a decedent:

Gross estate Separate Common


P 2,000,000 P 3,000,000
Deductions:
Funeral expenses
Judicial expenses P 180,000
110,000
Indebtedness and taxes 200,000 300,000
Losses 50,000 150,000
Transfer for public use 100,000

Compute the deduction for the share of the surviving spouse.


a. p 455,000 C. P 1,275,000
b. P 825,000 d. P 1,130,000

3. Mr. Maestro died leaving a family home valued at P18,000,000 which he


inherited during the marriage when it was worth P 8,000,000. What is the
deduction for family home?
a. P 0 c. P 9,000,000
b. P 8,000,000 d. Pl0,000,000

4. The spouses own a residential lot as their only real property. The lot had an
assessed value of PlS,000,000, zonal value of P18,000,000 and an
independent appraisal value of P25,000,000.

What is the deductible family home?


a. po c. P 9,000,000
b. p 7,500,000 d. p 10,000,000
5 d M Gallante constructed a house using
· Du~ing their marr!age, Mr. an r~he lot was inherited by Mrs. Gallante
j their salaries totaling P12,000,000. d . d when the house and the lot had
w~en it was P8,000,000. Mr. Galla~~O ~~d Pl2,000,000.
fair values respectively of PlS,000,

571
. f rom Gross Estate
Chapter 14 - Estate Tax: Deductions

What is the deduction for family ho~ e; •

pO c. p 7, 0 000
I

:~ P 6,000,000 d. p 10,000,000

6. A decedent died leaving a family home to his children:

P 3,000,000
Lot - separate property of the decedent
12,000,000
House - common property

Compute the amount of deductible family home.


a. p 3,000,000 C. p 9 ,000,000
b. P 7,500,000 d. P lO,OOO,OOO

7. The estate of the married decedent received P SO,OOO b dendefit under RA


4917. The heirs want to claim the benefit as part of; pecia1 e u ction. What
is the deductible amount of benefits under RA 4917 •
a. p O C. p 80,000
b. p 40,000 d. None of these

8. In the immediately preceding problem, if the b enefits und er RA 4917 is


claimed as part of ordinary dedu ction, which of the following is the proper
presentation of the deduction in the estate tax return?

Exclusive Common
a. p 80,000 p 0
b. p 0 p 80,000
C. p 40,000 p 0
d. p 0 p 40,000

9. A non-resident alien decedent died leaving a substantial estate in the


Philippines. He is m arried with six dependents. How much standard
deduction he can claim?
a. P 0 c. P 500,000
b. P 200,000 d . P 1,000,000

10. A non-resident alien decedent had the following possible deductions:

Funeral and judicial expenses p 400,000


Obligations, 40% are payables in the Philippines 500,000
Losses of prop_e~tie_s (60°/o occurred in the Philippines) 1,000,000
Transfer of Ph1l1pp1ne property for public use 400,000
The _decedent ~ad a P4M ancestral house in the Philippine and P6M total
foreign properties.

572
_ Estate Tax: Deductions from G
ter 14 ross Estate
~aP .
~- te the total d e dt1 ct1ons allrJwabl t·
. 1t1 e or Ph ·1· . .
(,0111~ 1,0() (), ()00 C. p 1,500,0QQ I lflJ)Jnc estate tax.
,1, p 1,o80,(J00 d. P S,soo,ooo
b
. J,1panese citizen residing in Japan had the folio ·
11 ;cdnctions: w1r1g properties and

rti es in the Philippines


prol~:rties i11 japan P 3,()00,000
pro! rties in Hong Kong 6,000,000
prope
I \.vorld estate
1,000,00Q
fota Pl O..QQQ-4000
• . and
indebtedness
1,.os:>e5 P 3,000,000
' d1cal expense
~ e
1
bl . f 450,000
Transfer for pu 1c use o properties located in Japan
500,000
compute the total deductions allowable.
a, p 1,400,000 C. p 2,035,QQQ
b. p 1,535,000 d. P 2,350,000

11 AChinese citizen re~i_


d in~ in Quezon City, Philippines, died leaving several
properties in the Ph1l1pp1nes. How much standard deduction can his estate
claim?
a. p 0 c. P 5,000,000
b. P 500,000 d. Pl0,000,000

11 On November 1, 2018, Gwen died leaving the following properties:

Agricultural land inherited February 2, 2016 P 1,200,000


House and lot 2,000,000
Cash 2,800,000
Car 500,000
Other personal properties 1,000,000
Total properties P 7,500,000

p 500,000
Mortgage on the agricultural land
1,200,000
Other indebtedness
400,000
Deductible losses •
P 2,100,000
Total ordinary deductions
G d
h The prior estate paid
thwen paid P300,000 in mortgage before ~er eatthen valued at Pl,000,000.
e estate tax on the agricultural land which was
0
; tnPute the vanishing deduction.
b, P403,200 c. P330,400
. P388,800 d. P302,400
573
Chapter 14 - Estate Tax: Deductions from Gross Estate

14. Mr. Y, single, died leaving properties he inherited 2 ½ Years a


I
current fair market value of P 800,000. The property Was inherit go \villi
Was worth Pl,000,0000 and had a P850,000 unpaid mortgage. ~d \vheh i:
Psso,ooo until his death. Oth_er properties of Mr. Y had a fair ma,{ YP,iq
of Pl,200,000 at the time of his death. •t ~.1 ,
'
1
The losses, taxes, and transfer for public purpose and Pt 40,0oo.

How much was the vanishing deductions?


a. P 139,500 c. Pl 17,000
b. P 180,600 d. P107,611

574
r

ter 1s - Estate Tax Payable


chaP

cflAPTER 15
esrATE
~ TAX -
;---- PAYABLE
-..::.._
tcr overview and Objectives
________
c11aP
~;:;;;;-;-~ha pter,.readers ~;;-;~~~~;~~-;~-~:~--~~~--~--:-----------------------
The computat1o na l procedures of the t P e d.
1, · ne taxable estate
The computat1ona1 procedures of the estate t d h .
2· tax credit ax an t e computat10n of

3. The other ad ministrative requirements·m r·t·


I mg t he estate tax return

DETERMINATION OF THE NET TAXABLE ESTATE

Illustration 1: Single resident or citizen decedent


An unmarried _Filipino decedent died leaving the following properties and
possible deductions:

Phili/!J!.ines Abroad. IfJlgJ_


p 120,000 p 120,000
Motorcycle
Business interests 6,500,000 p 800,000 7,300,000
Family home 10,200,000 10,200,000
Other personal properties 200,000 lQQ,OOQ 3QQ,QQQ
Gross Estate P17,020.000 p 900.000 P17,920.000

p 180,000 40,000 p 220,000


Funeral expenses
40,000 30,000 70,000
Judicial expenses
300,000 200,000 500,000
Obligations 100,000
50,000 50,000
Losses 550,000
150,000 400,000
Medical ex enses

The net taxable estate and tax due shall be computed in the estate tax return as:

Separate {ommon To1fil


p 0 Pl 7,920,000
Gross Estate Pl 7,920,000
Less: Ordinary deductions P 500,000
Obligations p 500,000
100.000
Losses 100,000
Net estate before share of p 0 Pl 7,320,000
surviving spouse Pl 7,320,000 0
Less·· Sh are of the surviving spouse
N Pl 7,320,000
et estate before s ecial deductions

575
Chapter 15 - Estate Tax Payable

Net estate before special deductions


l ess: Special deductions
Family home
Standard deductions
Net taxable estate
Multiply by:
Estate tax due

Note:
1. Since decedent is resident or citizen, gross estate covers all properties within
.
2. · · · or me d'1ca I expenses 1s
No more deduction for funeral, 1ud1c1al . a 11 owed. orw1th%t
3. Family home deduction= Lower of Pl0,000,000 and Pl0,200,000 actual fam ily h
orne

Illustration 2: Single non-resident alien decedent


A non-resident Japanese died leaving the following properties and po .
ss1b1
deductions: e

P.b.ili1212ine~ tlbrQad IfilgJ


Condominium unit P 2,000,000 P 2,000,000
Business interests 1,500,000 p 800,000 2,300,000
Family home 1,200,000 1,200,000
Other personal properties JSQ,OOQ 1.150,QOQ 1,SQO,OQO
Total P 3.850,000 P3.150.000 P 7.000.000

Funeral expenses p 50,000 240,000 p 290,000


Judicial expenses 40,000 110,000 150.000
Obligations 300,000 200,000 500,000
Losses 80,000 120,000 200,000
Transfer for public purpose 50,000 20,000 70,000
Vanishing deductions 140,000 80,000 220,000
Medical ex enses 150,000 400,000 550,000

The net taxable estate and tax due shall be computed in the estate tax return as:

Se12arate Common Total


Gross Estate P3,850,000 p 0 P 3,850,000
Less: Ordinary deductions
Pro-rated LIT
Obligations P 275,000 p 275,000
Losses 110,000 110,000
Transfer for public use 50,000 so,ooo
Vanishing deductions 14Q,QQO 140,QQQ
Net estate before share of
surviving spouse P 3,275,000 p 0 P3 275,000
Less: Share of the surviving spouse ~
Net estate before s ecial deductions p 3,275,000

576
r 15 - Estate Tax Payable
cnapte
ta~te;ib~e~fi;or;;e~s~p;e~c~iaJ°Idd;ed~u~cctti~·o » n ; ; s ~ - - - - - - - - - - - - -
.ret
1• es
• special d ed uc t·10ns P 3,275 ,000
iesS-
-iy home
Oo faIT1~ rd deductions
QQ sranta:able estate 500,000
Do Net . P 2,775,000
rJultiP1Y by.
~ ..,te tax due 6%
~ ESI-" P 166.500

Note: f
Gross estates o NRA decedents include Philip .
1· Matching rules apply to transfer for ubl' . pme pro~e~es only.
le
2. •erred within the Phi11'pp1· . pd d IC use and vamshmg deductions. Only those
trans1, nes 1s e uctible.
_ NRA decedents are allowed PS00,000 standard deductions.
3

:sibJe Jllustration 3: Married resident or citizen decedent


Mr. Rice, a resident alien, died leaving the following properties and estate
deductions:

Separate properties Conjugal


wcated in the Philippines: Mr. Rice Mrs. Rice Properties
Personal properties P 8,400,000 P 1,100,000 P 2,500,000
Family home 10,800,000
Other real properties 2,300,000 1.800,000 4,000,000
Total Philippine properties Pl0,700,000 P 2,900,000 Pl 7.300,000

located abroad:
Personal properties P 2,000,000 P 1,000,000 P 4,000,000
Real properties 1.000,000 2,500.000 4,200,000
Total foreign properties P 3,000,000 P 3,500,000 P 8,200,000

World Properties P13.700,000 P 6.400.000 P25.S00.000


as: The executor of Mr. Rice compiled the following expenses and deductions which
are rnatched to their respective sources:

Separate properties Conjugal


Philippines and Abroad: Mr. Rice Mrs, Rice Properties
Funeral expenses p 250,000 P - P 350,000
Judicial expenses 600,000
Obligations 1,000,000 1,800,000 2,000,000
Losses 200,000 400,000 300,000

The net taxable estate and tax due shall be computed as:

577
+
r
Chapter 15 - Estate Tax Payable
Common

~
Separate
rto,100,000 Pl 7,300,000
Philippine properties 8.200.000
~
3,000,000.
Foreign properties P13,700,000 P25,SOO,OOO
Gross estate
Less: Ordinary deductions 1,ooo,ooo 2,000,000 3,00 0,000
Obligations _ __,20.ol1lo:.11,o~oo!.!..-_ _.3"""o.....
o-,o...,._oo
_ _.....o.!csQQ.QQQ
Losses
Net estate before share of P12,500,000 P23,200,000 P3S,7oo, 000
surviving spouse 7 2
Less: Share of surviving spouse ~
P24,100, 000
Net estate before special deductions
Less: Special deductions
s,400,0 00
Family home (See Note 2)
Standard deduction s,ooo,ooo
P13,700,00Q
Net taxable estate
Multiply by: ---~6.%
Estate tax due P 822,0Q,Q

Note:
1. The gross estate of citizens and residents includes both separate property of the
decedent and common properties wherever situated.
2. The deducible family home is the lower of PS.4M (50% x P10.8M) and the P10M limit.

Illustration 4: Married non-resident alien decedent


Mrs. Kay Yakoto, a non-resident Japanese, died leaving the following properties
and estate deductions:
Separate properties Conjugal
located in the Philippines: Mr. Yakoto Mrs. Yakoto Properties
Tangible personal properties P 1,400,000 P 1,100,000 P 2,500,000
Intangible personal properties 300,000 600,000 200,000
Real properties 1.300.000 1,800,000 5,800,000
Total Philippine properties P 3.000,000 P 3,500,000 P 8,500,000
Located abroad:
Personal properties P 2,000,000 P 1,000,000 P 2,000,000
Real properties 1,000,000 2,500,000 2,500,000
Total foreign properties P 3,000,000 P 3,500,000 P 4,500,000
World Properties P 6.000.000 P 7.000.000 Pl 3.000.000

A breakdown of the items of LIT is presented as follows:

Philippines Abroad
Separate Common Separate Common Total
Obligations 1M 1.4M lM 1 6M SM
Losses 200K 400K 200 · BOOK
Transfer for public purpose 350K 2SO~ 600K

578
r 15 - Estate Tax Payabl
cnaPte e
r rnegr 0 55 estate s hall be comput d
e as:

. ;negrossestate ~ _ Corn,_____
3
philt~g~1 gross estate ~ ,soo,000 ~ o - Total
F0 ret gross estate , ,0oo P 12,000,000
world ~ - 4,SOO.OQU
. ~ PJ,J,000.002
8.000,00.Q
.e.zo.ooo ooo
lppine Gross Estate Ratio= Pl2M/P
phi t 2 OM= 60%

Note to readers:
must be noted that the NIRC requ· d
It D ire a pro r t
on LIT items. . ue to this there is a possibili ~ha a treatment of deduction
an become higher than the actual LIT . ty at the deductible amounts
crocedure is mandated by law· henc . m the Philippines. This prorata
p h ' e, It must be foll d
not be the more t eoretically acceptabl e procedure. owe even if it may

The total allowable deductions for LIT items sh 11 b


a e computed as follows:
Allowable
Global LIT Phil. Ratio Phil. LIT
Obligations P 5,000,000 x60% P 3,000,000
Losses 800.000 x60% 480,000
Total P 6,500.000 P 3,900,000
Note: Transfer for public purpose is a separate item of deduction.

The deductible amounts of LIT between separate and common properties shall
be pro-rated based on the ratio of actual LIT amou nts as follows:

Allowable Percentage = Total Proportional Value Allowed


Total Actual Philippine Value

Hence,
Total Allowed / Actual Phil. Value = _ __,_01<.. _o_ _
Obligations
p 3,000,000 p 2,400,000 125%
Losses
480,000 600,000 80%

Thus,
Common Total
Separate
® ligations: p 2,400,000
Actual p 1,000,000 p 1,400,000 125%
125% 125%
x Allowable % p 3,000,000
p 1,750,000
Deductible p 1,zso,000
~: 400,000 p 600,000
Actual p 200,000 p 80%
80%.
80% 480,000
xAJlowable % 320,000 p
Deductible p 160,000 p
579
.,.
Chapter 15 - Estate Tax Payable

du e o f Mr·s· Yakoto shall be computed as··


The net tax.lblc estate and tax

Separate Common ~
P
3,soo,ooo P a,soo,ooo P12,ooo.ooo
Gross estate
Less: Ordinary deductions
Pro-ra ted LIT: P 1,250,ooo P 1,1so,ooo P 3,ooo,000
Obligations
160,ooo 320,000 4Bo,000
Losses
Transfer for public use
350,000. - ~
Net estate before s hare of
p 1,740,000 P 6,430,000 P s,170,0 00
s urvivi ng spouse +2
Less: Share of surviving spouse ~
P 4,995,ooo
Net estate before special deductions
Less: Special deductions
Family home
Standard deduction 500,Q_QQ
Net taxable estate P 4,4ss,ooo
Multiply by: ---~6.%
Estate tax due P 267.3QQ

Note: Only properties located in the Philippines can be claimed as transfer for public
purpose. (Matching Rule}

DETERMINATION OF FOREIGN TAX CREDIT


The estate tax due of decedents who are taxable on global estate such as
resident citizens, resident aliens and non-resident citizens shall be further
reduced by foreign tax credit for estate taxes paid in foreign countries.

The foreign tax credit shall depend on whether the decedent has properties
in a single foreign country or multiple countries.

Single Foreign Country


The foreign tax credit s hall be whichever is lower of the actual foreign estate
tax paid and the following limit:

Foreign net taxable estate


x Philippine estate tax due
World net taxable estate

Multiple Foreign Countries


The lower of actual estate tax and the foregoin I' •t & h country is
determined first. g 1m1 1or eac

580
f cna
1ne
Pter
1
5 - Estat e Tax Payable

,, 1 foreign tax credit shall be th 1


flI1"
ble per country an d the world e et ower o f the total of th .
11owa s ate tax credit 1· . e tax credit
3 bl ,m,t computed
foreign net taxa e estate as:
'[Ota1~:...-- - -----=--
world net taxable estate
X
Philippine estate tax due

tion 1: One fore ign country


Ilws~
a
·dent
resl ctece.d ent pa1·d Pl 10,000 estat e tax m
. Jap
A
fl kdown of his net taxable estate: an. The following shows
brea a
,cable estate in the Philippines P 1,200,000
Netta . J
et taxable estate m a pan 1.800,000
N Id net taxable estate P 3,000,000
wor
e estate tax on the P3,000,000 world net taxabl e estate 1.s P180,000.
Th
The estate tax credit shall be computed as:
Actual foreign estate tax paid p l l 0,000
Limit: (P1,800,000/P3,000,000 x PlB0,000) p 108,000

foreign tax credit - LOWER P 108,000

The estate tax payable shall be computed as:


P 180,000
Estate tax due
Less: Foreign tax credit 108,000
Estate tax still due or payable P 72,000

Illustration 2: Multiple foreign countries


Acitizen decedent had the following data:
Net taxable Estate tax
Estate paid
P 1,700,000 P 200,000
Hong Kong 150,000
2,800,000
Korea
Philippines
1.soo.000
Total £_, 6,000,0QQ
36
The estate tax on Lhe P6,000,000 world net taxable estate is P 0,000.

Limit 1: Per country limit


Korea
_ Total
Hong Konl;_
Actual P 200,000 P
1so,ooo
. . estate tax paid
Llln1t:
· Pl ·7M/P6M x P360,000
.p 102,000
2 8
Low tR/P6Mx P360,000 _ _ _ _ 168,000 ~
p 102,0011 ~ ~
581
Chapter 15 - Estate Tax Payable

Limit 2: Total foreign countries


P 252,000
Lower in Limit 1 M)/P6M x P360,000 P 270,0QQ
Limit 2: (P1.7M+PZ.S
P 252.0Q_Q,
Final Foreign Tax Cred 1't (LOWER)

II be computed as :
The estate tax payable sha p 360,000
Estate tax due 252,000
Less: Foreign tax credit p 108,000
Estate tax still due or payable

Determination of the Net Taxable Estate Per Country

. d - foreign tax credit


Illustration
A 1: Smgle
single citizen dece
decedent ent
died . th e following gr oss estate and deductions:
leaving

Philivpines Lapan Toml


Gross Estate pz1,ooo.ooo p 9,000,000 P30,ooo.oD_Q
Deductions:
Obligations 300,000 200,000 soo,ooo
Losses 50,000 50,000
Family home 100,000
12,000,000
Estate tax aid
- p 320,000

The net taxable estate of each country s ha ll be computed as:

Gross estate Philippines Iapan Total


Less: Ord inary ded uctio ns P21 ,000,000 P 9,000,000 P30,000,000
Obliga tions
300,000 200,000
Losses 500,000
Net estate befo re share of 50,0QQ 50,0QQ 100,000
su rviving spouse P20,6S0,0OO P 8,750,000 P29,400,000
Less: Share of surviving spouse
Net estate before -----
special deduction
Less: Special deductions P20,6SO,ooo P 8,750,000 P29,400,000
Family home (Note 1) 10,000,000
Standard ded uction (Note 4) 3,500,0QQ 10,000,000
Net taxa ble estate ~ 7. 1S.OJlQQ 1 OQQ
Multi ply by: 5,000,000
Esta te tax due ~ ~ ~ OUQ Pl 4,400,000
Note: 6%
1. The deductible family home is the
d d 1
-- 8-64~00
e uct1'bl e tn
· its
· country of location. ower of Pl2•• 1•1 . Fam ily home
1• 1 and PlO"~
15

582
~[ pter 1s- Estate Tax Payable
c~a
pS 000,000 standard deduction .
'fhe fo~e be allocated in each country1bs deductible frorn total
z, there ~ gross estate. It must
allocable standard deduction Per tate.
'fhe Philippines
· P21 ,000,000/P30 ooo
country shall be:
. Japan P9,000,000/P3o ooo 0'00000 X PSM:::
• , , X PSM ::: P 3,soo,ooo
fotal - 1.Soo.oQQ_
~ soooo~
te tax credit and tax still due
£sto
The estate tax credit shall be computed as:

ACtual foreign est~~e tax paid


Limit: [P7,150,000, Pl 4,400,000 x P864,000J P 320,000
P 429,00Q
foreign tax credit - LOWER
P 320.00Q
The estate tax still due or payable shall be computed as:

Estate tax due


Less: Foreign tax credit P 864,000
Estate tax still due or payable 320.000
P 544.000
Illustration 2: Married decedent- foreign tax credit
Mr. Nasser, a citizen decedent, died leaving the following properties and estate
deductions:

Mr. Nasser Common Total


Properties in the Philippines P27,000,000 P45,000,000 P72,000,000
Properties in Egypt 3,000.000 5,000.000 8.000.000
World Total P30.ooo.ooo PS0.000.000 PS0.000.000

The executor of Mr. Nasser compiled the following deductions.

Philippines Egypt Total


Obligations P *700,000 P **800,000 P 1,500,000
Losses **200,000 *100,000 300,000
Family home **15,000,000
Estate tax paid ? P 290,000

*Attributable to exclusive properties of Mr. Nasser


**Attributable to common properties

583
Chapter 15 - Estate Tax Payable

The taxable net estate per country


shall be computed as follows:
r
(Amounts are in thousands}

Phili ines E&Ypt - G1t4


PP Total Separate Common Tota/ No
Separatoe
Gross estate P 27,00
C:Po4m5mogg P72,000 P 3,ooo P s,ooo
,
~ ~oTAL
,oo
0
Less: 800
700 800 l
Obligations 700 ZOO ZOO lOO _ _ _ JQQ ,Soo
Losses ___ - - ~
Net estate before
share of ss P 26,300 P 44,800 P71,100 P z, 9 oo P 4,zoo P 7,lOO P7a,
Less: Sh are of Ss '2, .22,400 - - - 7
2 2,10° 200
r::n:
. - ~ _24 ~
Net estate before
special deduction P48, 700 p S,OOO P53, 7
Less: 00
Family home 7,500 - 7,Soo
Standard deduction 4,500 _ _S=QQc.. : ~
Net taxable estate (in thousands) P36.700 P 4.S0Q ~
Note:
1. Family home= lower of (PlS,000,000 x 50%) or lOM = P7,S0O,000.
2. Standard deduction is allocated based on gross estate as follows:
Philippine Std. Deduction = P72M/P80M x SM P 4,500,000
Foreign Std. Deductions= P8M/P80M x PSM 500,000
Total
P s,ooo OQ.Q
Tax credit and estate tax still due
The estate tax shall be computed as follows P41,200,000 x 6% = P2,4 72,000.

The estate tax credit shall be computed as:


Actual foreign estate tax paid
Limit: (P4,500/P41,200 x P2,472,000) P 290,000
P 270.000
Foreign tax credit - LOWER
P 270,000
The estate tax still due or payable shall be computed as:

Estate tax due


Less: Foreign tax credit P 2,472,000
Estate tax still due or payable 270,000
P 2,202,000
Determination of estate tax due with . .
In our example we used only on multiple foreign countries
, e country to · t·fy · ·I
countries are involved similar p d simp I the 11Iustration. If mult1p e
, roce ures and logic are applied.

584
r 15 - Estate Tax Payable
cnaPte
r£ TAX REQUIREMENTS
~srAE tate tax return
1· c:rtified Public Accountant (CPA) Certification
2.
state taX return and its contents
ffte executor, a d mm1s
· · t rator or any of th h .
rne e tax return under oath, setting forth eh e1rs s~a]] file in duplicate an
estate t e foHowmg·
vaJue of gross estate at the point of death or i · .
1, . n that part of his gross estat 't . ' n the case of non-resident
a11e , . e si uated m the Philippines
The deduct10ns allowed from gross estate
2
3.. supplemental data which may be necessary to est a bl1s ' h t h e correct tax

CPA certification
Where the value of th e gross estate exceeds PS,000,000, the return shall be
accompanied by a sta tement certified by a Certified Public Accountant.

Contents of the statement:


1. Itemized assets of the decedent with their corresponding gross value at
the time of death or, in the case of a non-resident decedent, that part of
his gross estate situated in the Philippines
2. Itemized deductions from gross estate
3. The amount of tax due whether paid or still due and outstanding
Deadline of filing the estate tax return
The estate tax return shaH be filed within one year after the date of death.

Extension of filing . .
The Commissioner is authorized to grant, in mentonous cases, a reasonable
extension not exceeding 30 days for filing the return.

Venue of filing

a. For resident decedents . t the estate of the decedent


11
The administrator or executor sha reg~s e;DO where the decedent is
and secure a new TIN therefor from t e
domiciled at the date of his dea th ·

b. For non-resident decedents . 'th executor or administrator in


.. or ahen w1 b
Whether non-resident citizen hall be filed and a new TIN shall _e
5
the Philippines the estate tax return h cutor or administrator 1s
, h e sue exe TIN
secured from the RDO w er t rn shall be filed and new
.1st d the re u .
registered. If he is not reg ere '. . . d·ction of his legal residence.
shall be secured form the R
oo having JUflS 1
585
Chapter 15 - Estate Tax Payable

Where to file the estate tax return?


1. Accredited Agent Bank
2. Revenue District Office
3. Collection Agent Cty Municipality ·
4. Duly authorized Treasurer of the II d otr th ti. f h. idn Which th
. .
d eced ent or a d mm1s trator was domici e
. . a e me o is eath e
5. Office of the Commissioner, if the admm1strator or executor has no legal
residence in the Philippines

Payment of Estate Tax . . .


The estate tax shall be paid at the time the return is filed following the ruJ
e,
"pay as you file."

Insufficiency of cash to pay tax


If there is difficulty in paying the tax, the same may be settled by:
a. Installment payment
b. Partial disposition of estate

Installment payment
The estate tax may be paid in installment within two years without the
imposition of interest or civil penalties.

Subject to approval of the CIR, the estate tax may be paid as follows:
a. 24 monthly payments
b. 8 quarterly payments
c. 4 semi-annual payments
d. 2 annual payments

In case of lapse of two years without payment of the entire tax due, the
remaining cash balance thereof shall be due and demandable subject to the
applicable penalties and interest reckoned from the prescribed deadline for
filing the return and payment of tax.

Partial disposition
Some of the properties of the estate may be conveyed for cash
considerations to be used to settle the estate tax due. A written request for
partial disposition shall be approved by the BIR. The said request shall be
filed, together with a notarized undertaking that the proceeds thereof shall
be exclusively used for the payment of estate tax due.

In case of a failure to pay the total estate tax due out from the proceeds of
said disposition, the estate tax due shall be immediately due and
demandable subject to the applicable penalties and interest reckoned from
the prescribed deadline of filing of the return.

586

• d
f r 15 - Estate Tax Payable
01aPte
. bilitY for payment of the estate tax
pit state tax sha ll be paid by the e
file e i,eir o f h 1' s ct·1stn' b utive s harexecutor o. d
f h I a ministrator b c .
to allY d . . o t e est t e1ore delivery
. executors or a mm1strators all f a e. Where th _
11101 e f tax , o them shall b ere are two or
113Y111ent o . e severally liable for the

executor or administrator of an est t h


fhe b th . a e as the ·
the estate tax ut .e heir or beneficiary has pn~~ry o~ligation to pay
ayment of that portion of the estate which hi s ~bs1_d1a~y liability for the
~1. value of the total net estate. The ext t f s_ di~tnbutive share bears to
u,e d h I en o his habil'ty
1 h
case excee s t e va ue of his share in th . h . , owever, shall in
no e m entance.

·scharge of executor or administrator from personal r13b T


D1 1
he executor or administrator sha ll make . . •~
T . · f th a written apphcat10n for the
comm1ssT10hner o ; amoundt o~ t.h e estate tax and discharge from personal
liability. e execu or or a mm1strator, upon payment of th f
·1 t'f
I d sh II b d • h e amount o
which he .s no te ' a e isc arged from personal liability for any
deficiency m ~he tax t~ereafter found to be due and shall be entitled to a
receipt in writing showing such discharge.

No judge shall authorize the executor or judicial administrator to deliver a


distributive share to any party interested in the estate unless a certification
from the Commissioner that the estate has been paid is shown.

If, after the payment of the estate tax, new obligations of the decedent shall
appear, and the persons interested shall have satisfied them by order of the
court, they shall have a right to the restitution of the proportional part of the
tax paid.

587
Chapter 15- Estate Tax Payable

CHAPTER 15: SELF-TEST EXERCISES

Discussion Questions . fthe net tax estate.


1. Illustrate the computational format o f th 1·m1·ts '" '" .
~I
.
2. D1scuss t h e comput a ti·onal procedures o e 1 1orb1ore1gn t,,...,
.......es p .
3. Discuss the computational proce~~re of th e net taxa le estate used ~~cl.
computation of the foreign tax ere it. the 1·
th
4. Discuss the estate tax requirements, eir deadlines and conditio
compliance. ns fo~

Multiple-Choice - Theory

1. The taxable estate of the following includes all properties Wh


situated, except a erever
a. Resident citizen
b. Non-resident alien
c. Resident alien
d. Non-resident alien

2. Which of the following is norm ally ded uctible aga inst commo n properties,
a. Medical expenses
b. Standard deductions
c. Funeral expenses
d. Vanishing deductions
9.
3. Which is not cons idered in the compu ta ti o n of t he s ha re of the surviving
spouse?
a. Medical expenses
b. Family home
c. Obligatio ns
d. Sta ndard ded uctions 10

4. Which is incl uded in th e ne t taxable estate?


a. Family home in excess of Pl 0,000,000
b. Separate property of th e s urviving spouse
c. Amo unt paid for medical expenses of the decedent
d. Share of the s urviving spouse

5. Which is excluded in th e computation of the net taxable estale and in the


computation of th e net dis tribu table estate?
a. Shar e of the s urviving s pouse
b. Vanishing ded uctio ns
c. Es tate tax
d. Fam ily home

588
' 1s- Estate Tax Payable
/ ciaPter
Which is inclfufded_lin hnet taxable estate?
6- Excess of am1 y _ome fair value above Pl OM
8· Losses o properties occurring w·th ·
1
b, f • in 12 m h
Losses o p_ro pert1es occurring before ont s after death
c. Clairns agamst the estate death
d.

7 Sta
tement 1: No estate tax is due on an estate 'th
· properties. . wi PS,000,000 worth of
Statement 2: An estate with only a fa .1 h
property will not pay estate tax. mi Y ome worth PlS,000,000 as its

Which is correct?
a. Statement 1 c. Both statements
b. Statement 2 d. Neither statement

8. Statement
. 1: An estate
· d with
fi several properties b u t w1.th a negative
. taxable
estate 1s not reqmre to 1le an estate tax return.
Stateme~t 2: The ~IR shall be notified of the death of the decedent if he has
properties exceeding Pl,000,000.

Which is incorrect?
a. Statement 1 c. Both statements
b. Statement 2 d. Neither statement

9. Which of these is required to file an estate tax return?


a. An estate consisting solely of Pl00,000 shares of stocks.
b. An estate which includes a motor vehicle worth P200,000.
c. An estate with a real property worth Pl,000,000.
d. All of these

10. Which decedent cannot claim deduction for tax credit?


a. Resident citizen
b. Non-resident alien
c. Resident alien
d. Non-resident alien
11 Wh' • . d d · for the share of the surviving
· 1ch 1s correct regarding the e uc0 on
spouse? .
a o ft r ordinary deductions
• ne-half of the gross estate a e . h se arate properties of the
b. One half of the common properties plus t e P
surviving spouse .
c. One-half of the net common propertie~ . ouse gets of the common
d. It is the actual share which the Survivmg sp
Properties.

589
13. ACPA C1!rt1flcat1on Is required Jf the gross estate
a. ~ceeds Pl,000,000.
b. exceeds P2,000,0001 1
c. exceeds PS,000,000.
d contains registrable properties without regard to the V'alue f
estate. o the

14. A decedent died with a gross estate of P4,000,000. Which of the folloWin .
required? &ts
a. Notice of death
b. Estate tax return
c. CPA certification
d. AJI of these

15. The estate tax return shall be filed within


a. 30 days from the date of death.
b. 45 days from the date of death.
c. 6 months from the date of death.
d. 1 year from the date of death.

16. What is the maximum extension allowable in filing of the estate tax return?
a. 5 years c. 2 months
4.
b. 2 years d. 30 days

17. What is the maximum period of extension in paying the estate tax?
a. 5 years c. 6 months
b. 2 years d. 30 days

18. The estate tax return of a non-resident alien decedent is filed with
a. An accredited agent bank
b. Revenue district office 5. I

c. Collection agent
d. Office of the Commissioner

Multiple-Choice - Problems: Part 1

1. A single decedent died with the following data regarding his estate:
Family home
P14,000,000
Agricul tural land
6,000,000
Cash a nd other personal properties
Total 8,000,000
P28.ooo.ooo
590
tate Tax Payable
ter
1s- E5
cn3P _ ,.,,,d ~ a i n s t the estate:
~"~ P 400,000
¢t~ 11 expenses .

200,000
funr ' expenses
~•crl _iC31c:,qicnses
rltl t,l 1
' 300,000
300,000
111 ·d 1,,-:es
500.QQ.Q
11t11'il' 111 st th e estate
c1,11n1 ..ig<1 P 1.700,000
rotal
t he net taxable estate.
coJ11Pute
p 2,200,000 c. P 22,200,000
1
3• p zz,o00,000 d. P 27,200,000
b-
te the net distributa ble estate before estate tax dedu ction.
2. co111p1J
r 25 ,100,000 c. P 26,soo,ooo
:: p 26,300,000 d. P 27,200,000
The followi ng data relates to t h e estate of a si ngle resident decedent
3. without dependents:
Gross estate, including P15M family home P30,000,000
2,800,000
Ordinary deductions
What is the net taxable estate?
a. p 11,200,000 c. P 22,200,000
b. p 12,200,000 d. P 27,200,000

4. Romeo died leaving his wife, Juliet, the following net conjugal properties:

Gross estate, including P12M family home P28,000,000


Less: ordinary deductions 2.400.000
Net conjugal properties P25.600.000

Compute the net taxable estate.


a P 10,600,000 c. P 1,800,000
b. P 5,600,000 d. P 0
5- Capt Jeffrey Magiting, an unmarried Filipino, died in action leaving the
following net estate to his mother who is living with him:

Family home P12,000,000


0th er properties 8,000.000
Total properties P20,000,000
Less:
Funeral expenses P 300,000
Judicial expenses 200,000
Unpai'd medical expenses 600,000
CI · 1.500.000 2.600.000
Net aims against the estate
estate Pl?.400.000

591
Chapter 15 - Estate Tax Payable

Compute the net taxable estat;- P3,S00,000


a. P 12.900,000 d. P2 400 000
b. P 11,500,000 · ' '
. t 'bu table estate of Capt. Magi ting before deduct'1
6. Compute the net d 1s n 0n or

esta~e lt:x.200 000 c. p 17,400,000


000
~: p 1s:ooo:ooo d. P 16,400,
•d t citizen bachelor had a net distributable propert'1
7. Mr. X, a non-res1 en ' Th d . .b es of
P6,800,000 including a Pl,000,000 family home. e 1stn utable property
is net of the following:
P 250,000
Funeral expenses
150,000
Judicial expenses
600,000
Claims against the estate
Losses 200,000

Compute the net taxable estate.


a. p 7,200,000 C. p 1,200,000
b. P 2,200,000 d. P 0

8. Mr. Y died leaving the following properties and estate deductions:

Separate properties of Mr. Y Pl 0,000,000


Separate properties of Mrs. Y 24,000,000
Communal properties 26,000,000

Funeral expenses P 500,000


Other ordinary deductions of communal properties 8,000,000
Ordinary deductions - exclusive of Mr. Y 4,000,000
Ordinary deductions - exclusive of Mrs. Y 5,000,000
Family home - exclusive of Mrs. Y 6,000,000

Compute the taxable net estate of Mr. Y.


a. P 10,000,000 c. P 15,000,000
b. P 13,000,000 d. P 16,000,000

9. What is the distributable estate of Mr. Y?


a. p 15,000,000 C. P 13,900,000
b. P 14,150,000 d. P 13,360,000

10. Assuming instead that Mrs. Y died, compute her net taxable estate.
a. p 23,000,000 C. P 17,000,000
b. P 20,000,000 d. P 13,000,000

592
f ter 1
5- Estate Tax Payable
cna P
wnat is the distributable est ate of Mrs. y-,
11. p 26,9BO,ooo c. P 26,320."ooo
~·. p 26,730,000 d. P 25,480,000

cedent had a net estate of p 23 7SO 000


Ade d ' , after d d ·
1Z· . debtedness an taxes, property dis os e uctmg all P2,100,000
11~
50
ooo judicial expenses and funeral e P al losses of P400,000 and
itis fa~1ilY home worth P12,000,000. xpenses. Included in the esta~e was

What is the net taxable estate?


a. p 24,700,000 C. P 9,700,000
b. p 14,700,000 d. P 9,300,000

Itiple-Choice - Problems: Part 2


r.fU .
_ The fo]lowmg . to t he estate of a citizen decedent:
pe rtam
1
separate properties of the decedent Plz,400,000
common properties of the spouses is, 600,000
E,Q_ssible deductions:
Funeral expenses p 480,000
Judicial expenses 220,000
Obligations (1/4 is separate property) 1,800,000
Family home - common property 15,000,000

What is the taxable net estate?


a. P 6,800,000 C. p 6,550,000
b. P 6,575,000 d. P 4,300,000

2. Compute the net distributable estate.


a. Pl 9,550,500 c. P 17,375,000
b. Pl 9,030,500 d. P16,980,500
3. Asingle non-resident alien decedent died with the following gross estates
and deductions:
P 4,500,000
Tangible properties in the Philippines 500,000
Intangible properties in the Philippines
5,000,000
Tangible properties abroad £.10,000.000
Total gross estate
Possible deductions p 200,000
Funeral expenses 400,000
Judicial expenses 300,000
Medical expenses 500,000
Obligations (20% Philippines) 260,000
Losses 500,000
T hT ines)
ransfer for public use (property in the p 1 ipp

593
Chapter 15 - Estate Tax Payable

Compute the net taxable estate.


a. P 4,120,000 c. P 3,640,000
b. P 3,870,000 d. P 3,620,000

4. Assuming that the reciprocity rule applies, compute the net taxable
a. P 3,820,000 c. P 3,140,000 estate.
b. P 3,640,000 d. P 3,120,000

5. The following data pertains to the estate of a married non-resid


decedent: . ent al·ie~

Philippine properties:
Separate properties of the surviving spouse P30,000,000
Separate properties of the decedent 18,000,000
Common properties of the spouses 32,000,000

Total foreign properties, excluding PS,000,000


separate properties of the surviving spouse PS0,000,000

Possible deductions:
Funeral expenses P 1,800,000
Judicial expenses 1,200,000
Obligations - common fund 9,000,000
Family home - common property 12,000,000

What is the taxable net estate?


a. P 31,750,000 c. P 26,250,000
b. P 31,250,000 d. P 25,250,000

6. The following pertains to the properties of Mr. Servano at the point of his
death:

Cash P 1,000,000
Investment in stocks 8,500,000
Car 2,000,000
Agricultural land 15,000,000
Family home 13,000,000

Items ofdeduction:
Funeral expenses (withdrawn from the bank account
of Mr. Servano; subjected to 6% final tax) p 400,000
Judicial expenses 200,000
Claim against the estate 2,000,000
Claim against insolvent person
(uncol/ectible receivables) 500,000
Theft of cash (before Mr. Servano's death) 700,000
594
"" i s-Estate Tax Payable
-pter
Cll"
ervano is a bachelor who i'i ., 111 lJ .
i.fr ·S )ot t1111~.tll ti, ~
You11H<'r ,;1 hlt11g,;
,tr 1hr ,wt t,1x,,hl1• r"'·"•'
conir'
r .::2 !-OO oo o l P11<
l1lo ono
: r 21.. 1oo.noo c.. Pl., Aoo.ono

fr M<>tl. ,1 , .,,dent J.1p.111 sc cfti7 C'n, h,1(1 the· fn llow111,

Ph IliJ) p 111 cs
pc, JPull _ Tot"l
r;",""' ('\(,1f (' _ 0,000,000 p ~0.IHHl.000
o,x1,nar\' dc:-<lurt1011 PH0.000.0()0
16,000,000 l l.000,000
20.000.000
1am,lv homt\ Ph1llpp111c ·
~
8,000,000
8,000.000
utr th'-' nc>t tc1x,1hlc cst.itc in the Phil 1pp1ncs
'
G.omr
a p 27 '100,000 C. P 22,875,000 .
b p 2b,000,000 d. P 16,250,000

compute the net taxable estate abroad.


8
a. p 18.000,000 C. P 16,125,000
b. p 16,500,000 d. P 13,250,000

0 Asingle resident citizen died leaving the following estate and deductions:

Phiiippines China Taiwan Total


Family home P 1,300,000 P - P - P 1,300,000
Other properties 6,200,000 3,000,000 4,500,000 13,700,000
Tora! estate P 7,500,000 P 3,000,000 P 4,500,000 PlS.000,000
Loss P 300,000 P 150,000 P 100,000 P 550,000
Obligations 1.250,000 450,000 200,000 1.900,000
Tora! P 1,550,000 P 600,000 P 300,000 P 2,450,000
Estate before
Srandard deduction P 5,950,000 P 2,400,000 P 4,200.000 P12.550,000

Philippines China Taiwan Total


Estate tax paid p - p 100,000 P 160,000 P 260,000

Compute respectively the net taxable estate in the Philippines, China, and
Taiwan.
a P 5,950,000; P 2,400,000; P 4,200,000
b. P 4,65 0,000; p 2,400,000; P 4,200,000
C. p 3,450,000; p 1,400,000; p 2,700,000
d. P 2,150,000; p 1,400,000; P 2,700,000

lO. Compute the estate tax due and payable in the Philippines.
a. P 207,000 c. P 131,000
b. P 193,000 d. P 129,000

595
r
Chapter 15 - Estate Tax Payable

Multi le-Choice - Problems: Part 3


P d dent borrowed a PZ,000,000 frolll
1 On October 31, 2016, the edce e·annually every October 31. Theda lender
· which charges 1ooA interest
° C u ute the total ded uct1·bl e mdebtedne
· eced ent
died on January 31, 2018· omp c. P 2,050,000 ss.
a. P 2,2so.ooo d. P 2,000,000
b. p 2,200,000
. . mained unpaid before estate tax return
2. The fo llowing obligations re s Were
about to be filed: p 2so,0 00
Unpaid funeral expenses
400,000
Unpaid judicial expenses
200,000
Unpaid medical expense
Real property tax (3/4 unpaid before death) so,oo 0
Unpaid bank loan:
- Balance at date of death 150,00Q
Balance before filing of return 120,000

Compute the deductible indebtedness.


a. p 1,080,000 c. P 410,000
b. p 810,000 d. P 210,000

3. Decedent x received a P2,000,000 donations from his father on July 1, 2016.


He used the money to purchase 100,000 shares of listed company, Sam
Miguel Company. The shares of Sam Miguel Company traded as follows:
Open - P20, High - P 24, Low - P18 and Close - P23 at the time of X's death
on July 4, 2018. X have other assets of P12,900,000. X had P3,600,000 total
obligations upon his death. Compute the vanishing deduction?
a. p O C. p 912,QQQ
b. P 608,000 d. P 2,000,000

2. A single resident alien without a family home had net properties of


Pl2,000,000 after allowable deduction for ordinary deductions items
totaling P 8,000,000. 40% of the properties of the decedent were in the
Philippines.

Compute the estate tax due.


a. P 0 C. p 420,QQQ
b. P 240,000 d. P720,000

3. In the im~ediately preceding problem, compute the estate tax assuming the
decedent 1s a non-resident alien.
a. p 258,000 C. p 528 000
b. P 288,000 d. p 978,000

596
f pter 15
_ Estate Tax Payable
C~~ ··
·ied c1t1zen d I"edl eavmg
· the foll .
AJ11art owing net estate:
4
· sive properties of the d ecedent
Cl(ciU
i,,
0 11
.
properties o f t I1e spouses P s ,000,000
on101 d
C allowable e uct1on d . for LIT _ c 20,000,000
rota I bl . ommon 3,800,000
sfer for pu 1c use - exclusive
rran·shing de duct1ons
. - common 800,000
\fllJll 500,000
common properties include a family h
f he ome worth P12,00,000.
te the net taxable est ate
~ n1pu .
a p 6,950,000 C. p 7,150,000
b: p 7,050,000 d. P 7,250,000

In the immediately preceding problem com t th


5. surviving spouse in the net common pr~perti~~ ~f
1 the actual share of the
· • . , ere were expenses of
the death an d est a t e a d mm1stration costs totaling p400 000
a. p 7,650,000 C. p 7,900,000 , .
b. p 7,850,000 d. P 8,100,000

_ compute the distributive estate of the decedent.


6
a. p 12,750,000 C. p 12,327,000
b. p 12,500,000 d. P 12,077,000

7. The heirs of an unmarried decedent were too excited to determine their


share of inheritance. Details of the estate of the decedent were as follows:

Gross estate Pl 7,000,000


Total allowable deduction for LIT 1,800,000
200,000
Transfer for public use
4,500,000
Family home

There were P300,000 expenses for funeral and estate administration costs.
PlS0,000 of the funeral expenses were paid by friends and relatives.

Compute the net distributable estate.


a. P14,670,000 c. P 14,370,000
b. P 14,550,000 d. P 14,240,000
8 · l b ·1ct·ng
1 which
· On January 2 2017 Mr Andrews inherited a commercia UI .
was valued a; P30 000 000 in the estate tax return of the prelde7ce2s0so2r0. TThh1s
· ' ' d d' d on Ju Y , · e
ts the only property that remained when An rew ie P6 500 000 of which
Property was encumbered by a P12,000,000 mortgagtye, ha~ a i'onal value of
Was paid by Andrews before his death. The proper h. d th
P4o,ooo,ooo and a ppraisal value of pso,ooo,ooo upon IS ea .
597
C
Chapter 15 - Estate Tax Payable

• h"ng deduction.
Compute the vanis I c p 12,161,250
a. p 8,107,500 d. p 12 549,000
b. P 8,366,000 . '
(
9. Compute the estate tax.
c. p 1,868,040 ,.
a. P 1,040,325
d. p 1,628,040
b. P 1,283,550 ~
1
10 An American residing in Japan died leaving net p~operties of P4,000,0oo in 2
· .. . d pz 6 OOO 000 abroad. He paid Pl,300,000 estat
the Ph1hppmes an , , e~ 3
abroad. 4
Compute the estate tax payable in th e Philippines. 5
a. p 210,000 c. p 440,000 6
b. P 236,000 d. p 470,000 ,,,.1
11. Mr. Masiba died leaving the following properties:

Bank deposit
P 6,000,000 D
12,000,000 D
Business interest
Commercial building 20,000,000 (c
Family home, inclusive of P2,000,000 lot 14,000,000
E!
Additional information:
• The commercial building was purchased using donations received by 1.
Mr. Masiba during the marriage. The same building sustained a
Pl,000,000 fire loss shortly after Mr. Masiba died.
• The lot where the family home stands was received by Mrs. Masiba as
inheritance during the marriage.
• The commercial building was mortgaged to a bank for P2,SOO,OOO. Mr.
and Mrs. Masiba paid PS00,000 prior to Mr. Masiba's death. There were
Pl00,000 accrued interest at the death of Mr. Masiba. 2.
• The spouses were under the absolute community of property.
Compute the net taxable estate.
a. p 20,950,000 C. p 22,950,000
3,
b. P 21,950,000 d. P 23,750,000

4.

598

+
T , cnapte
iG-lntroduction to Donor's Tax

cJ-JAPTER 16
NfR-O=D=U=-=C_T=IO
=N_ T..,,.,.
O_ D= O=-N
= O=-R=='.::..S~T.:..:A:.:X_ _~ - - -
1/
~apter overview and Objectives:
C ---------------------------------------------------------------------------------------
..~~~ this chapter, readers a_re expected to:
A comprehend the esse ntial requisites and formal requisites of donation
1
· Appreciate the rationale and purpose of donor's taxation
;: Understand the types of donor's and their tax rules
4_ Understand the treatment of donation of common properties
S. Understand the rules on renunciation of inheritance
6. Understand the treatment of donations with joint or several donees
7. Master the list of exempt donations and the donor's tax format

DONATION
Donation is the gratuitous transfer of property from one living person
(donor) to another (donee).

ESSENTIAL REQUISITES OF DONATION


d by 1. Capacity of the donor
id a The donor must be legally competent to make a donation. A donation
made by a minor, an insane, or by one under hypnotic spells, force or
intimidation is unenforceable.
The donor's capacity shall be determined as of the time of making of the
Mr.
vere donation (Art 737, Civil Code).
2. Intention to donate
The donation must be intentional or voluntary. There is no such thing as
an implied donation.
3. Donative act or delivery
Donation is a real contract and is completed by the delivery of the
property to be donated.
4. Acceptance by the donee
No one shall be compelled to accept the generosity of another. The
donee has the prerogative to accept or reject the gratuity. The
acceptance of the donee perfects the contract of donation. The donation
is deemed perfected when the donor knows of the acceptance of the
donee.

599
Chapter 16 - Introduction to Donor's Tax

A transfer which does not manifest all of these attributes is not a donar
and will not be subject to tax. However, in transfers for insuffic/on
consideration, the acceptance by the donee is not a condition to taxation. ent

FORMAL REQUISITES OF DONATION


For a donation of properties to be valid, it must have to adhere to certa·
formalities required by law: in
Re uired fonnar
Public instrument*

Written
,000 Oral
Inta Public instrument

*Art. 749, New Civil Code; a public instrument is a written document annotated by a lawyer.

The acceptance of a donation of real properties may be made in the same


deed of donation or in a separate public document, but it shall not take
effect unless it is done during the lifetime of the donor. If the acceptance is
made in a separate instrument, the donor shall be notified thereof in an
authentic form, and this step shall be noted in both instruments (Art 749,
New Civil Code).

An oral donation requires the simultaneous delivery of the thing or of the


document representing the right donated (Art. 748, New Civil Code).
A donation which does not conform to these legal forms is not valid and is
unenforceable and therefore not subject to tax.

TYPES OF INTER-VIVOS DONATION


1. Direct donation
A direct donation is one made by the donor directly to the donee.

2. Indirect donation
An indirect donation involves transfer of property by the donor in favor
•of the donee but under the supervision of another party. This is called
donation in trust.

The designation of a donation in trust may either be:


a. Revocable - this is not a completed donation and is not taxable.
b. Irrevocable - this is a completed donation; hence, taxable.

600
VY
f r iG- Introduction to Donor's Tax
cnaPte
. on 1 - Revocable donation
stratl
Jl}ll tor made a revocable donation f .
rJr, Gran of his son, Melvin. The boardi~ a boarding house worth Pl0,000,000
ill favor tion. Mr. Grantor specified that g house shall be n:1anaged by Trustee
corpora sferred to Melvin Th t P200,000 annual mcome of the trust
hall be tran · e ru stee made payment of P200,000 to Melvin
s . g the year.
dunn
000 000 revocable donation 0 r b d'
fhe P10' ' . 'J a oar mg house is not subject to donor's
he P200,000 transfer of mcome 0,r th
taX, T M G t 1 e property to Melvin is a taxable
·on 0 r r. ran or.
dona tl 'l

Illustration 2 - Irrevoca~le donation


Assuming the same facts m the preceding problem, except that the donation is
designated by Mr. Grantor as irrevocable.

The Pl0,000,000 irrevocable ~onation of a boarding house is subject to donor's


Yer, tax. The P200,000 transfer of mcome of the property to Melvin is a deduction to
the income of the taxable trust The same shall be treated as income of Melvin and
11lle
isnot a donation subject to donor's tax.
ake
e is TYPES OF DONORS
1 an A. Resident or citizen - taxable on global donations, such as
749 I 1. resident citizen
2. non-resident citizen
3. resident alien
the
8. Non-resident alien - taxable only on Philippine donations, except
intangible personal property subject to reciprocity conditions
idis
Reciprocity rule for non-resident aliens
The Philippines exempts donations of intangible personal property by non-
resident alien donors if their country also exempts the donations of
intangible personal properties by Filipino non-residents therein.

Donees
The NIRC required classification of donees between relatives and strangers.
·avor Donation to relatives are then subject to a progressive tax while donation to
ailed strangers are then subject to 30% tax.

Under the TRAIN law there is no need to classify donees as donation to


any donee is now subject to a Oat 6% tax.

601
Chapter 16- Introduction to Donor's Tax

Summary of rules on taxable donation


Residents or NRA without

✓ ✓ ✓ X ✓
T
✓ ✓ ✓ X X

Illustration 1
Mr. Kumar donated the following properties:
Philippines Abroad
Real properties P 1,000,000 p 800,000 ~
P 1,soo,
000
Tangible personal properties 400,000 300,000 700,0 00
200,000 100,000
Intangible personal properties
p ~
Total P 1,600.000 1.200,000
~
The following shall be the taxable donation in each of the following cases:

If Kumar is a The taxable donation is


Resident or citizen donor P 2,800,000
Non-resident alien
With reci P 1,600,000
Without P 1,400,000

Illustration 2
A non-resident Filipino working abroad donated a sports car abroad worth
P12,000,000 to his American best friend.

This is subject to Philippine donor's tax since citizens are taxable on global
donations.

Illustration 3
Mr. Coolaz, a Canadian tourist, left his scuba diving kit worth P200,000 as
donation to his Pilipino tour guide in the Philippines.

This is subject to Philippine donor's tax since the property is located in the
Philippines at the time of donation. The reciprocity rule does not apply since the
property is a tangible property.

Illustration 4
Gret's future father in law, a non-resident Chinese, informed her that he is
donating $1,000,000 for Gret's. Gret gratefully accepted the donation.

602
l6 - Introduction to Donor's Tax
cttapter
.. not subject to Philippine donor's tax since the donor ts a non -resident alien
15
fh 1 property is not located in the Philippines at the time of donation.
and the
oNOR'STAX
V , taxi a tax upon the gratuitous transfer of property between two or
oonorliving persons at the time of transfer whether the transfer is direct or
fllore t and without regard to the type of prope rty transferred.
in trU
NATURE OF DONOR'S TAX
privilege tax - donor's tax is a tax upon the privilege to transfer
1
· property gratuitously during the lifetime of the donor
proportional tax - donor's tax is based on a fixed percentage of net gift
z.3. Annual tax - donor's tax is imposed on yearly net gifts of donors in
excess of P250,000
4. Ad valorem - donor's tax depends upon the value of the property
donated
5, National tax- donor's tax is imposed by the national government
6. Revenue or fiscal tax - donor's tax is intended to provide the
government income

RATIONALE OF DONOR'S TAXATION


1. To control tax evasion of the estate tax
If there is no tax on donation, a person may transfer his properties while
he is still living to avoid the estate tax. The donor's tax is partly intended
to minimize tax evasion on estate tax.

2. To control tax evasion on income tax


Sellers of goods and properties may intentionally set the selling price
below the fair value for some personal reasons. The gratuity is an
unrealized benefit which will not be taxed under income taxation. The
donor's tax supports income tax by taxing the gratuity.

Illustration
Mr. A had an art collection item with a fair value of P4,000,000 which he
previously acquired for P2,000,000. Mr. A sold the collector's item for only
P2,SOO,OOO.
Fair market value P 4,000,000
} P 1,500,000 - donation
Selling price P 2,500,000
} P 500,000- income
Cost or tax basis P 2,000,000

603
T , cnapte
iG-lntroduction to Donor's Tax

cJ-JAPTER 16
NfR-O=D=U=-=C_T=IO
=N_ T..,,.,.
O_ D= O=-N
= O=-R=='.::..S~T.:..:A:.:X_ _~ - - -
1/
~apter overview and Objectives:
C ---------------------------------------------------------------------------------------
..~~~ this chapter, readers a_re expected to:
A comprehend the esse ntial requisites and formal requisites of donation
1
· Appreciate the rationale and purpose of donor's taxation
;: Understand the types of donor's and their tax rules
4_ Understand the treatment of donation of common properties
S. Understand the rules on renunciation of inheritance
6. Understand the treatment of donations with joint or several donees
7. Master the list of exempt donations and the donor's tax format

DONATION
Donation is the gratuitous transfer of property from one living person
(donor) to another (donee).

ESSENTIAL REQUISITES OF DONATION


d by 1. Capacity of the donor
id a The donor must be legally competent to make a donation. A donation
made by a minor, an insane, or by one under hypnotic spells, force or
intimidation is unenforceable.
The donor's capacity shall be determined as of the time of making of the
Mr.
vere donation (Art 737, Civil Code).
2. Intention to donate
The donation must be intentional or voluntary. There is no such thing as
an implied donation.
3. Donative act or delivery
Donation is a real contract and is completed by the delivery of the
property to be donated.
4. Acceptance by the donee
No one shall be compelled to accept the generosity of another. The
donee has the prerogative to accept or reject the gratuity. The
acceptance of the donee perfects the contract of donation. The donation
is deemed perfected when the donor knows of the acceptance of the
donee.

599
Chapter 16 - Introduction to Donor's Tax

A transfer which does not manifest all of these attributes is not a donar
and will not be subject to tax. However, in transfers for insuffic/on
consideration, the acceptance by the donee is not a condition to taxation. ent

FORMAL REQUISITES OF DONATION


For a donation of properties to be valid, it must have to adhere to certa·
formalities required by law: in
Re uired fonnar
Public instrument*

Written
,000 Oral
Inta Public instrument

*Art. 749, New Civil Code; a public instrument is a written document annotated by a lawyer.

The acceptance of a donation of real properties may be made in the same


deed of donation or in a separate public document, but it shall not take
effect unless it is done during the lifetime of the donor. If the acceptance is
made in a separate instrument, the donor shall be notified thereof in an
authentic form, and this step shall be noted in both instruments (Art 749,
New Civil Code).

An oral donation requires the simultaneous delivery of the thing or of the


document representing the right donated (Art. 748, New Civil Code).
A donation which does not conform to these legal forms is not valid and is
unenforceable and therefore not subject to tax.

TYPES OF INTER-VIVOS DONATION


1. Direct donation
A direct donation is one made by the donor directly to the donee.

2. Indirect donation
An indirect donation involves transfer of property by the donor in favor
•of the donee but under the supervision of another party. This is called
donation in trust.

The designation of a donation in trust may either be:


a. Revocable - this is not a completed donation and is not taxable.
b. Irrevocable - this is a completed donation; hence, taxable.

600
VY
f r iG- Introduction to Donor's Tax
cnaPte
. on 1 - Revocable donation
stratl
Jl}ll tor made a revocable donation f .
rJr, Gran of his son, Melvin. The boardi~ a boarding house worth Pl0,000,000
ill favor tion. Mr. Grantor specified that g house shall be n:1anaged by Trustee
corpora sferred to Melvin Th t P200,000 annual mcome of the trust
hall be tran · e ru stee made payment of P200,000 to Melvin
s . g the year.
dunn
000 000 revocable donation 0 r b d'
fhe P10' ' . 'J a oar mg house is not subject to donor's
he P200,000 transfer of mcome 0,r th
taX, T M G t 1 e property to Melvin is a taxable
·on 0 r r. ran or.
dona tl 'l

Illustration 2 - Irrevoca~le donation


Assuming the same facts m the preceding problem, except that the donation is
designated by Mr. Grantor as irrevocable.

The Pl0,000,000 irrevocable ~onation of a boarding house is subject to donor's


Yer, tax. The P200,000 transfer of mcome of the property to Melvin is a deduction to
the income of the taxable trust The same shall be treated as income of Melvin and
11lle
isnot a donation subject to donor's tax.
ake
e is TYPES OF DONORS
1 an A. Resident or citizen - taxable on global donations, such as
749 I 1. resident citizen
2. non-resident citizen
3. resident alien
the
8. Non-resident alien - taxable only on Philippine donations, except
intangible personal property subject to reciprocity conditions
idis
Reciprocity rule for non-resident aliens
The Philippines exempts donations of intangible personal property by non-
resident alien donors if their country also exempts the donations of
intangible personal properties by Filipino non-residents therein.

Donees
The NIRC required classification of donees between relatives and strangers.
·avor Donation to relatives are then subject to a progressive tax while donation to
ailed strangers are then subject to 30% tax.

Under the TRAIN law there is no need to classify donees as donation to


any donee is now subject to a Oat 6% tax.

601
Chapter 16- Introduction to Donor's Tax

Summary of rules on taxable donation


Residents or NRA without

✓ ✓ ✓ X ✓
T
✓ ✓ ✓ X X

Illustration 1
Mr. Kumar donated the following properties:
Philippines Abroad
Real properties P 1,000,000 p 800,000 ~
P 1,soo,
000
Tangible personal properties 400,000 300,000 700,0 00
200,000 100,000
Intangible personal properties
p ~
Total P 1,600.000 1.200,000
~
The following shall be the taxable donation in each of the following cases:

If Kumar is a The taxable donation is


Resident or citizen donor P 2,800,000
Non-resident alien
With reci P 1,600,000
Without P 1,400,000

Illustration 2
A non-resident Filipino working abroad donated a sports car abroad worth
P12,000,000 to his American best friend.

This is subject to Philippine donor's tax since citizens are taxable on global
donations.

Illustration 3
Mr. Coolaz, a Canadian tourist, left his scuba diving kit worth P200,000 as
donation to his Pilipino tour guide in the Philippines.

This is subject to Philippine donor's tax since the property is located in the
Philippines at the time of donation. The reciprocity rule does not apply since the
property is a tangible property.

Illustration 4
Gret's future father in law, a non-resident Chinese, informed her that he is
donating $1,000,000 for Gret's. Gret gratefully accepted the donation.

602
l6 - Introduction to Donor's Tax
cttapter
.. not subject to Philippine donor's tax since the donor ts a non -resident alien
15
fh 1 property is not located in the Philippines at the time of donation.
and the
oNOR'STAX
V , taxi a tax upon the gratuitous transfer of property between two or
oonorliving persons at the time of transfer whether the transfer is direct or
fllore t and without regard to the type of prope rty transferred.
in trU
NATURE OF DONOR'S TAX
privilege tax - donor's tax is a tax upon the privilege to transfer
1
· property gratuitously during the lifetime of the donor
proportional tax - donor's tax is based on a fixed percentage of net gift
z.3. Annual tax - donor's tax is imposed on yearly net gifts of donors in
excess of P250,000
4. Ad valorem - donor's tax depends upon the value of the property
donated
5, National tax- donor's tax is imposed by the national government
6. Revenue or fiscal tax - donor's tax is intended to provide the
government income

RATIONALE OF DONOR'S TAXATION


1. To control tax evasion of the estate tax
If there is no tax on donation, a person may transfer his properties while
he is still living to avoid the estate tax. The donor's tax is partly intended
to minimize tax evasion on estate tax.

2. To control tax evasion on income tax


Sellers of goods and properties may intentionally set the selling price
below the fair value for some personal reasons. The gratuity is an
unrealized benefit which will not be taxed under income taxation. The
donor's tax supports income tax by taxing the gratuity.

Illustration
Mr. A had an art collection item with a fair value of P4,000,000 which he
previously acquired for P2,000,000. Mr. A sold the collector's item for only
P2,SOO,OOO.
Fair market value P 4,000,000
} P 1,500,000 - donation
Selling price P 2,500,000
} P 500,000- income
Cost or tax basis P 2,000,000

603
r 16 - Introduction to Donor' Tax
apte

ft th t i In mpl t b au of r -'rved powers becomes complete


· n ei th r :
'"<hfhr d 11 r r noun , th pow r; r
8· thi ri ht t . h
, r 1 t r erv d pow r ceases because of the
b, haPP nin of ome v nt or continge ncy or the fulfillment of some
nditio n, oth r than be a u of the donor's dea th

111u rration
Q 111 ade a re cabl don tion of a car to her da ughter H. The car was worth
-· 0 ooo wh n delivered to H.
PL40 I

f\rn if the car i delivered, th~re is no d~nation in this case since Mrs. Q still owns
car. Th value of the car 1s not sub1ect to donor's tax on delivery. The same
all be taxed only when Mrs. Q renounces the power to revoke.
,4sSUming Mrs. Q died without revoking the car, the same shall be subject to estate

ruination to the ~ovemment for public use


Gifts made to or for the use of the national government or any entity created
by any of its agencies which are not conducted for profit or to any political
subdivision of the said government are exempt from donor's tax.

illustration
Mr. Escala donated P200,000 worth of goods to the Land Bank of the Philippines
and PZ00,000 to the Dep-Ed.

The donation to Dep-Ed is exempt being a government agency. The donation to


land Bank which is a government-owned and controlled corporation is subject to
t.ax.

Donation to accredited non-profit organization


Gifts in favor of an educational and or charitable, religious, cultural or social
welfare corporation, institution, accredited nongovernment organization,
trust, or philantrophic organization or institution are exempt from donor's
tax (See Sec. 101 (A) (3), NIRC).

Requisite for exemption


1. Not more than 30% of said gift shall be used by such donee for
administrative purposes.
2- The donee entity must be organized as a non-stock entity.
3- The donee entity does not pay dividends.
4· The do nee entity's board of trustees earns no compensation.

609
Chapter 16 - Introduction to Donor's Tax

· is subject to donor's tax to


The p 1 500 000 indirect donatwn
I I
control
ta~
evasion on the income tax.

3. To recoup future loss of income tafx revenutye will cau th


Similar to succession a donation
'
°
pro~er . . se e spread
· t O ultiple md1v1dual taxpayers Th· of
income-generating properties m . · Is Will
.
cause a re d uct10n m . future income tax. The transfer
. 1s subjected to
d onor ,s tax to recoup f u ture loss of government mcome tax revenue .

EXEMPT GIFTS
The following are exempt donations: .
. t
1. Donat 10ns o exemp t donees under the NIRC and special laws
2. Donations for election campaign
3. Transfers for insufficient consideration involving real property
classified as capital assets
4. General renunciations of inheritance
5. Donations with reserved powers
6. Donation to the government for public use
7. Donation to accredited non-profit insitution
8. Quasi-transfers
9. Void donations
10. Foreign donations of non-resident alien donors
11. Donations of property exempt under reciprocity
~
illlssi fi ed.
Donation to certain exempt donees under the NIRC and special laws The gratw
Donations to the foll owing donee entities are exempt:
transfers)
1. Aquaculture Department of the Southeast Asian Fisheries Development
income ta)(
Center (Sec. 2, P.D. 292)
2. Aurora Pacific Economic Zone and Freeport Authority (Sec. 7, R.A. No. However,
10083) The sale,
3. Development Academy of the Philippines (Sec. 12, PD 205) capital ass
4. Girl Scouts of the Philippines (Sec.11, R.A. No. 10073) gross sell i1
5. Integrated Bar of the Philippines (Sec. 3, PD 181) This inco n
6. International Rice Research Institute (Art. 5(2), PD 1620) lower tha r
7. National Commission for Culture and the Arts (Sec. 35, R.A. No. 10066) taxevasi o,
8. National Social Action Council (Sec. 4, P.D. 294) need to in1
9. National Water Quality Manageme nt Fund (Sec. 9, R.A. No. 9275)
It must h
10. People's Television Network, Inco rporated (Sec. lS, R.A. No. 10390)
11. People's Survival Fund (Sec. 13, R.A. No_ 10174)
subiect tc,'
12. Philippine-American Cultural Fo undat' (.S Henc
.. . ion ec. 4, P.D. 3062)
13. Ph1 hppme Normal University (.Sec 7 RA N a ~. th,
· , · · 0. 9647) b. S l <

604 . Sa1,.
r
ter 16 - Introduction to Donor's T
cnaP ax
Philippine Investors Commission (S 9
14- . d ec. , R.A. No. 3850)
. philippme Re Cross (Sec. 5, R.A. No. 10072)
15
Ramon Magsaysay Award Foundation (S 2 R
16· h ec. , .A. 3676)
7. Rural Farm Sc ool (Sec. 14, R.A. No. 10618)
1
18. Task Force on Human Settlements (Sec. 3(b)(8), E.O. 419)
Tubbataha Reefs Natural Park (Sec 1 7 RA N
19· . .. · , · · 0. 10067)
_ University of the Ph1hppines (Sec. 25, R.A. No. 9500)
20

The list of exempt donee~ herein is non-exhaustive. There may be other


exempt donees under special laws.

oonations for election campai 2n


w contributio_n in cash or in kind to any candidate, politicai party, or
coalition of parties for campaign purposes shall be governed by the Election
Code, as amended. Exemption is not automatic. These donations must be
reported to the Commission on Elections to be exempt from donor's tax.

Supporting political candidates or parties by making campaign donation to


them during elections is an exercise of one's political freedom - a
constitutionally vested right. Hence, it is not subject to taxation.

Transfers for insufficient consideration involvin1: real property


classified as capital assets
The gratuitous portion of transfers for insufficient consideration (complex
transfers) is subject to donor's tax. This is intended to control tax evasion on
,pment income tax when the selling price is intentionally set at a lower amount.

However, this rule does not apply to the sale of real property capital asset.
'. A. No.
The sale, exchange and other disposition of real property classified as
capital asset is subject to a capital gains tax of 6% based on fair value or
gross selling price, whichever is hi~her.
This income tax scheme has a flexible tax base. If the selling price is set
lower than the fair value, the fair value is taxed. There could be no income
tax evasion to arise from manipulation of the selling price. Hence, there is no
need to impose the donor's tax.
It must be noted, however, that exemption applies only to real properties
subject to the 6% capital gains tax.

Hence, the exemption does not extend to: .


a. Sale of real properties classified as ordinary asset
b. Sale of personal or movable property

605
. Donor's Tax
Chapter 16 - Introduction to

Illustration
Id the following properties:
A corporat10n so
Fair value Selling price
p 2,000,000 P 1,000,000
Vacant and unused lot 4,000,000 2,500,000
Building 1,000,000 400,000
Investment in shares of stocks 200,000
400,000
Equipment
, h II be computed as:
The donation and the donor s tax s a
P 1,500,000
Building (P4,000,000- P2,S00,OOO) ~ 600 000
4
Investment in stocks (Pl,000,000 - P 00,000 zoo'.ooo
Equipment (P400,000- P200,000} p 2 300 000
Net taxable gifts . ,zso'.ooo
Less: Exempt donat10n p Z,050,000
Excess , 6%
Multip,ly by: Donors tax rate p 123,000
Donors tax
Note: h 01 ·t I ·
1. The sale of the vacant and unused lot is subject to t e 6 ;,o cap1 a gams tax on real
because it is an onerous disposition.
2. The building and equipment are properties used in business which are ordinary assets
rather than capital assets. . . .
3. The investment in stocks is a personal property subJect to the 15 0Yo capital gams tax I
where actual gain is taxed, not the 6% capital gains tax.
[
f
Insufficient consideration on transfer of other properties
e
For other properties, the insufficient consideration is a gift to the extent of
the difference between the fair value and the selling price of the property 1
.disposed of. However, in the absence of a donative intent, the same shall be a
exempt. The donor shall have the burden of proving the absence of an ti
intention to donate.
II
Illustration 1 D
Mr. Ekkon had a car with a fair value of P4,000,000. His brother Juan is B
interested to buy the car but could only offer P2,500,000. Another friend offered
P3,800,000 for the car but Ekkon preferred to sell the car to Juan.

The ~1,500,000 discount is a taxable gift. The donation is clearly intentional since
d~sp1te the pres~nce of a willing buyers at almost fair value Mr. Ekkon decided to
give the car to his brother. '

Illustration 2
Liam Mado is having a business liquidity rob! . . . ntly
maturing debt which he could P em. Faced with an 1mmme
value of P3,000,000 for only Pl,~~~~;6~~, he sold his truck with a second ha nd

606
cnaPter 16- Introduction to Donor's Ta x
P2 000,000 discount is not a tax bl .
rhe '. ce the sale is forced b . a e gift there being total lack of donative
;11 tent sin Ya circum stance beyond the Liam Mado's control.
~ Renunc~at~on o~ .l nheritan~
Ageneral r enunc10t1011 of mheritance occurs wh h . th ..
nounces his or h h . . en an eir or e surv1vmg
spouse re . er s ai e 111 the hereditary estate of a decedent in
of no particular coheir A g l . .
favo r . h .· enera renunc1at10n is a repudiation of
1
inheritance w uc cannot be imputed as a donation.
()lustration_ .
oon Juan died wi th a . net di~tributable estate of Pl,200,000 for his children
Clara, Bre nd a, a nd Cedie as heirs. Cedie renounces his P400 000 share in the net
1

estate in favor of no particular assignee. Thus, Cedie's P400, 000 is reallocated as


pz00,000 each to Clara and Brenda.

The renunciation made by Cedie is a general renunciation. In the first place,


Cedie did not_accep~ ownership of his share; hence, the reallocation of his would-
be share to his coheirs Clara and Brenda cannot be considered as a donation. This
reallocation is exempt from donor's tax.

Important point to consider


To be exempt, the renunciation of inheritance must not be done
categorically in favor of an identified heir to the exclusion of other heirs
(RR2-2003).

Illustration 1: Specific renunciation in cases of more than two heirs


Don Juan died with a net distributable estate of Pl,200,000 for heirs: Clara,
Brenda and Cedie. Cedie renounced his P400,000 share in favor of Brenda to the
exclusion of Clara.
This is called specific renunciation. The specific renunciation operates as if Cedie
accepted his inheritance from Don Juan and then transferred his ownership
thereto to Brenda. This is a constructive donation subject to the donor's tax.

Illustration 2: Specific renunciation in the case of only two heirs


Don Juan died with a net distributable estate of PB00,000 for his two heirs:
Brenda and Cedie. Cedie renounced his P400,000 share in the net distributable
estate.
Note that in this particular case, the effect of specific renunciation by Cedie will
produce the same effect as a general renunciation. Either way, . Brenda w_ill still
receive Cedie's P400 000 share to the exclusion of no other he1r/s. In this case,
even if the renuncia~ion is specific, it is not construed as a donation. Hence, it is
exempt from donor's tax.

Renunciation by the surviving spouse . . .


The renunciation by the surviving spouse of his/her s~are m the hereditary
~ of the decedent will be subject to the aforementwned rules.
607
Chapter 16- Introduction to Donor's Tax

However, the renunciation by the surviving spouse of his/her sha .


. d. I t·10 f re in h
net conjugal or communal properties. upon
. . isso "f u n °
the rnarr·iage _t e
taxable donation regardless of whether it 1s spec1 1c or general. Is a

Illustration
Mr. Y died with three heirs: his spouse, son Sand daughter D. Mr. Y indic
his last will and testament that his estate shall be equally distributed arnoatect in
heirs. ng the

The following is a breakdown of Mr. Y's net distributable estate:


Common
Separate
~
Gross estate P 1,400,000 P 3,600,000 P s,ooo, 000
1,600,000
Deductions 400,000
P 2,000,000 ~
Net estate after deductions P 1,000,000 P 3,000,0 00
Less: Share of surviving spouse (P2M/2) 1,000,000
Estate tax 24s,oo 0
Net distributable estate P 1,755,QDil
-
Thus, Mrs. Y shall receive PSBS,000 (Pl,755,000+3) in the net distributable
estate.
If Mrs. Y renounces her PSBS,000 share, the same shall be taxable if the
renunciation is specific, but exempt if the renunciation is general.
[IIU!
If Mrs. Y renounces her Pl,000,000 share in the common properties in favor of her
Mr. I
children, the same is a taxable donation regardless of whether the renunciation is
specific or general. Note that this portion is owned by Mrs. Y.
and

Summary of rules on renunciation The 1


land
Type of renunciation General Specific tax,
Renunciation with more than 2 heirs Exempt Taxable
Renunciation with only 2 heirs Exempt Exempt
Renunciation by the surviving spouse of his
!mru
Gifts
share in the common properties Taxable Taxable
Welfa
trust
Do_nation ~ith reserved powers (Incomplete transfers) J

This pertams to t_ransfers of property wherein ownership will transfer only tax (S
upon the happenmg of a future event which is specified by the donor, such
as: Requ,
1. Conditional donation t N(
2. Revocable transfers ad
l, Th
The donor's tax shall not apply unless and until there is a completed gift J, Th
(RR2-2003). Incomplete transfers are not subject to tax upon delivery of the 4, Th
property. They are taxable upon completion and perfection of the donation.

608
r 16 - Introduction to Donor' Tax
apte

ft th t i In mpl t b au of r -'rved powers becomes complete


· n ei th r :
'"<hfhr d 11 r r noun , th pow r; r
8· thi ri ht t . h
, r 1 t r erv d pow r ceases because of the
b, haPP nin of ome v nt or continge ncy or the fulfillment of some
nditio n, oth r than be a u of the donor's dea th

111u rration
Q 111 ade a re cabl don tion of a car to her da ughter H. The car was worth
-· 0 ooo wh n delivered to H.
PL40 I

f\rn if the car i delivered, th~re is no d~nation in this case since Mrs. Q still owns
car. Th value of the car 1s not sub1ect to donor's tax on delivery. The same
all be taxed only when Mrs. Q renounces the power to revoke.
,4sSUming Mrs. Q died without revoking the car, the same shall be subject to estate

ruination to the ~ovemment for public use


Gifts made to or for the use of the national government or any entity created
by any of its agencies which are not conducted for profit or to any political
subdivision of the said government are exempt from donor's tax.

illustration
Mr. Escala donated P200,000 worth of goods to the Land Bank of the Philippines
and PZ00,000 to the Dep-Ed.

The donation to Dep-Ed is exempt being a government agency. The donation to


land Bank which is a government-owned and controlled corporation is subject to
t.ax.

Donation to accredited non-profit organization


Gifts in favor of an educational and or charitable, religious, cultural or social
welfare corporation, institution, accredited nongovernment organization,
trust, or philantrophic organization or institution are exempt from donor's
tax (See Sec. 101 (A) (3), NIRC).

Requisite for exemption


1. Not more than 30% of said gift shall be used by such donee for
administrative purposes.
2- The donee entity must be organized as a non-stock entity.
3- The donee entity does not pay dividends.
4· The do nee entity's board of trustees earns no compensation.

609
'T
. t Donor's Tax c~
Chapter 16 - Introduction o
t devote all its income, donations, subsict• 1111
5. The donee entity_lmu~ PY to the accomplishment and promotio Jes, Ot &J
other forms of phi ant. r~ A tides of Incorporation. n Of its re~
purposes enumerated m its r fO
. d e entities are accredited by the applicable governh-,1
oo
In practice, .one . . t·wns are exempt. Donati ·• ent
dited donee mst1tu
agency. Donat10ns to accre . ons to po
non-accredited donee institutwns are taxable. 5Ul

Accrediting Agencies . . . 6f
Previously, the accreditation of donee mS t itUtiOns wa~ handled by th p,s
Philippine Council for NGO Certification (PCNC). Effective ~ovember 16e (A
2007, the accreditation function was transferred by Executive Order No' JlO
671 to the following government entities: · [111
1. Department of Social Welfare and Development - for charitable and 0
social welfare organizations, foundations and associations including bu; on
not limited to those engaged in youth, child, women, family, disabled DO
persons, older persons, welfare and development do
2. Department of Science and Technology - for research and other pe
scientific activities
3. Philippine Sports Commission - for sports development Ilh
4. National Council for Culture and Arts - for cultural activities Eu
5. Commission on Higher Education - for educational activities ITT€
ani
Illustration 1 - Donation to an accredited NPOs
Mr. Isidoro donated PS00,000 to BIBAK, a non-profit cultural organization
established in the Cordilleras. Th
sul
If B1B1f! is a do~ee cultural organization accredited by the appropriate
accred1tmg agency (1.e., the National Council for Culture and Sports) the donation
shall be exempt The net gift shall be nil. ' ~
Qu
If BIBAK is a non-accredited donee or9 a . . rei
The net gift shall be PS00,000. mzatwn, the donation shall be taxable.
ta)

Illustration 2 - Direct donation to benefi . . . Ex;


Don Ambrocio was so touched b T c1aries ~f aids 1.
Kapamilya Foundation regard ' y a V ad which is sponsored by Sagip
" . mg a parent h O · .
,or the medication of his son h . w IS seekmg financial assistance 2,
w O 1s under t · . •
proceed ed to the hospital and d ermmal illness Mr AmbrocJO
onated P2 000 0 · ·
' , 00 to the parent.
The P2,000,000 donation shall b b. 3,
d t' e su '}ect to d ,
ona t~n must be given to an ac d ' onor s tax. In order to be exempt the
K~pamilya Foundation in our exam;;: ;;d non -profit organization like Sagip
re ease the same to the beneficiary. . e non-profit organization shall in turn

610
l6 - Introduction to Donor's Tax
cnaPter
tion 3 - Transfer of donation by NPOs
st
I11t1 ra ·
foundat10n . d P2 ,000 ,00 0 contnbut1on
receive · · ·
from Don Juanico which he
st
Bagi~ ted for the re-con ruction of the houses of five families Eagle
restr~ation disbursed th e PZ,000,000 contribution according to the wishes of
faun
J
uanico.
van
Juanico shall not be subject to donor's tax. Eagle Foundation shall not also be
D0~ t to donor's tax.
s11b1ec
tuitoUS Donations to Associations
GrO .
A5sociat10ns dO no t qua 1·fy
1 as exempt donee institutions under Sec. 101
(A)(3) of the NIRC. Hen~e, endowmen~ or gifts received by associations are
t exempt from donors tax. All donat10ns to associations for tax purposes
: : st be covered by a donor's tax return (RMC 53-2013).

onerous donations to associations


Donations with an oner ous ca use are not in the nature of an endowment or
donation. They are in the concept of a fee or price in exchange for the
0 ther
performance of a service, use of property, or delivery of a n object. (Ibid)

Illustration
Europa Condominium Home Owners Association accepts donations from
members in exchange for stickers for the right of way in t heir subdivision road
and toll fees for the maintenance of the road.

ation / The The donation in this case is not subject to donor's tax. However, it may be
subject to income tax and VAT or percentage tax (See RMC 9-2013).
riate
1tion I Quasi-transfers
Quasi-transfers involve delivery of property to another person but will never
results in transfer of ownership thereto. These are not subject to donor's
rble. tax.
Examples:
1. Merger of the usufruct in the owner of the naked title during the lifetime of
gip the usufructuary
ice 2. The transmission or delivery of the inheritance or legacy by the fiduciary
heir or legatee to the fideicommissary during the lifetime of the fiduciary
heir
3. The transmission from the first heir, legatee, or donee during his lifetime in
:: I
ip I
favor of another beneficiary, in accordance with the desire of the
predecessor
·n

611
Chapter 16 - Introduction to Donor's Tax

Void donations . .
Void donations are invalid donations .. Vmd_ donati~ns include t
th ~0se
P rohibited by law and those with defects m e1r execution. Void do natio
are not objects of taxation. ns
Prohibited donation under the Civil Code:
1. Donation between spouses, except minor ?ifts
2 Donations between persons who were gmlty of adultery or concub·
. m~
at the time of donation
Criminal conviction for adultery or concubinage is not material to h
disqualification of the donee. Hence, a d onatwn
.
°t
a common-law Wif
· Of a I ega II Y marne
. d man Is
t e
· held Void
e as
beneficiary in the life insurance pohcy
(The Life Assurqnce Company vs. Ebrado, G.R. L-44059 October 28, 1977).
3. Donations between persons found guilty of the same criminal offense, in
consideration thereof
4. Donations to a public officer or his wife, descendants or ascendants by
reason of his office
5. Donations to incapacitated persons
6. Donations of future property

Donation with defects at execution


1. Donation by a person who has no legal title to the property
2. Oral or written donation of real property or intangible personal
property
3. Donation refused by the donee

Foreign donations of non-resident alien donors


Donations of property situated in a foreign country by non-resident alien
(NRA) donors are not subject to donor's tax.

Donation of property exempt under reciprocity


Th~ donati_ons of intangib_le personal property in the Philippines by a non·
resident ahen are exempt if the reciprocity exemption applies.

The reciprocity rule


No ta~ shall be imposed with respect to intangible personal property
donations of NRA donors if:
a. The donor at the time of th d · fa
f . . e onat1on was a citizen and resident 0
.ore1gn country which at the time of his death or donation did not
impose a transfer tax of any character i . . ersonal
property of citizens of th PhT . n respect of mtang1ble P
e I ippmes not residing therein.

61?
T cnapte
r !6 - Introduction to Donor's Tax

'fhe laws of the f~reign count~ of which the donor was a citizen and
P· resident at th e time of donation allows a similar exemption from
1)1:ls ansfer tax of every character in respect of intangible personal
:il:ll'\: ;roperty of citizens of the Philippine not residing therein .

. ution of Gift as Specified by the Donor


piJillD . . .
donor may specify that a portion of the donation will be given to
1
'[he her person other than the donee. Diminution is not exempt from
<lo.
tic
anot r's tax. It IS
· mere lYa de d uc t 10n
· against the original donation b ut 1·t 1.s 1·n
~~:I~ another form of donation from the same donor which may be subject
. the ~o donor's tax.
e as
void ntustration 1 ..
I. Arma ndo donated PS00,000 cash to his older sister subject to the co nd1t1on that
e, in she will give Pl 00,000 to her nep hew.

Th re are two donations in this case:


e Sister N phew
Gross gift . P 600,000 _J~ P 200,0 00
Less: Diminuti on of gift
Net gift

Armando's donor's tax due hall b mput 11 w :

Gross gift
Less: Exempt don ation
Excess
Multiply by:
ien Donor's tax due

TAXAB LE DONATION
ex mption rit ri ubj tt
Donations that do not qu llfy m ng th
tax.

Examples of taxab l don ti n ·:


rt)' 1. Direct don atio n of prop I f noth r per on
a. Tran f r of pr per t th n m u~led with deliv ry o th s me
b. Tran f r of per on I pr perty
2. Donation in tru t, if irr ~o b! unle ther r only two h _ir
3. Specific renun i tion of rnh nt nc ' f hi· hare in th con Jug I or
. . spou o
4. Renun ciatio n b th ur ivmg
community prop rty . onsider tion of any prop rty oth r
S, Tran fer inter-vivo f r tn uffi ient
than re l prop rty c pita! t
613
Chapter 16 - Introduction to Donor's Tax

CHAPTER 16: SELF-TEST EXERCISES

Discussion Questions
1. What is donor's tax? Briefly discuss.
2. Explain the difference of donation inter-vivos from donation mortis
· 1· t· fd ' caus
3. Enumerate and explain each of the ratwn~ iza 10 ~ ~ onor s taxation. a.
4. Enumerate and explain each of the essential reqmsites of donation.
5. Discuss the formal requisites of donation.
6. Enumerate and discuss the classification of donor's.
7. Discuss the rules on donation of common properties and donation to
several donees.
8. Enumerate the list of exempt donations.
9. Enumerate the list of prohibited or defective donati ons.
10. Illustrate the donor's tax format.

Exercise Drills: Identification of transfer or exchange


On the blank provided for, indicate w hether the transaction is a sale, transf
21.
inter-vivos or mortis causa. For complex transfers, indicate sale and -~r
appropriate transfer counterpart. Leave the space blank if the transactio/s
22 .
.h 1 IS 23,
ne1t er a sa e nor a transfer.

1. Revocabl e donation in trust 24.


-
2. Revocable do nation
3. Revocable donation pre-termina ted by th e death of 25.
t he donor
4. Conditional donatio n Trn
5. Revocable do nation pre-terminated by the dea th of 1.
th e donee
6. Conditional donation, the condition of which is w a ived 2.
by the donor during his lifetime 3.
7. Irrevocable donation in trust
8. Waiv~r by the donor of his right to revoke a revocabl 4.
donabon e
9. Sale of personal property with fair value o f
5.
cost of PlM for only Pl.ZM P2M a nd
6.
10. Fulfillment by the donee of th t
donations e erms of a cond itio n a l
11. Transfer of personal pro er . .
7
1
and cost of PlM for onl: p{ ~ ~h fair value of P2 M 8
death ·2 m contempl ation o f g
12. Cancellation of a debtor' . d ~

out oflove and affection s m ebtedness by a cred itor


13. Cancellation of a debt ,
· . ors
m consideration of service
.
indebted
d ness by a cred itor
-
ren ered bv th e fo rmer

614
~ er 16 - Introduction to Donor's T
chaPt ax
sale of real property classified as .
14· f irvalueof P2M and costof PlM f capital asset with
~ or only Pl 2M
,al of real property classified as O d. ·
15. ~ f r mary as t . h
fair value o P2 .M and cost of Pl M for only Pl 2M
se wit
~
1 . e of domestic stocks classified as capital . as
· t .h
fair value of P2M and cost
. of PlM f 1 Pl.2M
or only se wit
~ e surrender by the fiduciary heir u on h'
the property designated by a p pd Is death of
fideicommissary re ecessor to a
11f31ransfer
· from. the first heir to th e second heir . m . .
accordance
L.----::- with the desire of a ored ecessor .
19- Sale of a prope~ty costing PlM with fair value of 2M
for only PlM.
· l Title shall not transfer unt"lI d eath o f the
seller. Fair va ue at death is Pl.BM.
1
w,Sale of a property costing PlM with fair value of ZM
for only ~lM. Title shall not transfer until death of the
~ seller. Fair value at death is PB00,000.
21. Donation of property not owned
22. Rendering of services for a comoensation
23. Transferring of money in consideration of a promise
bv the recipient for repayment
24. Return by the trustee of the property held in trust to
the grantor
25. Transfer of propertv for an adequate consideration

I True or False 1
1. Donor's tax is a tax on the gratuitous disposition of property by a living
donor.
2. Donor's tax is a property tax.
3. Donees and benefactor of donees have the primary obligation to pay the
donor's ax.
4. The amount of donor's tax is paid depends on the value of the property
donated.
5. Donor's tax is applicable to donation mortis causa made inter-vivas.
6. Donor's tax is intended to minimize loopholes in both income tax and estate
tax.
7- Donor's tax is also imposed to counter evasion from business tax.
8. A donor must be at least 18 years of age to make a valid donation.
9. Donation may be implied when the transferor is not claiming the return of
his property.
lO. Donations void or valid are subject to tax.
ll. The done~'s acceptance\s required for the completion of the donation.
12. The delivery of the donor of the property to be donated perfects the
contract of donation. . .
13 Accept • . d en 1·n transfers for inadequate considerations.
· ance 1s require ev
615
C~3P
Chapter 16 - Introduction to Donor's Tax ..,;tio,
14. The donation of intangible personal property muS t be in writing. 3· ~- (

,
15. The donation of tangible personal property may be made orally if the "al \J·
('..,
exceeds PS,000. bl' . lie
16. The donation of real property must be in a pu ic mS t rument:
17. The donation of intangible personal property muS t be made m Writing if th d-
value does not exceed PS,000. e wni<
18. Donors are classified into residents or citizens a nd non-resident aliens
4, 1
19. Non-resident aliens are subject to donor's tax only on donations of ta~gib a- I

·1· · s
properties located in the Ph i ippme •
le b· I

20. The donation to a non-resident alien is exempt from donor's tax. c.


d-
True or False 2
w11i,
1. Donations to any person is exempt if the amount of donation does not 5,
exceed P250,000.
a.
2. Donations by non-resident alien donors may . be exempt from Philippin e
donor's tax for intangible personal properties. b.
3. Intangible personal properties of non-resident alien donors, regardless of
their location, are subject to the rule on reciprocity exemption. C,
4. Aliens, residents or non-residents, are subject to donor's tax on tangible d.
personal properties located in the Philippines.
5. Aliens, residents or non-residents, are subject to donor's tax on intangible
6. Whi1
personal properties located in the Philippines. a.
6. Resident or citizen donors are taxable on donation of any property
b.
regardless of the location of the property.
c.
7. Donations to relatives are exempt from donor's tax.
8. Donor's tax is a local tax intended to raise money for the government. d.
9. The renunciation by the surviving spouse of his share in the hereditary
estate is always taxable to donor's tax. 7. Whi1
10. The general renunciation of inheritance is generally exempt from donor's a.
tax. b.
C.
Multiple Choice - Theory: Part 1 d.
1. Which is an incorrect statement?
a. Donor's tax is imposed on donation inter-vivas 8. Whi1
b. Estate tax is imposed on donation mortis ca cons
I . . usa
c. ncome tax is imposed on onerous transactions a.
d. None of these
b.
C,
2. Donor's tax is not intended to
a. raise money for the government d.
b. recoup government loss on fut .
ure tax revenue 9,
c. contra l tax evasion of incom t
d 'd e ax.
. Wh
. provi e a system of business regulat·10n. a. 1

b.
t.
616 ct

-
~ l6- Introduction to Donor's Tax

I
cnaPter
Who is the one directly liable to the payment of donor's tax?
3. The donor
a. The d onee
b. .
The Person tn possession of th .
c. · e property upon d1Scovery of the
donation
d. Any of these

Which of these donations is subject to donor's tax?


4· a. Irrevocable donation
b. Transfer in contemplation of death
c. Transfer under special power of appointment
d. Transfer under general power of appointment

Which of the following donation is not subject to donor's tax?


S. a. Donation intended by the donor to see his children enjoy the property
while he still lives
b. Donation intended by the donor to reward services rendered by the
donee
c. Donation intended by the donor to settle family disputes
d. Donation intended to take effect upon the death of the decedent

6. Which is subject to donor's tax?


a. Merger of the usufruct in the owner of the naked title
b. Delivery of the property by the trustee to the beneficiary
c. Support payment to children
d. Cancellation of indebtedness of a debtor out of gratuity

7. Which is not required in a contract of donation?


a. Legal capacity of the do nee
b. Acceptance by the donee
c. Delivery
d. Intention to donate

. h 1s
8. Wh1c • not reqmre
· dm· taxation of transfers for less than adequate and full
consideration?
a. Intention of the donor
b. Acceptance of the do nee
c. Delivery
d. A and B
. th d ation of real property?
9. What is the required form m e on
a. Written agreement
b. Public instrument
c. Oral agreement
d. A or B

617
Chapter 16- Introduction to Donor's Tax

10. The donation of shares of stocks must be in


a. writing.
b. writing if the value exceeds PS,000.
c. a public instrument.
d. any form.

11. Which is a correct statement?


a. Donation of tangible personal property not exceeding PS,000 can be
made orally.
b. Donation of intangible personal property exceeding PS,000 rnay b
made in writing · e
c. The donation of tangible personal property worth PS,000 or more rnust
be in writing.
d. The donation of real property must be in writing.

12. Which donor is subject to tax on global donation?


a. Mexican donor, residing in Africa
b. American donor, residing in America
c. Japanese donor, residing in the Philippines
d. Chinese donor, residing in Japan

13. Which is taxable only on Philippine donations?


a. Filipino residing in Jolo, Sulu, Philippinnes
b. A Filipino citizen, residing in Canada
c. An Australian citizen, residing in the Philippines
d. An Italian citizen, residing in the United Arab Emirates

14. Which of the following donated properties of a non-r sident ali n donor
may be exempt from donor's tax?
a. Investment in domestic stocks
b. Condominium unit
c. Cars
d. None of these

Multiple Choice - Theory: Part 2

1. Statement 1: Donat!ons to residents or citizens are subject to donor's tax.


Statement 2: Donat10ns to non-resident aliens are exempt from donor's tax.
Which is correct?
a. Statement 1 c. Both statements
b. Statement 2 d. Neither statement

2. Statement 1: Donations of non-resi'dent ali·ens to resident citizens are


exempt.
Statement 2: Donations of non-resid t 1. . re
exempt. en a Jens to non-resident allens

618
ter 16- Introduction to Donor's Tax
cnaP
Which statement is correct?
Statement 1 - c. Both statements
a. 2
b, Statement d. Neither statement

In donor's taxation, which is not considered in determining taxability of


3· donations?
a. Tax classification of the donor 1
b. Location of the property
c. Relationship of the do nee to the donor
d. A and B

4. The donation of a non-resident alien donor is exempt from donor's tax if the
property is
a. located abroad.
b. a real property located abroad.
c. an intangible property located in the Philippines.
d. located in the Philippines at the date of donation.

s. Which of the following donation to is not exempt from donor's tax?


a. Donation to the International Rice Research Institute
b. Donation to a government-owned and controlled corporation
c. Donation to the Integrated Bar of the Philippines
d. Donation to the Philippine Inventors' Commission

6. Which of the following donation is not exempt?


a. Philippine Institute of Certified Public Accountant
b. Intramuros Administration
c. Development Academy of the Philippines
d. Ramon Magsaysay Award Foundation

7. Which is not excluded in net gift?


a. Donations to politicians or political parties
b. Donations between spouses
c. Donations with reserved powers
d. Foreign donations of resident aliens

8. The donation by a corporate donor is


a. Always subject to donor's tax regardless of amount.
b. Presumed made to a stranger hence always taxable.
c. Presumed always made to a relative; hence exempt.
d. Subject to the same tax rules on donation of individuals.

9. Statement 1: Any person, natural or juridical, resident or non-resident who


transfers or causes to transfer property by gift shall file an estate tax return.

619
Chapter 16- Introduction to Donor's Tax

Statement 2: A donation is taxable, whe th er made in trust or Oth


whether the gift is direct or indirect, and whether the property • er\vise
is teal ,
personal. Ot

Which is correct?
a. Statement 1 c. Both statements
b. Statement 2 d. Neither statement

10. Which of these is taxable in the Philippines?


a. Donations made abroad
b. Donations made in the Philippines
c. Donations of property situated in the Philippines
d. Allofthese

11. Donation of foreign properties is exempt from tax if the donor is a


a. resident citizen. c. non-resident citizen.
b. resident alien. d. non-resident alien.

12. Which is subject to a reciprocity exemption on donor's tax?


a. Donation of intangible personal property situated in the Philippines by
a non-resident citizen.
b. Donation of intangible personal property situated in the Philippines by 18,
a resident alien.
c. Donation of intangible personal property situated in the Philippines by
a non-resident alien.
d. All of these

13. When the reciprocity rule applies, which of these is subject to donor's tax to
a non-resident alien donor? Ml
1.
frn12~~ LQcation
a. Cash Philippines
b. Land Japan
C. Car Philippines
d. Franchise Japan

14. Which is incorrect? 2,


a. Do~ations of non-resident aliens involving properties located abroad to
residents of the Philippines are exempt from d , t
b R · · f . h onor s ax.
· enu~ciatwn
donors tax.
° m eritance by an heir to another is not subject to

c. Ghifts ahrelre~koned on an annual basis. The donor's tax is determined for


t e w. o e gift throughout the year. 3,
d. Donation by a non-resident ·t· t
from donor,s tax. Cl izen to a non-resident alien is exemP

620
ter 16- Introduction to Donor's Tax
cnaP
hich of the following donations may be e
15 W . f ·ct xempt.1
· a. °
Do~datiotn . a resi ent alien of its properties located abroad to a non-
rest en a11en.
b.
Donation of a non-resident ali
.. . . ° f· .
en mtang1ble properties located in the
Ph1hppmes to a non-resident alien.
c. Don~ti~n of a no~-resident alien of real properties located in the
Phihppmes to a resident alien.
d. D~nation of intangible properties located in the Philippines by resident
ahens.

l6. Statem~nt 1: Corporate donors are not entitled to the P250,000 annual gift
exempt10n.
Statement 2: E~cumbrance on the property donated, if assumed by the
donor, is deductible for donor's tax purposes.
a. True, True b. True, False c. False, True d. False, False

17, Statement 1: A donation between husband and wife during the marriage is
generally void.
Statement 2: A donation can be made to conceived or unborn children.
a. True, True b. True, False c. False, True d. False, False
(RCPA]
18. Which is incorrect?
a. Donor's tax shall not apply until and unless there is a completed gift.
b. Donation is perfected from the moment the donee accepts delivery.
c. Donation is completed at the moment the donor makes delivery.
d. Donor's tax is a tax on the transfer of property at death.

Multiple-Choice - Problems: Part 1


1. Mr. Merta sold his stock investments with a tax basis of Pl,000 ,000 and fair
value of P2,000,000 for only Pl,200,000. Compute the amount of gratuity
subject to donor's tax.
a. P 0 c. P 800,000
b. P 200,000 d. Pl,000,000

2. Mr. Berto sold his family home for P2,000,000. His f~mily horn\ ~as
acquired 10 years ago for Pl,000,000. At the date of sale, his pr,operty a a
fair value of P2,500,000. Compute the gratuity subject to donors tax.
a. po c. P 1,000,000
b. p 500,000 d. Pl,500,000
donated a car worth PB00,000 to Mr. Palaboy
3. On June 4, 2020, Mr. Momoy t duate college not later
subject to the condition that Mr. Palaboydmfus g~s course The property
th O b 020 M p laboy graduate rom .
an cto er 2 . r. a urchased by Mr. Momoy at a cost
was now worth P700,000. The same was P
of Pl,000,000. .
621
Chapter 16 - Introduction to Donor's Tax ct,.1r ti.
Compute the gratuity subject to donor's tax. rJI
a. po c. P 800,000 9• f(I

b. P 700,000 d. P 1,000,000 i,1

51
4. In 2015, Don Bosco transferred a lot to Basco revocable within five y 51
The property had a fair value of P2,000,000 at that time. Don Bose O ct· ears.
· d h.IS ng 1d n
· h t to the proper
revoke the property. In 2018, Don Bosc_o waive ~t \I
favor of Basco. The property had a fair value of P2,500,000 in 2018 ty tn y
P3,000,000 in 2020. and 9
t
Compute the gratuity subject to donor's tax.
a. p 0 c. P 2,500,000
b. P 2,000,000 d. P 3,000,000

5. Intending to motivate his brother Jay, Alex transferred a commercial


valued at P2,000,000 for only Pl,000,000 subject to the condition that
1
l;t
must become a lawyer in 5 years. Jay passed the Philippine Bi
Examinations on the fourth year. The property was worth P2,S00,000 at
that time.

Compute the amount of gratuity subject to donor's tax.


a. P 500,000 c. P1,S00,00 0
b. P 1,000,000 d. P 2,500,000

6. Mr. John Hay, a resident alien, sold a r sidential lot with a fair value of
P6,000,000 for only P3,000,000. H acquir d the prop rty 20 year ago for
only PS00,000. Compute the amount ubject to donor's tax.
a. PO c. P3,000,000
b. P 2,500,000 d. P6,000,000

7. In 2020, ABC Holdings sold a building to DEF C mp n , wholly-owned 2


subsidiary of ABC Holdings, for P2,000,00 0. Th building had book value
of Pl,500,000 but with a fair market value of PS,000,000 t the d te of sale.

What is the amount subject to donor's tax?


a. p 500,000 C. P 3,000,000
b. P 2,000,000 d. P 3,500,000

8. Which is correct with respect to the above transaction ?


a. The gratu~ty poti~n o_f the sale is subject to progre iv donor's taX-
b. The gratmty port10n 1s subject to final income tax.
c. The gra~uity portion is a gift made betwe n stran ers subject to
progressive donor's tax. g
d. The gratuity portion is a gift subject to a proportion donor's tax.
I

622
ter 16- Introduction to Donor's Tax
chaP
Anderson had the followin · "1 .
r.,1rs. d d h b · g m ormation
9· from her ecease us a nd where she and her three regarding her inheritance
children were the only
heirs:
share from comm_u n~l properties P 1,200,000
Share in the net distributable estate 300,000
If Mrs. An~erson renounces both shares categorically in favor of the
youngest child, what amount is subject to donor's tax?
a. p O C. p 1,200,000
b. P 300,000 d. P 1,500,000

1o. In the ~mmediately prece~ing problem, if Mrs. Anderson renounces both


shares m favor of all the heirs, what is the amount subject to donor's tax?
a. p O C. p 1,200,000
Dt b. P 300,000 d. P 1,500,000
1y
It Multiple-Choice - Problems: Part 2
lt 1. On May 2, 2020, Uno donated the following properties:

Donee Prop erty and location Value


Dos Car, in Somalia P 600,000
Tres Business interest, in the USA 400,000
Quattro Cash, in the Philippines 300,000
Zinco Land, in the Philippines 90 0,000
Total P 2,200.000
Compute the taxable donation if Uno is a non-resident citizen.
a. p O C. p 1,200,000
b. P 900,000 d. P 2,200,000

2. Compute the taxable donation if Uno is a non-resident alien.


a. p O C. p 1,200,000

b. P 900,000 d. P 2,200,000

3. Compute the taxable donation if Uno is a non-resident alien and assuming


the reciprocity condition applies.
a. p O c. P 1,200,000
b. P 900,000 d. P 1,500,000

4. Adonor transferr ed the fo llowing properties:

Eroperty Donee Location Value


Car Resident citizen Philippines P 300,000
Cash No n-resident citizen Philippines 200,000
Stocks Non-r esident alien Abroad 150,000
Land Resident alien Abroad 250,000
Total P 900,000

623
. •. v0r1•,:.

,r

Chapter 16- Introduction to Donor's rax ·· · . · ·· · · , ,;:.,~


. . donor's ta:x'if the ddnor is,g!resident al· ~ ·
Computl:! the amount sobJect to 500 000 • , ' , . , , . •·11) i ie11.
4 . ' ( I C p , ", ' ,
1
• . pO ' d. p 900 000 i'-"l,tt f
b. P 400,000 · '
. d or's tax if the donor is' a hdn~re .
S. Compute the amourlt subJect to on ·, Sldent
.j
alien. p 500 000
~ : ~00,000 ~·. p 900,000
. • the USA made the following donations·
6. A Mexican donor who 11ves m ·
P 5,000,000
Deposit in Philippine banks t· 2,000,000
Shares of stocks iri a non-res1•ct ent,1 0 reign corpora 10n
I

3,000,000
Business interest in the Philippines
Total
p10.ooo.ooo
Compute the donation subject to tax.
a. p 0 c. PB,000,000
b. P 3,000,000 d. Pl0,000,000 1
7. In the.immediately preceding problem, compute the donation assuming the
reciprocity rule applies.
a. P 0 c. PB,000,000
b. P 3,000,000 d. Pl0,000,000

8. Anjo, a resident citizen, donated PZ00,000 cash to his second cousin abroad.
This was his only donation in 2020. Which is correct?
a. The donation is subject to progressive donor's tax.
b. The donation is exempt because it did not exceed P250,000.
c. The donation is subject to a 30% tax because it is a donation made to a
stranger.
d. The donation made to a non-resident is subject to a 30% tax.

9. Mr. Swastika, a Russian citizen residing in Ukraine, made the following


donations in Philippine currency equivalent:
• A commercial land in Russia, to a Filipino citizen - P3,000,000
• A sports car to his Filipina girlfriend - PS,000,000
• Cash to his Russian maid - PZ,000,000

Determine the total amount subject to Philippine donor's tax.


a. p Q C. p 8,QQQ,QQQ
b. P 2,000,000 d. p 10,000,000

10. Mr. Crown, a Canadian cit1·zen wh · . the


.. . o 1s married and residing in
Ph1hppmes, made the following donations d .

cas h, to h'1s brother-in-law_ Pl,000,00Ourmg t h e year··
• Shares of stocks in Canada to hi b h .
s rot er m Canad a - P2, 000,000
624
r iG- Introduction to Donor's Tax
cnapte
canada does not impose donor's tax to aliens in their territory.

cornpute the amount subject to Philippine donor's tax.


pO C. p 2,000,000
~: p 1,000,000 d. P 3,000,000

_A donor made the following transfers during the year:


11
Donation in revocable trust P 500,000
Donation in irrevocable trust 1,200,000
Direct donation to wife 800,000
Direct donation to sister 500,000
Total donation P 3,000.000

What is the amount of donation subject to Philippine donor's tax?


a. P 500,000 c. P2,SOO,OOO
b. Pl,700,000 d. P3,000,000

12. Ms. Dina Nalo believes in the chances of Mrs. May Laban for the
congressional seat in their district. She donated P400,000 in cash for the
latter's campaign expenses. Ms. Dina Nalo reported the donation to the
COMELEC.

Which is correct?
a. Ms. Dina Nalo shall pay P120,000 donor's tax.
b. Ms. Dina Nalo is exempt from donor's tax on the first Pl00,000 of the
net gift.
c. The donation is exempt from donor's tax.
d. None of these.

625
Chapter 17 - Donor's Tax t
I
CHAPTER 17
~D~O~N~O~R~'S~T~AX~==============~
Chapter Overview and Objectives
----------------------------------------------------------- ----------
After this chapter readers are expected to:
1. Understand a~d master the concept of net gift
2. Appreciate the valuation rules for gifts
J,
3. Be able to compute donor's tax

THE DONOR'S TAX


The donor's tax is imposed on annual net gifts reckoned over a calendar ye
basis. For every taxable donation, the donor shall determine and repo~
his/her net gift. The tax is paid within 30 days after every donation.

Net gift refers to the net economic benefits from the transfer that accrues to
the donee.

Donor's Tax Rate:

~U~
p to----=--- 0 0~0=-=---=______ _-4-__E
P~2~50~,~ ~x mpt
Excess above P250,000 6%

Note to readers
The revenue regulations defin d net gifts n t on mi b n fits from the
transfer that. accrues to the do nee · H n ,comm n ul th tthe
P2S0,0?0 gift exemption under the TRAIN I w h t d
deduction because it is in no way a d'1mmut1o
. . n t . n
accrues to the donee. The P2 50,000 ift n mt
tax rate structure. g x mpti n i r th r p rt

Illustration 1
I~ 2019, Mr. Generoso donated ondominium unit ith
his fi rst cousin, Mr. Alfonso. v lu ,000,000 to

Am ount
Net gift
p 4,000,000
Less: Exempt donation
Net gift subject to tax
250,000
Multiply by: Tax rate
p 3,750,000
Donor's tax 60/n
£ 225 OO_Q
,..- ter 17- Donor's Tax
cnaP
1ostration 2 .
~ donor rnade the followmg donations in 2Ql g:

~ ~ - , - - ,~D~o~n~e~e-:----+-~N~et,tu,,u·LJ
_4 2019 Best friend p 100,000
30 1
tv1arch 7, 2019 Brother, as weddin ift 300,000
NoV- 10, 2019 Sister, as birthda ift S00,000
oec. 25, 2019 Mother as Christmas ift 400,000
feb-14, 2020 Girl friend 700,000

~ 4 . 2019 Donation
The donor's tax due sh a ll b e nil.
Amount
P 1 00,000
Net gift 250,00Q
Less: Exempt donation
(P 150,000)
Net gift subject to tax

March 7. 2019 Donation


The donor's tax due shall be P9,000, computed as follows:
Amount
P 400,000
Total net gift (1 00K + 300K)
Less: Exempt donation
250.000
P 150,000
Net gift subject to tax
Multiply by:
Donor's tax to date
Less: Tax due in prior don ti n
Donor's tax still due

November 1O, 2019 Donation


The donor's tax du hall b P 0,000, omput d follow :

Total net gift (l00K + 300K SOOK)


Less: Exempt don tion
Net gift subj e t to t x
Multiply by:
Donor's tax to d t
Less: Tax due in prior do n tio n
Donor' tax s till due

Qgcember 25. 2019 Donation


The donor' tax du shall be P24,000, c mputed s foll w :

627
• 71',
~ ....... --,f,.

· Chapter 17 - Donor's Tax


Amount ,
1 P 1,300,000 ,
Total net gift (1 00K + 300K + S00K+400K) ro
250,00Q
Less: Exempt donation P 1,050,000
Net gift subject to taX ··
6%
Multiply by: •1 p 63,000
Donor's tax to date
Le s: Tax due in prior donation
39,000
p 24,0lli)_
Donor's tax still due

February 14, 2020Donatton


The donor's tax due shall be P27,000, computed as follows:
·Amount
P 700,000
Net gift
250,000 1111
Less: Exempt donation
Net gift subject to tax P 450,000 ro
Multiply by: 6% fiV
p 27,000 se<
Donor's tax due to date
Less: Tax due in prior donation 0 pr1
Donor's tax still due p 27,000
Th
Note that the gifts made in 2019 shall not be included since the donor's tax is a me
tax on annual donation. The accumulation of donation stops at the end of every
calendar year. . 1ll1
On
VALUATION OF NET GIFT Co
The following are considerations in measuring net gift: fol
1. Valuation rules 0
2. Timing of valuation H
3. Donation of common properties L
4. Encumbrances on the property donated V
L
Valuation of property donated
For purposes of donor's taxation, the valuation of properties donated follow Th
the same rules as those of estate taxation, namely: Pi
101
A. Real property- the higher of zonal or fair value per assessor's office (Sec.
102, NIRCJ
B. Personal properties - Ill
1. Shares of stock
Mi
Co
a. If listed in the PSE - the average high and low price of the stocks
at the date of donation
b. If not listed:
i. Preferred stocks - par value •I
·· C
11 · fjnancia
1
• ommon stocks - the book value appearing in
statements published nearest to the date of donation

628
_ Donor's Tax
,,ter 17
cnar
Other properties -
2, a. Newly purchased - purchase price
b. Old items - second hand value
c. Mon etary claims - the amount fl d 10
.
xe the contract
tion t: Fair value of real propertie
nustra
r donated a parceI o f Iand m . s
favor of his Th
W ~O ooo, an assessed value of P2,000 000 as~n. . e land had a zonal value of
p3,0 00 000. , , n an independent appraisal value
of p4,0 I

mount to include in net gift shall be p3 ooo 000 d


rhe a t be used in accordance with the NIRC ' . 1n ependent appraisal value
shall no .
ntustration Z: Fair value of old items
onY donated a Toyota Fortuner to his brother Andrew Th . d
T t Pl 800 000 A h • e car was acquire
five years ago a ' ' · t t e date of donation, the car was saleable as
second hand at Pl,000,000 but was subject to a P300,000 mortgage which Ton
promised to pay before transfer of title. Y

The donation shall be included in net gift at Pl,000,000. Note that the P300 000
mortgage is not assumed by the donee. '

Illustrative 3: Fair value of listed shares


on June 4, 2019, Mr. Angelo donated 1,000 shares of the stocks of Albatross
Corporation, a listed corporation. The shares of Albatross Corporation had the
following trading prices on June 4, 2019:

Opening price P 800.00


Highest intra-day trading price P 870.00
Lowest intra-day trading price 790.00
Weighted average traded price 820.00
Last trading price 850.00

The donation shall be included in net gift at P830,000, computed as [(P870 +


P790)/2 x 1,000}. The regulation requires the use of simple average of the high and
low not the weighted average price.

Illustrative 4: Fair value of non-listed shares


Miss Beauty donated her 20,000 common stocks of Suyoc Mines, a closely-held
corporation. The latest balance sheet of Suyoc Mines disclosed the following:

Tota\ assets P20,000,000


Less: Total liabilities 5,000,000
st0ckholder's equity PlS,000,00Q
Suyoc issued 40 000 preference shares at par value of P100 per share and
1oo I
' O,OOO ordinary shares.
629
Chapter 17 - Donor's Tax

At the balance sheet date, the entity disclosed th e following assets wi th fair values
different from their recorded book values:
Tax Independent
declaration Zonal value appraisal
Book value
fair value
p 2,500,000 P 5,000,000 p 6,000,000
Land P 2,000,000
1,500,000 p 2,000,000 2,500,000 2,400,0oo
Building
Investments 500,000 With a quoted market price of P800,000 ---

Under the new rule the book value per share of the non-liS t ed shares shall simply
-
be computed with~ut regard to the fair value of the assets held by the investee D
corporation as follows: . 01
fl\
Stockholder's equity P15,000,000
Less: Par value of preferred stocks (P400 x 10,000) 4,000,000
H1
Residual equity to common shares P11,000,000
1,000,000 ~
Divided by: Number of outstanding common shares
P 11.00 ia:
Book value per share
va
The donation of Miss Beauty shall be reported in net gift at ?220,000, computed as pr
Pll.00 x 20,000 shares. Ph

Timing of valuation of gift Hit


Donation is valued at the point of completion or perfection of the donation.
Mr
his
Donation is perfected upon acceptance by the donee. In conditional
donations, the donation is completed and perfected upon satisfaction by the Mr.
do nee of the terms of donation or upon waiver by the donor of the conditions. tax
Illustration 1: Revocable donation
Mr. A made a revocable transfer of a residential property in favor of his brother
B. The value of the property was P3,000,000. B's family moved to the property
when it was worth P3,500,000. Mr. A subsequently waived his ownership and
transferred title in favor of B when the property was worth P4,000,000.
The donation shall be valued at P4,000,000.

Illustration 2: Conditional donation


On November 13, 2019, Mr. Cebu made a conditional donation to his brother,
Talisay, in~olving an in~estment portfolio with an aggregate value of P3,000,000.
The donation was contmgent upon the latter's passing of the CPA Board Exam.
Mr. Talisay passed the CPA Board Exam on July 12, 2020 when the value of the
portfolio was P2,400,000. Three days later, Mr. Cebu died The investment
portfolio was worth P2,500,000. ·

630
_ Donor's Tax
17
cnaPter .
. n shall not be subject to estate tax, but to donor's tax since the
nat10 h . ,r .
fhe d _ was living at t e time 01 perfectwn of the donation. Mr. Cebu no longer
0

transfe1or ,-operty at the time of his death.


0,vned the p
. tshall be valued at P2,400,000, the value of the portfolio upon completion of
fhC Bifi ct of donation.
the contra
. n of common properties
ponatI~ and wife are considered as separate and distinct taxpayers for
Husbanes of the donor's tax. .
purpos
ation of conjugal or community property by the spouses is deemed ½
vo~e by the husband and½ made by the wife. The husband and the wife shall
~:~ donor's tax returns for the donation.

However, if what was donated is a conjugal or community property and only


the husband signed the deed of donation, there is only one donor for donor's
tax purposes, without prejudice to the right of the wife to question the
validity of the donation without her consent pursuant to the pertinent
provisions of the Civil Code of the Philippines and the Family Code of the
Philippines.

Illustration
Mr. and Mrs. Japson donated a brand new car worth P2,000,000 to their son on
his graduation.

Mr. and Mrs. Japson shall report half of the donation in their respective donor's
tax return asfollows:
··---------------------------------------------------------------------------------,
Mr. lapson Mrs. [apson
Net gift P 1,000,000 P 1,000,000
Less: Exempt gift 250,000 250,000
Net gift subject to tax P 750,000 P 750,000
Multiply by: 6% 6%
Donor's tax due p 45.0ilQ "=p==4=5-.0-Q==Q
·----------------------------------------------------------------------------------
This rule applies to donation of co-owned properties

~:~um~rances on the property donated assumed by the donee


ditn~ati?ns on the donated property which are assumed by the donee are
.inutions to the gratuity accruing to the donee. These are onerous
:signrnents of debt and are not gratuity; hence, these are deductible from
eValue of the donation.

631
Chapter 17 - Donor's Tax

Illustration 1 • son, sayno. Th e lan ct which


. ltural land to his •
Don Pablo donated an agncu l e of PZOM. The land also had PSQ 'Was
encui:ribered by an PBM mortgage had va u e mort a e w hile Don Pab lo 0,0oo
unpaid real property tax. Sayno assumed th g g as urned
the real property tax.

The net gift of Don Pablo shall be computed as:


P20,000,000
Gross gift
8,000,000
Less: Obligations assumed by donee
Net gift
Pl 2,000.000

Note: An obligation assumed by th e donor is not deductibt it 1, ill n t re u e the


economic benefits accruing to the donee.

Illustration 2
Don Pedro donated 25 buffalos with an aggr g t f ir ht t hi
brother, Juan. The donation was subject to th n it i nth Ju
the donor's tax on th e transfer.

The net gift shall be PS00,000. The donor n ,n , i. ot


present obligation on the prop rt c the

DONOR'S TAX RETURN


Any person m akin g d n ti n , h, h •r 1 r n hr
donation is spec ifi ca lly x mpl un I lh •
required to accomplis h und •r ath n r' t
dona tion.

Note that th e law r q uir pi . In pr


tripli ca te opie . Two pi . t k n b th
taxpayer's copy.

Filing Date
BIR Fo rm 18 00 is fil d \A, ' thin O d y •
r e turn i r eq uired for d n tion m, d ~ t
reflecting th r in any p r iou n t 1tt m
r.
Only one r tu rn is r quir d f r d n ti n
m a de to s veral done s. lf the don ti n 1
each pou s hall fil p r t r turn
sha r in th onjugal or communit pr p ,rr:.

6 2
~ 1-
Donor's Tax
teP
ci,aP donor's tax return
ofthC h
tettt I1all set fort :
011
c -eturn .5f made during the calendar year which is to be included in
r11e I I gt t
file 1 . 1g net gifts;
J. con1Pudtll tions claimed and allowable;
de uc d d .
fhe -evious net gifts ma e urmg the same calendar year;
i. AI1Y pi e of the do nee;
>- I1e nan1 · as th e Comm1ss10ner
· ·
r f ther informat10n may require.
4. such ur
5,
file the return?
\vltere.: cases where the Commissioner otherwise permits, the return shall
~xcept I d the tax paid to any of the following:
be fi[ed anthorized agent bank
a. An aunue district. o ff'1cer
Reve
b. enue collection officer
c. 6~~y authorized treasurer of the city or municipality where the donor is
d. domiciled
office of the Commissioner, if the taxpayer has no legal residence in the
e. .
Philippines

DONOR'S TAX COMPUTATION AND REPORTING


The computation of the donor's tax is on a cumulative basis over a period of
one calendar year. The net gift of every taxable donation during the year is
determined and the additional tax is computed, reported, and paid to the
government within 30 days from the day the gift was made or completed.

INTEGRATED ILLUSTRATIONS

Integrated Illustration 1
Mr. Caticlan, a resident citizen, made the following donations during the year:
February 2 - donation to his brother who is getting married: P300,000
March 18 - P 200,000 to a public school for public use
April 15 - donation to daughter on account of marriage: P400,000
August 5 - donation to an accredited non-profit research institution:
P400,000
October 30 - donation of land valued at PS00,000 to Mr. Caticlan's father
subject to a condition that the latter shall assume the PlS0,000 unpaid real
property tax on the property
The net 'ft
gi for each donation shall be:

· Net Gift '?

P 300,000 P 300,000
Exem t 0

633
Chapter 17 - Donor's Tax

Aoril 15 P 400,000 400,00 0-


August 5 Exempt 0
October 30 P 500,000 - P150,000 350,000

The cumulative gift tax and the tax on each donation shall be:

Cumulative Less:
Donor's
Gift subject Donor's previous
tax Still
Date Net Gift to tax tax due tax due
Feb.2 p 3,000 p 0
~
P300,000 P 50,000 p ~
April 15 27,000 3,000
400,000 450,000
~
Ober 30 350,000 800,000 48,000 27,000
2Tooo-
----.:_
Note: The donor's tax payables are due 30 days from the date of donat10n.

Integrated Illustration 2
Mr. Mamoud made the following donations in 2020:

Date Prooertv Value -


Feb.24 Car in the Argentina to donee Philippines P20~
May2 Land in the Philippines to a donee in the abroad 600,000
Oct. 10 Shares of stocks in the Philippines a do nee abroad 200,000
Nov. 11 Laptop in the Germany to a donee in the Germany 250,000
Foreign ti
The tax er
Assuming Mr. Mamoud is a resident or a citizen donor
The following would be the donor's tax after each of the following taxable gifts: foregoing

The final f
Cumulative Less: Donor's
Gift subject Donor's previous tax still
allowable
Date Net Gift to tax tax due tax due due
p p
To
Feb. 24 P 200,000 0 0 p 0 p 0
May2 600,000 550,000 33,000 0 33,000
Oct. 30 200,000 750,000 45,000 33,000 12,000
Nov.11 250,000 1,000,000 60,000 45,000 15,000 lllustratic
AFilipino
Assuming Mr. Mamoud is a non-resident alien donor
The following would be the donor's tax after each of the following taxable gifts:
t
Cumulative Less: Donor's
-
Gift subject tax still
Date
Mav2
Net Gift
P 600,000
to tax
P 350,000
Donor's
tax due
P 21,000
o.revious

p
tax due
0
~--
p 21,QQQ_
Oct. 30 200,000 550,000 33,000 21,000 12,000

634
ter 17- Donor's Tax
chaP

1. :n
Note: esident or ali en donor is taxable on global if
Philippine gi~s.
.
g ts while a non-resident donor is taxable only
The location of the property is the controllin fa .. , . .
2, I' 115 not the location of the do nee or th d g ctm in donors taxation of non-resident
a ,e tion of inta ngible property . e onor at the time donation is made.
3, ~:~:en by non -r~sident ~lie_n don~~-presumed taxable unless a reciprocity exemption is

4. The tax payable 1s due w1th10 30 days from the date o f each donat10n.
.

FOREIGN TAX C_R_EDIT .


Resid~nts a nd cit_iz~n~ w~ th forei_gn donation may be subjected to foreign
donors tax. To mmimize mternat10nal double taxation our tax law allows a
credit for taxes paid in a foreign country. '

Foreign tax credit is computed depending on whether a single foreign


country or multiple foreign countries are involved.

Foreign Tax Credit Limit: One foreign country


The foreign tax credit shall be the lower of the actual foreign donor's tax paid and
;----- the following limit:
00 Foreign country net gifts
-----
00
World net gilts
x Philippine donor's tax due
----
00

-
----
00
Foreign tax Credit Limit: Multiple foreign countries
The tax credit limit for each country is individually determined first using the
s: foregoing computations.
~
The final foreign tax credit shall be the lower of the total of the donor's tax credit
's
r, allowable per country and the world donor's tax credit limit computed as:

I Total foreign net gifts

~,
~
0I
World net gitts

Illustration 1: One foreign country


x Philippine donor's tax due

~ A Filipino citizen made the following donations during the year:


Philippines Abroad Total
Total net gifts p 600,000 P 300,000 P 900,000
Foreign donor's tax paid 21,000 14,000

The donor's tax due on the donations shall be computed as:

0 p 900,000
Total global net gifts
0 Less: Exempt donation
250,000
p 650,000
Total net gifts subject to tax
Multiply by: 6°Lb
p 32,QQQ
Total donor's tax due
635
Chapter 17 - Donor's Tax

The donor's tax credit shall be computed as:


Actual foreign donor's tax paid p 14,000
Limit: (P300,000/P900,000 x P39,000) p 13,000
Foreign tax credit - LOWER p 13,000

The donor's tax payable shall be:


Total donor's tax due p 39,000
Less:
Foreign tax credit p 13,000
Philippine donor's tax paid 21,000 34,000
Donor's tax still due p 5,000

Illustration 2: Multiple foreign countries


Assume a resident alien made the following donations during the year:
Donation Philippines Taiwan Korea Total
Total net gifts P 500,000 P 300,000 P 200,000 P 1,000,000
Donor's tax paid 15,000 21,000 5,000 41,000

The tax due on the donations shall be computed as:

Total global net gifts p 1,000,000


Ma
Less: Exempt donation 250,000
p Ap
Total net gifts subject to tax 750,000
Ma
Multiply by: 6°Lb
Total donor's tax due p 15,Q QQ Au

The donor's tax credit shall be determined as follows: The

Per country tax credit: Sine


kno,
Taiwan:
P 21,000 pre/
Actual amount paid
Country limit: (P300K/1M x P45,000) 13.500 -,
lower amount i. P 13,500
Korea:
Actual amount paid p 5,000
Country limit: (P200K/1M x P45,000)
lower amount
Total of tax credit allowable per country
9,000
1 5,000
P 18,500
World tax credit limit:
[(300K+200K)/1M x P45,000 tax due] P 22 ,500
Foreign income tax credit (LOWER) P 18,500

636
r ,
17- Donor's Tax
chapter .
or's tax payable shall be:
r11edOO
donor's tax due P 45,000 ·
rota I
Le55F: reign tax credit p 18,500
0
philippine donors tax pa1"d
I
15,000 33.500
oono r
's tax payable P 11.500

pRACTICAL CASE A~PLICATION • •

ne Jaw merely provides for the allowance of tax credit for donors taxes paid but
1

~e regula_tions is devoid of any guidel_ine for this pu~pose. Thus, there is a need
to scrutimze the law and the regulat10ns for clues m order to effect the legal
intent.

The donor's tax in the tax code is a tax on annual donations but is paid on a
transactional basis. Since it is an annual tax, the donor's tax credit on foreign gifts
must be computed on the annual balance of the donor's tax; not on the
transactional amount of tax.

Aresident donor made the following donation in 2020:


Net gift Location of
Date Net gifts subiect to tax property Donor's tax paid
March 5 P 400,000 P 150,000 Philippines P 9,000
April 3 200,000 350,000 Philippines 12,000
May7 150,000 500,000 Abroad ?
August 4 250,000 750,000 Philippines ?

The donor paid Pl0,000 on the donations made abroad.

Since tax credit cannot be computed until all donations throughout the year are
known, the donor's tax paid in the foreign country should be deducted as a
weliminary tax credit. The final tax credit will be determined at year-end.

May 7, 2020 donation

Tax due on P450,000 global net gifts p 27,000


Less: Donor's tax paid on prior gifts
Tax paid, prior Philippines gifts P 21,000
Foreign donor's tax paid 10,000 31,000
Donor's tax still due (P 4,000)
Note: This negative figure is simply ignored. The donor-taxpayer cannot ask for a tax refund or
t~ credit until determination of the final tax credit at year-end. A resultant positive amount is
Paid to the BIR.

637
C
Chapter 17 - Donor's Tax
C
Au~ust 4, 2020 donation I

p 45,000
Tax due on P750,000 global donation
Less: Donor's credit
Tax paid prior gifts - Philippines P 21,000
Foreign donor's tax paid 10,000 31.000
Donor's tax still due
p 14,00Q

Adjustment return at year-end


1
Although neither mandated by the law nor the regulations, an adjustment retur
1
at year-end is inherently necessary to finalize the amount of the donor's tax erect~
and the final amount of tax still due or refundable.
2
3
Year 2020 concluded with Pl,000,000 total gifts and a P750,000 net gift subject
to tax.

The donor's tax for the 2020 donation is P45,000 since there is no more
succeeding donation from thereafter: 6.

The foreign donor's tax credit should be computed as:

Actual foreign income tax paid .._P_--=1=-0,=0-=0=0


Limit: (PlS0,000/Pl,000,000 x P45,000) .....
P_--=6=,7-=S-=0
Foreign tax credit - LOWER ~p===!c6~,7~5~0

The donor's tax still due shall be:

Total donor's tax due p 45,000


Less:
Foreign tax credit - final amount P 6,750
Philippine donor's tax paid 35,000 41.7 50
Donor's tax still due p 3,25 0
Note: This Philippine donor's tax paid during the year is computed as: T1
p
1.
March 5, 2020 donation 9,000
April 3, 2020 donation 12,000
August 4, 2020 donation 14,000 2.
Total p 35.000
3.
Tax credit is limited to residents or citizens
It ~u_st be note~ that the privilege to claim foreign tax credit is allowed only 4.
to c1t1zen or resident donors at the time of donation. S.
6.
Non-resident alien donors cannot claim foreign tax d't Th. • d to the
t: h h · c · d ere 1 . JS JS ue
1act t at t eir 1ore1gn onations are not taxable in the Philippines.

638

1111111
ter 17 - Donor's Tax
cnaP
c»A~P~T;;;...ER=17=:=S=E=LF=-=T=E=ST=EX=E=R=C=l=SE=S==,.,,,=,=====-======

·scussion Questions
DJ Enumerate examples of taxable donations.
1· What is net gift? Explain briefly.
; : Why is the P2SO,O?O gift exemption not a deduction in computing net gift?
Discuss the valuation rules on donated properties.
4
5.· Explain the treatment of donation of common properties.
rrue or False 1
1. The donor's tax rate to relatives is a progressive tax.
z. The donor's tax rate to strangers increases as the net gift increases.
3. Donation between juridical person is not qualified to PZS0,000 gift
exemption.
4. The gift of real property may be measured at independent appraisal value.
5. The gift of listed shares is the closing price of the shares at the date of
donation.
6. Corporate donors are subject to 6% flat donor's tax. Individual donors are
subject to a tabular donor's tax.
7. Donation to corporate donees is not qualified to PZS0,000 gift exemption.
8. The gift to non-residents is exempt from donor's tax.
9. The gifts made to residents are generally subject to donor's tax.
10. Donations to donees abroad are exempt from donor's tax.
11. The donation to a future heir is not subject to donor's tax.
12. Donation in trust which is irrevocably designated is subject to donor's tax.
13. The registration of a property in the name of another person is a taxable
donation.
14. The cancellation of the indebtedness of another for a consideration is a
donation.
15. Donation in revocable trust is excluded in net gift.
16. Husband and wife are considered separate taxpayers.

True or False 2
1. The donation of communal or conjugal properties is presumed made one-
half by the husband and one-half by the wife.
2. Under the TRAIN law, the donation of a donee has no difference with a
donation to several donees.
3. Transfers mortis, whether inter vivos or mortis causa, for an insufficient
consideration are subject to donor's tax.
4. Gifts of preferred shares are measured at the par value of the shares.
IY
S. Net gift must be valued at the point of death of the donor.
6· Net gift must be valued at the completion of a conditional donation.
7· The value of gift on revocable transfers is determined when the contingent
donee accepts the property.

639
r's Tax . ..
Chapter 17- Dono ent tP the ,(donat10li). of!a •,f onj,ugaJ
ses did net eons .• or for purposes of the donor'st 0t
. 8. If either .of the ::~there is only one ~~~uctible in c;:omput,i ng n~t gift. cl){,
• 'Communal prop b the donor are. the donor is a form df don .
9 Obligations assumed Y 's oblig· at10n by , • ., , , 1-,,u.. . ation .
· . f th donee .· · ·
1OTheassumpt1ono e . , . · d ' "f ' u '·
. subject to donor's tax. d~nor on th~ · property · ~ona_t ~ , I ' as~ . n:1ed by the
11. The obligations of th e . net gift. ,• . - • . .
donee is deducted in computi?g I 1·ng·propert1es are exei;npt / rom donor's
12. Donations, to the gove~n ment mvo· ,. ,v1 · ' .,
- ' i
• •
,

' '
f
t ax . t·tutions for admm1strat1ve purposes
· • d non -profit ms 1 • ., 1
13. Donations to accredite,
are exempt frorp don~r_s t_ax.

Multiple Choice -Theo~: Pa,rt ~·


1. A gift is valued at the date of
a. death of the donor.
b. nota~izatiop of the donation. h d f on
c. completion or acceptance oft e ona 1 ·
d. payment ofthe,donor's tax. .
9.
2. A donation ofreal property is valued at
a. zonal value of the property.
b. value per tax declaration.
c. independent appraisal value.
d. the higher of zonal value or value per tax declaration.

3. Which type of donation is exempt? 10. ,


a. Renunciation of the right to collect an indebtedness of a debtor
b. General renunciation of inheritance by an heir
c. Transfer for an insufficient consideration
d. Irrevocable donation in trust
11. ,
4. The donation of preferred stocks ar e valued at
a. Book value
b. Parvalue
c. Second-hand value ·
d. Independent appraisal value

5.
For purposes of donor's taxation th
(
determined as its book value using ;he e fair value of common stocks is
a. Asset appraisal method
b. In~ependent appraisal method (

c. AdJusted net asset method


d. Market price averaging method

640
7- Donor's Tax
cnapter 1
Which is correct regarding donation of common properties?
6, Both spouses shall file separate donor's tax returns
~: Either spouse shall file a donor's tax return.
The spouses shall file a consolidated donor's tax return.
~-- Any of these at the discretion of the spouses

le The donation of common property shall be considered


7· a. one-half made by the husband and one-half made by the wife.
's b. totally made by the spouse signing the deed of donation.
c. void unless with the consent of both spouses.
d. donation made by the head of the family.

8. Donations of properties to joint donees are treated as


a. separate donations to be reported under a separate donor's tax return.
b. donations to each do nee which will be classified according to the relation
of each do nee to the donor.
c. a donation to a partnership; hence, these are subject to 30% donor's tax.
d. a single donation which will be reported in a single donor's tax return.

9. Which of the following donations is taxable?


a. Donation to an accredited charitable institution
b. Donation to an association
c. Donation to an educational institution accredited by CHED
d. All of these

10. Which of the following obligations, even if assumed by the donee, is not
deductible in computing net gift?
a. Mortgage payable c. Donor's tax
b. Real property tax d. All of these

11. Which of these donations to a non-profit institution are exempt from donor's
tax?
a. Donations restricted by the donor for salaries of staff of the non-profit
institutions
b. Donations restricted by the donor for utilities expenses of the non-profit
institutions
c. Donations restricted by the donor to be used only for program expenses
of the non-profit institution to carry out its organizational mission
d. All of these

12 - Obligations, if assumed by the donor, are


a. deductible by residents or citizens.
b. deductible by non-resident aliens.
c. deductible by any donor.
d. never deductible.

641
l , , t' h' t : \\ ht rt' pt'<'l t'rty is tr.111st; rt't.·d lurlnr.· 11wrs n's lifetitne
0
th.m .tdt.'qll.ttt' .1nd full t' ,nsi kr.1til111 n 1m m•,· nr mnne ,·s worth, t~ tless
,-lllh)unt hy which till' .1111 ~ l f thl' pr, \wrty l'X 'l.'1:'ckd the alue en the
nsidcc'r.1ti n sh. 11, fot· the purp,)St' l,t' tht' ,h,1tor·s t.1x. be deemed a i of the
"' 1tn It nt · : Encumbr;mce on thl' i rnpt'l'l. dtlll,\l 'd, if,, - ·urned by th~
0
d'
' d 'd U(til I f r ctonur· .1:< pu q lJS '"· tlor,

\,\ hkh Is tru .


~t, t:em nt 1 . Both . utenwnts
tat m nt ct. -ither ·r.1t 'Ill ,nt

Multipl Choi - Th ory: Part


1. tot7 m nt 1: D n ti n brad ar t r m n r' x to n a1·1e
11
d n r.
tat m nt 2: D n ti n in th Philip( in :< rnpt t
n n-r ident
don
hi
tat m nt 1 th ·t t ment.
b. tat m nt 2 ith r t

2. Which is corr ct re rdin th 'mpti n drnut nt n n·· k n non.


profit in tituti n .
a. It must b mad t a t ht• 1 m ·ttrn t< n · Hll .
b. ltmu tb m,ti t b n, h, 11 n-11, tit 111,tttu I n . vh •ch r r dit d
or not.
C. The don r mus l ., · 1tl •n I ltil'll ,t h·lh ,
d. Th prop rt lon,H • I mu l ,tl • I 1n lhc.• ~ h1h h
don tion.

3. Don tion in th t,1 r 'turn I pr 1' I


Thi · i. tru t
onati ns t individu. L nl
b. Don, tion t juriti1 ·.111wr-,un nl
Both don._ilion · to ind1v1tlu~1t-. n Id n
d. Don ti n m,1d • tn th· Phll11 pin· nl

4. tat m nt l: Donor• I \l fl h l )~ h I. l l ~t
th time of gift.
tat ment : Th ll hn th
gin h II 0 rn th · m t

hi hi
. 8 th . H •f 1 ll
b. tatem nt d. l
FTPIILWW

T Chapter
1
7 _ Donor's Tax
.
·ch of these donations to the govern
5. Whl ations of tanks and ammunitio:tt~\~re su_b_jec~ to donor's tax?
0011
a.b- oonations
.
of computers to the O e Phihppme military
epartment of Education
oonat10ns to government-owned and 11 ·
c. None of these contro ed corporations
d.
counted from the date of donation, the donor's tax is 'd . .
6. a. 10 days. c. 30 days. pai w1thm
b. 15 days. d. 60 days.

Which is not a require~ disclosure in the donor's tax return?


7, a. current and past gifts during the year
b. Donations made last year
alien Name of the do nee
c.
d. Deductions claimed
,ident
Which is not included in net gift?
8.
a. Transfers to the government
b. Transfers to non-profit and non-stock charitable institutions
c. Portion of the fair value of the property adequately paid for
d. All of these
non-
9. Which of the following is included in net gift?
a. Transfers to the government
lited b. Transfers to non-profit and non-stock charitable institution
c. Portion of the donation representing dowry
d. Excess of the fair value of the property over the consideration paid by
:e of the donee

10. Which of these is not included in net gift?


tsis. a. Dowry gift to a step-child
b. Gift to a spouse
c. Gift to relatives
d. Gift to a stranger

ll. Under the law, the donor's tax return shall be filed in
a. duplicate. c. quadruplicate.
at b. triplicate. d. quintuplicate.

· Which is not entitled to claim tax credit for foreign donor's tax paid?
12
a. Resident alien c. Non-resident citizen
b. Resident citizen d. Non-resident alien

13 · The donor's tax may be paid to the following except


a. An authorized agent bank
b. Revenue district officer
l 643
I
~
Chapter 17- Donor's Tax

15. Where the donor has no residence in the Philippines, th e donor's tax return
shall be filed with
a. the office of the President.
b. the office of the Commissioner.
c. an authorized government depositary bank near the residence of the
donee.
d. the Revenue District Office having jurisdiction over the place where the
donee is located.

16. Which of these donations to the Philippine government is subject to donor's


tax if the donor is a resident?
a. Donation of properties located abroad
b. Donation of properties located in the Philippines
c. Both A and B
d. Neither A nor B

17. Wh~ch of these ~onations to the Philippine government is subject to donor's


tax If the donor IS a non-resident alien?
a. Donation of properties located abroad
b. Donation of properties located in th e Philippines
c. Both A and B
d. Neither A nor B

18.
Which hof th ~ following properties, w hen donated are not subJ' ect to donor's
tax tot e wife? '
a. Capital properties
b. Paraphernal properties
c. Common properties
d. Band C

Multiple Choice - Problems: Part 1


1. Mr. an d Mrs. Gaspar donated . 'th
the wishes of Don Silvano, Mar piece of.land to Mr. Lee in accordance w~e
property to Mr. Gaspar as th fi . Gaspar s father . Don Silvano donated
e IrS t donee with Mr. Lee as the second donee-

644
onor's Tax
17- D
ter
C~aP orrect?
whi ch iS c r Ga par shall fil e th e donor' tax return.
onlY~ .' Ga par a nd Mrs. Gas par shall file se parate donor's tax return .
a- th 1vil . • d ,
b· 8° ift i exempt from onor s tax.
c. The g shall fil e th e donor' s tax return.
Mf• Lee
d-
Eddi e made a sole donation of P 300,000 cash to an accredited
Ill Z?t~:'ie institution for its program expenses. Which is correct?
2
· c11an di e shall pay P3,000 donor's tax. .
a. Edh donation is exempt because the donee is an accredited chantable
b T e .
. ·nstitut10n.
~he donation ~s exe~pt because it d~es not excee? Pl 00,000.
c. The donation 1s subJect to the tax usmg the donor s tax table.
d.
5
was indebted to Juan for PS00,000. Out of compassion to his brother
3. Jame 5 Pedro assumed and paid James' obligation.
Jame,
Which is correct?
James shall report the PS00,000 as gross income.
~: James shall report the PS00,000 in his donor's tax.
c. Pedro shall report the PS00,000 as donation and pay the donor's tax.
d. Both James and Pedro shall file a donor's tax return.

4. In 2019, Mr. Mabait made a lone donation of P 200,000 cash to Mrs. Malait,
his half-sister. Which is correct?
a. The donation is subject to 6% tax.
b. The donation is exempt from donor's tax.
c. Mrs. Malait shall pay the donor's tax.
d. None of these.

5. In 2019, Mr. and Mrs. Mapagbigay made a single donation of P200,000 to


their son on account of his graduation from college. The donation was taken
from the salaries of Mr. Mapagbigay.
Which is correct?
a. The donation is exempt from donor's tax.
b. The donation shall be subject to income tax.
c. Both spouses will pay donor's tax.
d. Either of the spouses pays the donor's tax.
6
· Jay donated Pl,000,000 to the Autism Foundation, an accredited social
Welfare institution. Jay stipulated that 1/ 4 of the donation shall be given to
Cancer Survivor Society, another accredited non-profit institution.
~: The ent~re PlM donation is exempt fr?m donor's t~.
The entire PlM donation shall be sub1ect to donors tax.
c. P7 so,ooo donation shall be subject to donor's tax.
d. P2so,ooo donation shall be subject to donor's tax.
Chapter 17 - Donor's Tax

7. Ms. Kris gave her longtime best friend ah~d ABS-C~N Kapamilya co-host~
Abunda, a Hummer worth P14M. Whic is correc . ' r.
a. Th " donation is subject to PB4-0,000, tax.
b. The donation is exempt from donors tax. ,
c. The donation is subject to P825,000 donor_ s tax. b M A
d. The donation is a reportable item of gross mcome Y r. bunda.

8. Mr . Realty , a vAT taxpayer , transferred a commercial


h . lot
fd
to his son as a
wedding gift. The \ot was worth Pl,000,000 at t e time o onation.

Which is incorrect?
a. The transfer is subject to VAT.
b. The transfer is subject to capital gains tax. ·
c. The transfer is subject to donor's tax.
d. All of these

9. Mr. and Mrs. Aguinaldo donated a piece of land worth PS00,000 to their
daughter, Hanna, who is getting married. In the same year, Mr. and Mr
Aguinaldo made a prior donation of Pl00,000 to a family friend. s.

Which is correct?
a. Either Mr. or Mrs. Aguinaldo shall report a net gift of P350,000.
b. Either Mr. or Mrs. Aguinaldo shall report a net gift of Pl00,000.
c. Mr. and Mrs. Aguinaldo are exempt to pay donor's tax.
d. Both Mr. and Mrs. Aguinaldo shall report a net gift of PS0,000.

10. Miss Gelen wanted to help her brother who is financially challenged. She
donated an agricultural land worth Pl,000,000 and designated her brother
and sister-in-law as donees.

How much shall be reflected in net gifts subject to tax?


a. p 1,00,000 C. p 500,000
b. P 750,000 d. P 0

11. Mr. and Mrs. Tehan donated a commercial lot as dowry to their daughter.The
commercial lot is a conjugal property valuing P800,000 at the date of
donation. The spouses do not have other donation during the year.

Which of the following properly depicts the presentation of the donation in


the donor's tax return of the spouses?

Mr. Teban Mrs. Teban


a. P 400,000 P 400 ,000 1I

I
b. P 275,000 P 275,000
C. P 150,000 P 150,000
d. P 750,000 p 0

646
_ Donor's Tax
17
cnaPter
Nicod emo di sposed an agri cultural land to his granddaughters, Mara
12. oo; ctara for a consid ~ation of P200,000 . The property was worth
;: 000 ,ooo at the date of d1 spo al. Compute th e net gift subject to tax.
' p soo,ooo C. P 300,000
a. p 550,000 d. P 0
b,
Mr-and Mrs. Manol_o d?n.at ed a property which Mr. Manolo inherited during
1 · their 111 arri_age. Whi ch 1s mco_rrect?
a. There 1s only on e do nor m th e donation.
b. Only Mr. Manolo shall file a donor's tax return.
c Mrs. Manolo shall not file a donor's tax return.
d·. Both spouses shall file separate donor's tax returns.

. Mr. and Mrs. Miranda donated a car from the sole salaries of Mrs. Miranda.
14
Which is correct?
a. There is only one donor in the donation.
b. Only Mr. Miranda is required to file a donor's tax return.
c. Only Mrs. Miranda is required to file a donor's tax return.
d. Both spouses shall file separate donor's tax returns.

15. Mr. and Mrs. Jerry made a donation of P800,000 to their daughter, Jemma, on
July 1, 2018. On August 1, 2019, they made another donation of P700,000 to
Jemma. Compute the net gift of the spouses subject to tax as of August 1,
2019.
Mr. ferry Mrs. ferry
, She a. P 750,000 P 750,000
Jther b. P 500,000 P 500,000
C. P 100,000 P 100,000
d. P 550,000 p 0

Multiple Choice - Problems: Part 2


1. Mr. Alfonso donated a brewery with a value of P24M to Mr. Ginebra. Mr.
Ginebra assumed the mortgage on the property valuing P18M. Compute the
·.The net gift.
te of a. P 24M c. P6M
b. P 18M d. P 5.75M

on in 2. Magaling Company donated a commercial lot worth Pl,000,000 to Bolatsing


Company. The lot was encumbered by a P400,000 mortgage which was
assumed by Magaling. The property had Pl00,000 accrued property tax
which Bolatsing assumed.

The net gift shall be


a. P 1,000,000 C. p 600,000
b. P 900,000 d. P 500,000

647
Chapter 17 - Donor's Tax phTppine government. The lot had a
th 1
3. Anderson donated a Jot to e ~bered by a Pi,000,000 mortgage ~tial
value of P4 '000,000
. and wasnencu . 7
the donation. · hi
O
is the net gift sub1ect to tax 000 ooO
a PO c. p 3, ,
b.
p 1,000,000 d. p 4,000,000
. d d p600 000 to his son as dowry f
4. On July 1 ' 2020 ' Mr · Carnon onate ther dowry ' of P300,000 to his d or his
upcoming marriage. He also gave ano aughter
who got married last April 30, 2020.
What is the total reportable net gift as of July 1, 202 O?
a. Mr. Carrion shall report a P900,0 00 net gift.
b. Mr. Carrion shall report a P650,000 net gift.
c. Mr. Carion shall report a P400,000 net gift.
d. Both donations are exempt since they are dowry.

5. Henrie donated 20,000 common stocks of Quickie Birds Corporation. At th


date of donation, Quickie Birds Corporation h ad the folio in•: e
P 2 ,000 ,000
Shareholder's equity 100,0 00
Outstanding common stocks
ith fair
Per financial statements disclo sure , Qui cki h d
value exceeding their book v \u b t t \ of Pl,20 ,
i ft.
Compute the amount to b r p r t
a. P 200,000 .p
b. P 540,000 .p

l n h publi hed
6. Bruno had 120,000 in th
financial statement of Jon t th n ti n r v I the
following:

Total assets at fair va\u p


Total assets at historic l co t
Total liabilitie
Outstanding common t ck

Jonas Corporation i n t liquid\ tr d d hi\ippin to k E han eat


P2.8/share. The donation h \\ b th
a. P 120,000 v lu d t
b. p 288,000 C. p 336,000
d. P 408,000

7. Mr. and Mrs. Julio tr n ferr d th foll in 201


• P400,000 ca h to d ughter l o
• P600 ,000 \ nd to on E ' an ' gr du tion
• P30 0,000 car to Hor 'ceug n ho · n t
M • . • is s he ul d to
• r. Juho s b t n• nd, s ddin g

648
r 11aPter
C
l7-Donor'sTax

. the respective net gift of Mr. and Mrs. Julio subject to tax as of the
wt,at I5
first gift?
p O; p 0
a. p 200,000; P200,000
b. p 400,000; P 0
c. p O; P400,000
d.
What is the respective net gift of Mr. and Mrs. Julio subject to tax as of the last
8· iftin2019?
g p 525,000; P 525,000
a. p 500,000; PS00,000
b.
c. p 400,000; P400,000
d. p 1oso,ooo; Po
Shanon, a non-resident Japanese, donated domestic stocks valued at
9
· pS00,000 to his half-sister who is getting married abroad. Shanon designated
the donation to the new spouses. He also donated her used car in Japan worth
Pl,000,000 to the spouses. What is the net gift subject to tax?
a. p O C. p 1,250,000
b. P 250,000 d. P 1,500,000

10. Mr. and Mrs. Crockie are resident aliens. They donated one of their
investment in domestic bonds worth P2,000,000 to Rey, their common child,
who just got married. The country of the spouses where they are nationals
allows Filipinos therein exemption from gift taxes.

Compute the respective net gifts of Mr. and Mrs. Crockie subject to tax.
a. P 1,000,000; P 1,000,000
b. P 500,000; PS00,000
C. p 250,000; p 250,000
d. PO; P 0

11. Andro donated the following properties:


• P 2,000,000 car on November 2, 2018 to his favorite nephew
• P 3,000,000 agricultural lot on January 5, 2019 to his son who is getting
married
• P 4,000,000 building on February 14, 2019 to his father who is getting
married
• P 5,000,000 cash on July 1, 2019 designated by Andro for the
administrative expenses of a charitable institution
• P 1,000,000 cash on August 1, 2020 to his mother-in-law

Compute the total net gift subject to tax as of February 14, 2019.
a. p 8,750,000 C. p 4,000,000
b. P 6,750,000 d. P 3,750,000

649
C?,iil'"''. ,i,llil
,_._ ~;;')J,j
. 1:1· ,, 1
. 1,.:,-
cojSl
~
1'·
coll
1· 8•
-; 11:

s. rvtr-
wo
dOl
fili

Wl
a.
b.

p~
Pl
Net gift to strangers P 340,000 c.
Net gift to relatives 200,000 d
Total net gift P 540,000
10. A
Compute the total donor's tax due. •
a. p O C. p 17,400
b. P 2,400 d. P 20,400

4. In 2019, Andy made a total donation of P350,000 , inclusive of PS0,000
donations to the government for p ublic purpose . Compute t h e total donor's
tax due.
a. P 1,200 C. p 21 ,800
b. P 6,000 d. P 21,000

5. Derrick made th e following donations in 20 2 0 :


11.
January 5, 2019 - cash, to half-sister
August 17, 2019 - Car, to gi rl fri end P 200,000
November 8, 2 020 - Stocks, to granddaughter 800,000
400,000

650
n th Ja nu a ry S don a tion .
c. P 12,000
d. P ~ 0,000

iftt du on th Au gu t 17 donatlon.
. P 45 .000
.i d . P 6 0,000
C !),
t!t th gih tax on t he Nov mb r 8 don ati on.
C 111P o00 c. P 28, 000
a. ; 2 4,000 d. P 6 9,000
b,
J("rn Chi, a non -resid e nt Korean donor donated a . _ . h p .. .
~r.oru--":• pS00.' 000 to hi s
r esident Filipino best friend Mcar iCnh~ e . hil1pp111es
. · r. I paid P12,000
r , taX 10 Ko r ea on the donation. Korea do not im . d ,
do no .d t pose a onor s tax on
Filipino no n-res1 en s.

~,vhat is th e donor's tax still due in the Philippines?


a p 18,000 C. p 3,000
b. p 15,000 d. P 0

9_ Mr. Kinoto, a non-resident Japa?ese, donated an agricultural land in Japan


worth PB00,000 to a non-resident Filipina girlfriend. Mr. Kinoto paid
PZ0,000 donor's tax in Japan. What is the donor's tax due and payable in the
Philippines?
c. P 48,000 C. p 13,000
d. P 33,000 d. PO

10. Anon-resident citizen made the following donations in 2020:


• July 1, 2020 - P 600,000 cash, to a non-resident Japanese girlfriend, paid
Pl0,000 donor's tax abroad
• August 28, 2020 - P 900,000 cash as dowry to his daughter who is getting
married in the Philippines

What is the donor's tax to be paid respectively on the July donation and
August donation?
a. P 21,000; P 39,000 c. P 11,000; P 39,000
b. P 21,000; P 54,000 d. p 11,000; P 54,000

l l. What is the required donor's tax due and payable in the annual adjuS t ment
return?
a. PO c. p 10,000 refundable
b. P 10,000 payable d. p 20,000 refundable

651
Chapter 17 - Donor's Tax lc
12. A resident alien made the following donations during the year tor l f
e ath.,,,
'-'S·
Net gift in the Philippines (paid P 27,000 donor's tax] P 700,0oo · 1
N t gift abroad (paid P14,000 donor's tax abroad) ~
Total n t gift ~

What is the donor's tax due and payable?


a. P 45,000 c. P 4,000
b. P 4,500 d. P 0

13. Manso, resident citizen, donated several properties during the year:

Net gift in the Philippines p 0


Net gift made abroad 500,000
Total global net gift p 500,000 ~

Gift tax paid abroad p 10,000

What is the donor's tax due and payable in the Philippines?


a. PO c. P 7,500
b. PS,000 d. P 15,000

14. Armando, a resident citizen, made the following donations to relatives on


December 31, 2020:

Net gift Tax paid


Finland P 2,000,000 P 110,000
Norway 1,500,000 120,000
Philippines 500,000

There are no other donations during the year.

Compute the donor's tax due and payable after tax credit.
a. P 225,000 payable c. P 30,625 payable
b. P 74,000 payable d. P 5,000 refundble

15. The following relate to a non-resident alien donor:

Net gift Tax paid


Japan - relatives P 2,000,000 P 60,000
Malaysia - stranger 1,500,000 120,000
Philippines- relatives 500,000

Compute the donor's tax due and payable.


a. p O C. p 45,000
b. P 15,000 d. P 80,625

652

You might also like